Top Banner
VI. EXECUTIVE DEPARTMENT: THE PRESIDENT (Art. VII) PRIVILEGE AND SALARY (Sec. 6) b.1 Executive Immunity EN BANC [G.R. No. 82585. November 14, 1988.] MAXIMO V. SOLIVEN vs. THE HON. RAMON P. MAKASIAR Facts: Pres. Cory Aquino filed a criminal complaint for libel against Beltran Petitioner Beltran argues that "the reasons which necessitate presidential immunity from suit impose a correlative disability to file suit". He contends that if criminal proceedings ensue by virtue of the President's filing of her complaint-affidavit, she may subsequently have to be a witness for the prosecution, bringing her under the trial court's jurisdiction. This, continues Beltran, would in an indirect way defeat her privilege of immunity from suit, as by testifying on the witness stand, she would be exposing herself to possible contempt of court or perjury. Issues: 1. Whether or not the President of the Philippines, under the Constitution, may initiate criminal proceedings against the petitioners through the filing of a complaint-affidavit. If she may initiate, what are the repercussions of such initiation to her executive immunity? 2. Can Beltran invoke the executive immunity of the president as a defense? Held: First Issue: Yes. The rationale for the grant to the President of the privilege of immunity from suit is to assure the exercise of Presidential duties and functions free from any hindrance or distraction, considering that being the Chief Executive of the
158
Welcome message from author
This document is posted to help you gain knowledge. Please leave a comment to let me know what you think about it! Share it to your friends and learn new things together.
Transcript
Page 1: ARTICLE VII the executive department

VI. EXECUTIVE DEPARTMENT: THE PRESIDENT (Art. VII)

PRIVILEGE AND SALARY (Sec. 6)b.1 Executive Immunity

EN BANC[G.R. No. 82585. November 14, 1988.]

MAXIMO V. SOLIVEN vs. THE HON. RAMON P. MAKASIARFacts:

Pres. Cory Aquino filed a criminal complaint for libel against BeltranPetitioner Beltran argues that "the reasons which necessitate presidential immunity from suit

impose a correlative disability to file suit". He contends that if criminal proceedings ensue by virtue of the President's filing of her complaint-affidavit, she may subsequently have to be a witness for the prosecution, bringing her under the trial court's jurisdiction. This, continues Beltran, would in an indirect way defeat her privilege of immunity from suit, as by testifying on the witness stand, she would be exposing herself to possible contempt of court or perjury.

Issues: 1. Whether or not the President of the Philippines, under the Constitution, may initiate criminal

proceedings against the petitioners through the filing of a complaint-affidavit. If she may initiate, what are the repercussions of such initiation to her executive immunity?

2. Can Beltran invoke the executive immunity of the president as a defense?

Held:First Issue:Yes. The rationale for the grant to the President of the privilege of immunity from suit is to

assure the exercise of Presidential duties and functions free from any hindrance or distraction, considering that being the Chief Executive of the Government is a job that, aside from requiring all of the office-holder's time, also demands undivided attention.

Second Issue:No. But this privilege of immunity from suit, pertains to the President by virtue of the office

and may be invoked only by the holder of the office; not by any other person in the President's behalf Thus, an accused in a criminal case in which the President is complainant cannot raise the presidential privilege as a defense to prevent the case from proceeding against such accused.

Moreover, there is nothing in our laws that would prevent the President from waiving the privilege. Thus, if so minded the President may shed the protection afforded by the privilege and submit to the court's jurisdiction. The choice of whether to exercise the privilege or to waive it is solely the President's prerogative. It is a decision that cannot be assumed and imposed by any other person.

EN BANC

Page 2: ARTICLE VII the executive department

[G.R. Nos. 146710-15. April 3, 2001.]JOSEPH E. ESTRADA vs. ANIANO DESIERTO

Facts:Estrada argued that he cannot be criminally prosecuted because he is covered by the

executive immunity during the entire period of his term and that he should be removed through impeachment first before he can be prosecuted.

Issues:Whether the president is immune from suit during his term as president.Whether impeachment is a conditio sine qua non to his prosecution.

Held:

First Issue:No. This is in accord with our ruling in In re: Saturnino Bermudez that "incumbent Presidents

are immune from suit or from being brought to court during the period of their incumbency and tenure" but not beyond. Considering the peculiar circumstance that the impeachment process against the petitioner has been aborted and thereafter he lost the presidency, petitioner Estrada cannot demand as a condition sine qua non to his criminal prosecution before the Ombudsman that he be convicted in the impeachment proceedings.

Petitioner, however, fails to distinguish between term and tenure. The term means the time during which the officer may claim to hold the office as of right, and fixes the interval after which the several incumbents shall succeed one another. The tenure represents the term during which the incumbent actually holds office. The tenure may be shorter than the term for reasons within or beyond the power of the incumbent. 50 From the deliberations, the intent of the framers is clear that the immunity of the president from suit is concurrent only with his tenure and not his term.Indeed, petitioner's stubborn stance cannot but bolster the belief that the cases at bar were filed not really for petitioner to reclaim the presidency but just to take advantage of the immunity attached to the presidency and thus, derail the investigation of the criminal cases pending against him in the Office of the Ombudsman.

The cases filed against petitioner Estrada are criminal in character. They involve plunder, bribery and graft and corruption. By no stretch of the imagination can these crimes, especially plunder which carries the death penalty, be covered by the alleged mantle of immunity of a non-sitting president. Petitioner cannot cite any decision of this Court licensing the President to commit criminal acts and wrapping him with post-tenure immunity from liability. It will be anomalous to hold that immunity is an inoculation from liability for unlawful acts and omissions. The rule is that unlawful acts of public officials are not acts of the State and the officer who acts illegally is not acting as such but stands in the same footing as any other trespasser.

Second Issue:No. We reject his argument that he cannot be prosecuted for the reason that he must first be

convicted in the impeachment proceedings. The impeachment trial of petitioner Estrada was

Page 3: ARTICLE VII the executive department

aborted by the walkout of the prosecutors and by the events that led to his loss of the presidency. Indeed, on February 7, 2001, the Senate passed Senate Resolution No. 83 "Recognizing that the Impeachment Court is Functus Officio." Since the Impeachment Court is now functus officio, it is untenable for petitioner to demand that he should first be impeached and then convicted before he can be prosecuted. The plea if granted, would put a perpetual bar against his prosecution. Such a submission has nothing to commend itself for it will place him in a better situation than a non-sitting President who has not been subjected to impeachment proceedings and yet can be the object of a criminal prosecution. To be sure, the debates in the Constitutional Commission make it clear that when impeachment proceedings have become moot due to the resignation of the President, the proper criminal and civil cases may already be filed against him.

EN BANC[G.R. No. 152259. July 29, 2004.]ALFREDO T. ROMUALDEZ vs. The Honorable SANDIGANBAYAN

Facts:Petitioner was charged in an Information for violation of Section 5 of the Anti-Graft Law for

intervening in a sale between the National Shipyard and Steel Corporation, a government-owned and controlled corporation, and the Bataan Shipyard and Engineering Company, a private corporation, the majority stocks of which were allegedly owned by his brother-in-law, the late President Marcos. In this petition, petitioner alleged that the Sandiganbayan erred in not dismissing the criminal case on the ground that, among others, he enjoys derivative immunity, because he allegedly served as a high-ranking naval officer — specifically, as naval aide-de-camp — of former President Marcos.

Issue:Whether the petitioner is immune from criminal prosecution.

Held:No. In Estrada v. Desierto, this Court exhaustively traced the origin of executive immunity in

order to determine the extent of its applicability. We explained therein that executive immunity applied only during the incumbency of a President. It could not be used to shield a non-sitting President from prosecution for alleged criminal acts done while sitting in office. The reasoning of petitioner must therefore fail, since he derives his immunity from one who is no longer sitting as President. Verily, the felonious acts of public officials and their close relatives "are not acts of the State, and the officer who acts illegally is not acting as such but stands on the same footing as any other trespasser.

b.2. Executive Privilege

Page 4: ARTICLE VII the executive department

[G.R. No. 95367. May 23, 1995.] COMMISSIONER JOSE T. ALMONTE vs. vs. HONORABLE CONRADO M. VASQUEZ and CONCERNED CITIZENS

Facts:This is a petition for certiorari, prohibition, and mandamus to annul the subpoena duces

tecum and orders issued by respondent Ombudsman, requiring petitioners Nerio Rogado and Elisa Rivera, as chief accountant and record custodian, respectively, of the Economic Intelligence and Investigation Bureau (EIIB) to produce "all documents relating to Personal Services Funds for the year 1988 and all evidence such as vouchers (salary) for the whole plantilla of EIIB for 1988" and to enjoin him from enforcing his orders.

Petitioner Jose T. Almonte was formerly Commissioner of the EIIB, while Villamor C. Perez is Chief of the EIIB's Budget and Fiscal Management Division. The subpoena duces tecum was issued by the Ombudsman in connection with his investigation of an anonymous letter alleging that funds representing savings from unfilled positions in the EIIB had been illegally disbursed. The letter, purporting to have been written by an employee of the EIIB and a concerned citizen, was addressed to the Secretary of Finance, with copies furnished several government offices, including the Office of the Ombudsman. The petitioners questioned the issuance of the said subpoena on the ground that the documents asked for are classified. They argued that knowledge of EIIB's documents relative to its Personal Services Funds and its plantilla . . . will necessarily [lead to] knowledge of its operations, movements, targets, strategies, and tactics and the whole of its being" and this could "destroy the EIIB."

Issue:Whether petitioners can be ordered to produce documents relating to personal services and

salary vouchers of EIIB employees on the plea that such documents are classified.

Held:Yes. At common law a governmental privilege against disclosure is recognized with respect to

state secrets bearing on military, diplomatic and similar matters. This privilege is based upon public interest of such paramount importance as in and of itself transcending the individual interests of a private citizen, even though, as a consequence thereof, the plaintiff cannot enforce his legal rights.

The expectation of a President to the confidentiality of his conversations and correspondence, like the claim of confidentiality of judicial deliberations, for example, has all the values to which we accord deference for the privacy of all citizens and, added to those values, is the necessity for protection of the public interest in candid, objective, and even blunt or harsh opinions in Presidential decision-making. A President and those who assist him must be free to explore alternatives in the process of shaping policies and making decisions and to do so in a way many would be unwilling to express except privately. These are the considerations justifying a presumptive privilege for Presidential communications. The privilege is fundamental to the operation of the government and inextricably rooted in the separation of powers under the Constitution .

On the other hand, where the claim of confidentiality does not rest on the need to protect military, diplomatic or other national security secrets but on a general public interest in the

Page 5: ARTICLE VII the executive department

confidentiality of his conversations, courts have declined to find in the Constitution an absolute privilege of the President against a subpoena considered essential to the enforcement of criminal laws.

In the case at bar, there is no claim that military or diplomatic secrets will be disclosed by the production of records pertaining to the personnel of the EIIB. Indeed, EIIB's function is the gathering and evaluation of intelligence reports and information regarding "illegal activities affecting the national economy, such as, but not limited to, economic sabotage, smuggling, tax evasion, dollar salting." 18 Consequently, while in cases which involve state secrets it may be sufficient to determine from the circumstances of the case that there is reasonable danger that compulsion of the evidence will expose military matters without compelling production, 19 no similar excuse can be made for a privilege resting on other considerations.

Nor has our attention been called to any law or regulation which considers personnel records of the EIIB as classified information. To the contrary, COA Circular No. 88-293, which petitioners invoke to support their contention that there is adequate safeguard against misuse of public funds, provides that the "only item of expenditure which should be treated strictly confidential" is that which refers to the "purchase of information and payment of rewards."

The other statutes and regulations invoked by petitioners in support of their contention that the documents sought in the subpoena duces tecum of the Ombudsman are classified merely indicate the confidential nature of the EIIB's functions, but they do not exempt the EIIB from the duty to account for its funds to the proper authorities. Indeed by denying that there were savings made from certain items in the agency and alleging that the DBM had released to the EIIB only the allocations needed for the 947 personnel retained after its reorganization, petitioners in effect invited inquiry into the veracity of their claim. If, as petitioners claim, the subpoenaed records have been examined by the COA and found by it to be regular in all respects, there is no reason why they cannot be shown to another agency of the government which by constitutional mandate is required to look into any complaint concerning public office.

Above all, even if the subpoenaed documents are treated as presumptively privileged, this decision would only justify ordering their inspection in camera but not their nonproduction. However, as concession to the nature of the functions of the EIIB and just to be sure no information of a confidential character is disclosed, the examination of records in this case should be made in strict confidence by the Ombudsman himself. Reference may be made to the documents in any decision or order which the Ombudsman may render or issue but only to the extent that it will not reveal covert activities of the agency. Above all, there must be a scrupulous protection of the documents delivered.

With these safeguards outlined, it is believed that a satisfactory resolution of the conflicting claims of the parties is achieved, It is not amiss to state that even matters of national security have been inquired into in appropriate in camera proceedings by the courts. In Lansang v. Garcia this Court held closed door sessions, with only the immediate parties and their counsel present, to determine claims that because of subversion there was imminent danger to public safety warranting the suspension of the writ of habeas corpus in 1971. Again in Marcos v. Manglapus the Court met behind closed doors to receive military briefings on the threat posed to national security by the return to the country of the former President and his family. In the United States, a similar inquiry into the danger to national security as a result of the publication of classified documents on

Page 6: ARTICLE VII the executive department

the Vietnam war was upheld by the U.S. Supreme Court. We see no reason why similar safeguards cannot be made to enable an agency of the Government, like the Office of the Ombudsman, to carry out its constitutional duty to protect public interests while insuring the confidentiality of classified documents.

EN BANC [G.R. No. 169777. * April 20, 2006.] SENATE OF THE PHILIPPINES vs. EDUARDO R. ERMITAFacts:The petitioners alleged that the President has abused its power by issuing Executive Order No. 464 (E.O. 464) last September 28, 2005. They thus pray for its declaration as null and void for being unconstitutional. On September 21 to 23, 2005, the Committee of the Senate as a whole issued invitations to various officials of the Executive Department for them to appear on September 29, 2005 as resource speakers in a public hearing on the railway project of the North Luzon Railways Corporation with the China National Machinery and Equipment Group (hereinafter North Rail Project). On September 28, 2005, the President issued E.O. 464 which required all heads of departments of the Executive Branch of the government shall secure the consent of the President prior to appearing before either House of Congress. It also invoked the principle of separation of powers and executive privilege.

Author’s Note: I will reproduce some of the salient provisions of the EO 464 for the reader’s guidance and reference on the principles lengthtily discussed in the SC’s ruling.

E.O. 464, "ENSURING OBSERVANCE OF THE PRINCIPLE OF SEPARATION OF POWERS, ADHERENCE TO THE RULE ON EXECUTIVE PRIVILEGE AND RESPECT FOR THE RIGHTS OF PUBLIC OFFICIALS APPEARING IN LEGISLATIVE INQUIRIES IN AID OF LEGISLATION UNDER THE CONSTITUTION, AND FOR OTHER PURPOSES

SECTION 1. Appearance by Heads of Departments Before Congress. — In accordance with Article VI, Section 22 of the Constitution and to implement the Constitutional provisions on the separation of powers between co-equal branches of the government, all heads of departments of the Executive Branch of the government shall secure the consent of the President prior to appearing before either House of Congress.

When the security of the State or the public interest so requires and the President so states in writing, the appearance shall only be conducted in executive session.

SECTION. 2. Nature, Scope and Coverage of Executive Privilege. —(a) Nature and Scope. — The rule of confidentiality based on executive privilege is fundamental to the operation of government and rooted in the separation of powers under the Constitution (Almonte vs. Vasquez, G.R. No. 95367, 23 May 1995). Further, Republic Act No. 6713 or the Code of Conduct and Ethical Standards for Public Officials and Employees provides that Public Officials and Employees shall not use or divulge confidential or classified information officially known to them by reason of their office and not made available to the public to prejudice the public interest.

Page 7: ARTICLE VII the executive department

Executive privilege covers all confidential or classified information between the President and the public officers covered by this executive order, including:i. Conversations and correspondence between the President and the public official covered by this executive order (Almonte vs. Vasquez G.R. No. 95367, 23 May 1995; Chavez v. Public Estates Authority, G.R. No. 133250, 9 July 2002);ii. Military, diplomatic and other national security matters which in the interest of national security should not be divulged (Almonte vs. Vasquez, G.R. No. 95367, 23 May 1995; Chavez v. Presidential Commission on Good Government, G.R. No. 130716, 9 December 1998).iii. Information between inter-government agencies prior to the conclusion of treaties and executive agreements (Chavez v. Presidential Commission on Good Government, G.R. No. 130716, 9 December 1998);iv. Discussion in close-door Cabinet meetings (Chavez v. Presidential Commission on Good Government, G.R. No. 130716, 9 December 1998);v. Matters affecting national security and public order (Chavez v. Public Estates Authority, G.R. No. 133250, 9 July 2002).(b) Who are covered. — The following are covered by this executive order:i. Senior officials of executive departments who in the judgment of the department heads are covered by the executive privilege;ii. Generals and flag officers of the Armed Forces of the Philippines and such other officers who in the judgment of the Chief of Staff are covered by the executive privilege; EaCSTciii. Philippine National Police (PNP) officers with rank of chief superintendent or higher and such other officers who in the judgment of the Chief of the PNP are covered by the executive privilege;iv. Senior national security officials who in the judgment of the National Security Adviser are covered by the executive privilege; andv. Such other officers as may be determined by the President.

SECTION 3. Appearance of Other Public Officials Before Congress. — All public officials enumerated in Section 2 (b) hereof shall secure prior consent of the President prior to appearing before either House of Congress to ensure the observance of the principle of separation of powers, adherence to the rule on executive privilege and respect for the rights of public officials appearing in inquiries in aid of legislation.

Issue:Whether the executive privilege is properly invoked in this case.

Held:No. Even where the inquiry is in aid of legislation, there are still recognized exemptions to the

power of inquiry, which exemptions fall under the rubric of "executive privilege." Since this term figures prominently in the challenged order, it being mentioned in its provisions, its preambular clauses, and in its very title, a discussion of executive privilege is crucial for determining the constitutionality of E.O. 464.

While executive privilege is a constitutional concept, a claim thereof may be valid or not depending on the ground invoked to justify it and the context in which it is made. Noticeably

Page 8: ARTICLE VII the executive department

absent is any recognition that executive officials are exempt from the duty to disclose information by the mere fact of being executive officials. Indeed, the extraordinary character of the exemptions indicates that the presumption inclines heavily against executive secrecy and in favor of disclosure.

When Congress exercises its power of inquiry, the only way for department heads to exempt themselves therefrom is by a valid claim of privilege. They are not exempt by the mere fact that they are department heads. Only one executive official may be exempted from this power — the President on whom executive power is vested, hence, beyond the reach of Congress except through the power of impeachment. It is based on her being the highest official of the executive branch, and the due respect accorded to a co-equal branch of government which is sanctioned by a long-standing custom.

Section 3 of E.O. 464 requires all the public officials enumerated in Section 2(b) to secure the consent of the President prior to appearing before either house of Congress. The enumeration is broad. It covers all senior officials of executive departments, all officers of the AFP and the PNP, and all senior national security officials who, in the judgment of the heads of offices designated in the same section (i.e. department heads, Chief of Staff of the AFP, Chief of the PNP, and the National Security Adviser), are "covered by the executive privilege."

The enumeration also includes such other officers as may be determined by the President. Given the title of Section 2 — "Nature, Scope and Coverage of Executive Privilege" —, it is evident that under the rule of ejusdem generis, the determination by the President under this provision is intended to be based on a similar finding of coverage under executive privilege.

En passant, the Court notes that Section 2(b) of E.O. 464 virtually states that executive privilege actually covers persons. Such is a misuse of the doctrine. Executive privilege, as discussed above, is properly invoked in relation to specific categories of information and not to categories of persons.In light, however, of Sec 2(a) of E.O. 464 which deals with the nature, scope and coverage of executive privilege, the reference to persons being "covered by the executive privilege" may be read as an abbreviated way of saying that the person is in possession of information which is, in the judgment of the head of office concerned, privileged as defined in Section 2(a). The Court shall thus proceed on the assumption that this is the intention of the challenged order.

The letter dated September 28, 2005 of respondent Executive Secretary Ermita to Senate President Drilon illustrates the implied nature of the claim of privilege authorized by E.O. 464. It reads:In connection with the inquiry to be conducted by the Committee of the Whole regarding the Northrail Project of the North Luzon Railways Corporation on 29 September 2005 at 10:00 a.m., please be informed that officials of the Executive Department invited to appear at the meeting will not be able to attend the same without the consent of the President, pursuant to Executive Order No. 464 (s. 2005), entitled "Ensuring Observance Of The Principle Of Separation Of Powers, Adherence To The Rule On Executive Privilege And Respect For The Rights Of Public Officials Appearing In Legislative Inquiries In Aid Of Legislation Under The Constitution, And For Other Purposes". Said officials have not secured the required consent from the President.

While there is no Philippine case that directly addresses the issue of whether executive privilege may be invoked against Congress, it is gathered from Chavez v. PEA that certain

Page 9: ARTICLE VII the executive department

information in the possession of the executive may validly be claimed as privileged even against Congress. Thus, the case holds:

There is no claim by PEA that the information demanded by petitioner is privileged information rooted in the separation of powers. The information does not cover Presidential conversations, correspondences, or discussions during closed-door Cabinet meetings which, like internal-deliberations of the Supreme Court and other collegiate courts, or executive sessions of either house of Congress, are recognized as confidential. This kind of information cannot be pried open by a co-equal branch of government. A frank exchange of exploratory ideas and assessments, free from the glare of publicity and pressure by interested parties, is essential to protect the independence of decision-making of those tasked to exercise Presidential, Legislative and Judicial power. This is not the situation in the instant case.While the validity of claims of privilege must be assessed on a case to case basis, examining

the ground invoked therefor and the particular circumstances surrounding it, there is, in an implied claim of privilege, a defect that renders it invalid per se. By its very nature, and as demonstrated by the letter of respondent Executive Secretary quoted above, the implied claim authorized by Section 3 of E.O. 464 is not accompanied by any specific allegation of the basis thereof (e.g., whether the information demanded involves military or diplomatic secrets, closed-door Cabinet meetings, etc.). While Section 2(a) enumerates the types of information that are covered by the privilege under the challenged order, Congress is left to speculate as to which among them is being referred to by the executive. The enumeration is not even intended to be comprehensive, but a mere statement of what is included in the phrase "confidential or classified information between the President and the public officers covered by this executive order."Certainly, Congress has the right to know why the executive considers the requested information privileged. It does not suffice to merely declare that the President, or an authorized head of office, has determined that it is so, and that the President has not overturned that determination. Such declaration leaves Congress in the dark on how the requested information could be classified as privileged. That the message is couched in terms that, on first impression, do not seem like a claim of privilege only makes it more pernicious. It threatens to make Congress doubly blind to the question of why the executive branch is not providing it with the information that it has requested.

A claim of privilege, being a claim of exemption from an obligation to disclose information, must, therefore, be clearly asserted. As U.S. v. Reynolds teaches:

The privilege belongs to the government and must be asserted by it; it can neither be claimed nor waived by a private party. It is not to be lightly invoked. There must be a formal claim of privilege, lodged by the head of the department which has control over the matter, after actual personal consideration by that officer. The court itself must determine whether the circumstances are appropriate for the claim of privilege, and yet do so without forcing a disclosure of the very thing the privilege is designed to protect.Absent then a statement of the specific basis of a claim of executive privilege, there is no way

of determining whether it falls under one of the traditional privileges, or whether, given the circumstances in which it is made, it should be respected. These, in substance, were the same criteria in assessing the claim of privilege asserted against the Ombudsman in Almonte v. Vasquez 94 and, more in point, against a committee of the Senate in Senate Select Committee on Presidential Campaign Activities v. Nixon.

Page 10: ARTICLE VII the executive department

A.O. Smith v. Federal Trade Commission is enlightening:[T]he lack of specificity renders an assessment of the potential harm resulting from

disclosure impossible, thereby preventing the Court from balancing such harm against plaintiffs' needs to determine whether to override any claims of privilege.And so is U.S. v. Article of Drug:

On the present state of the record, this Court is not called upon to perform this balancing operation. In stating its objection to claimant's interrogatories, government asserts, and nothing more, that the disclosures sought by claimant would inhibit the free expression of opinion that non-disclosure is designed to protect. The government has not shown — nor even alleged — that those who evaluated claimant's product were involved in internal policymaking, generally, or in this particular instance. Privilege cannot be set up by an unsupported claim. The facts upon which the privilege is based must be established. To find these interrogatories objectionable, this Court would have to assume that the evaluation and classification of claimant's products was a matter of internal policy formulation, an assumption in which this Court is unwilling to indulge sua sponte.

Mobil Oil Corp. v. Department of Energy 99 similarly emphasizes that "an agency must provide 'precise and certain' reasons for preserving the confidentiality of requested information."Black v. Sheraton Corp. of America 100 amplifies, thus:A formal and proper claim of executive privilege requires a specific designation and description of the documents within its scope as well as precise and certain reasons for preserving their confidentiality. Without this specificity, it is impossible for a court to analyze the claim short of disclosure of the very thing sought to be protected. As the affidavit now stands, the Court has little more than its sua sponte speculation with which to weigh the applicability of the claim. An improperly asserted claim of privilege is no claim of privilege. Therefore, despite the fact that a claim was made by the proper executive as Reynolds requires, the Court can not recognize the claim in the instant case because it is legally insufficient to allow the Court to make a just and reasonable determination as to its applicability. To recognize such a broad claim in which the Defendant has given no precise or compelling reasons to shield these documents from outside scrutiny, would make a farce of the whole procedure.

Due respect for a co-equal branch of government, moreover, demands no less than a claim of privilege clearly stating the grounds therefor. Apropos is the following ruling in McPhaul v. U.S:

We think the Court's decision in United States v. Bryan, 339 U.S. 323, 70 S. Ct. 724, is highly relevant to these questions. For it is as true here as it was there, that 'if (petitioner) had legitimate reasons for failing to produce the records of the association, a decent respect for the House of Representatives, by whose authority the subpoenas issued, would have required that (he) state (his) reasons for noncompliance upon the return of the writ. Such a statement would have given the Subcommittee an opportunity to avoid the blocking of its inquiry by taking other appropriate steps to obtain the records. 'To deny the Committee the opportunity to consider the objection or remedy is in itself a contempt of its authority and an obstruction of its processes. His failure to make any such statement was "a patent evasion of the duty of one summoned to produce papers before a congressional committee[, and] cannot be condoned."

Upon the other hand, Congress must not require the executive to state the reasons for the claim with such particularity as to compel disclosure of the information which the privilege is meant to

Page 11: ARTICLE VII the executive department

protect. A useful analogy in determining the requisite degree of particularity would be the privilege against self-incrimination.

The claim of privilege under Section 3 of E.O. 464 in relation to Section 2(b) is thus invalid per se. It is not asserted. It is merely implied. Instead of providing precise and certain reasons for the claim, it merely invokes E.O. 464, coupled with an announcement that the President has not given her consent. It is woefully insufficient for Congress to determine whether the withholding of information is justified under the circumstances of each case. It severely frustrates the power of inquiry of Congress.

Petitioner Senate of the Philippines, in particular, cites the case of the United States where, so it claims, only the President can assert executive privilege to withhold information from Congress.Section 2(b) in relation to Section 3 virtually provides that, once the head of office determines that a certain information is privileged, such determination is presumed to bear the President's authority and has the effect of prohibiting the official from appearing before Congress, subject only to the express pronouncement of the President that it is allowing the appearance of such official. These provisions thus allow the President to authorize claims of privilege by mere silence.Such presumptive authorization, however, is contrary to the exceptional nature of the privilege. Executive privilege, as already discussed, is recognized with respect to information the confidential nature of which is crucial to the fulfillment of the unique role and responsibilities of the executive branch, 105 or in those instances where exemption from disclosure is necessary to the discharge of highly important executive responsibilities. 106 The doctrine of executive privilege is thus premised on the fact that certain informations must, as a matter of necessity, be kept confidential in pursuit of the public interest. The privilege being, by definition, an exemption from the obligation to disclose information, in this case to Congress, the necessity must be of such high degree as to outweigh the public interest in enforcing that obligation in a particular case.In light of this highly exceptional nature of the privilege, the Court finds it essential to limit to the President the power to invoke the privilege. She may of course authorize the Executive Secretary to invoke the privilege on her behalf, in which case the Executive Secretary must state that the authority is "By order of the President," which means that he personally consulted with her. The privilege being an extraordinary power, it must be wielded only by the highest official in the executive hierarchy. In other words, the President may not authorize her subordinates to exercise such power. There is even less reason to uphold such authorization in the instant case where the authorization is not explicit but by mere silence. Section 3, in relation to Section 2(b), is further invalid on this score. It follows, therefore, that when an official is being summoned by Congress on a matter which, in his own judgment, might be covered by executive privilege, he must be afforded reasonable time to inform the President or the Executive Secretary of the possible need for invoking the privilege. This is necessary in order to provide the President or the Executive Secretary with fair opportunity to consider whether the matter indeed calls for a claim of executive privilege. If, after the lapse of that reasonable time, neither the President nor the Executive Secretary invokes the privilege, Congress is no longer bound to respect the failure of the official to appear before Congress and may then opt to avail of the necessary legal means to compel his appearance.

Page 12: ARTICLE VII the executive department

In light of the above discussion of Section 3, it is clear that it is essentially an authorization for implied claims of executive privilege, for which reason it must be invalidated. That such authorization is partly motivated by the need to ensure respect for such officials does not change the infirm nature of the authorization itself.

EN BANC [G.R. No. 180643. March 25, 2008.] ROMULO L. NERI vs. SENATE COMMITTEE ON ACCOUNTABILITY OF PUBLIC OFFICERS AND INVESTIGATIONS

Facts:On April 21, 2007, the Department of Transportation and Communication (DOTC) entered

into a contract with Zhing Xing Telecommunications Equipment (ZTE) for the supply of equipment and services for the National Broadband Network (NBN) Project in the amount of U.S. $329,481,290 (approximately P16 Billion Pesos). The Project was to be financed by the People's Republic of China. Respondent Committees initiated the investigation, in aid od legislation, by sending invitations to certain personalities and cabinet officials involved in the NBN Project. Petitioner was among those invited. In particular, he refused to answer, invoking executive privilege, the questions on (a) whether or not President Arroyo followed up the NBN Project, 6 (b) whether or not she directed him to prioritize it, and (c) whether or not she directed him to approve. In a letter sent to the Senate, Ermita justified the claim of executive privilege saying that the information sought to be disclosed might impair our diplomatic as well as economic relations with the People's Republic of China. Given the confidential nature in which these information were conveyed to the President, he (Neri) cannot provide the Committee any further details of these conversations, without disclosing the very thing the privilege is designed to protect.

Issue:Whether the Senate can compel Neri to disclose the information asked for.

Held: No, because those informations are covered by executive privilege and the same was

properly invoked by Ermita. At this juncture, it must be stressed that the revocation of E.O. 464 does not in any way diminish our concept of executive privilege. This is because this concept has Constitutional underpinnings. Unlike the United States which has further accorded the concept with statutory status by enacting the Freedom of Information Act 23 and the Federal Advisory Committee Act, 24 the Philippines has retained its constitutional origination, occasionally interpreted only by this Court in various cases. The most recent of these is the case of Senate v. Ermita where this Court declared unconstitutional substantial portions of E.O. 464.

In In re: Sealed Case, 30 the U.S. Court of Appeals delved deeper. It ruled that there are two (2) kinds of executive privilege; one is the presidential communications privilege and, the other is the deliberative process privilege. The former pertains to "communications, documents or other materials that reflect presidential decision-making and deliberations and that the President believes should remain confidential." The latter includes 'advisory opinions, recommendations and deliberations comprising part of a process by which governmental decisions and policies are formulated."

Page 13: ARTICLE VII the executive department

Accordingly, they are characterized by marked distinctions. Presidential communications privilege applies to decision-making of the President while, the deliberative process privilege, to decision-making of executive officials. The first is rooted in the constitutional principle of separation of power and the President's unique constitutional role; the second on common law privilege. Unlike the deliberative process privilege, the presidential communications privilege applies to documents in their entirety, and covers final and post-decisional materials as well as pre-deliberative ones 31 As a consequence, congressional or judicial negation of the presidential communications privilege is always subject to greater scrutiny than denial of the deliberative process privilege.

Turning on who are the officials covered by the presidential communications privilege, In Re Sealed Case confines the privilege only to White House Staff that has "operational proximity" to direct presidential decision-making. Thus, the privilege is meant to encompass only those functions that form the core of presidential authority, involving what the court characterized as "quintessential and non-delegable Presidential power", such as commander-in-chief power, appointment and removal power, the power to grant pardons and reprieves, the sole-authority to receive ambassadors and other public officers, the power to negotiate treaties etc.In Chavez v. PCGG, 38 this Court held that there is a "governmental privilege against public disclosure with respect to state secrets regarding military, diplomatic and other security matters." In Chavez v. PEA, 39 there is also a recognition of the confidentiality of Presidential conversations, correspondences, and discussions in closed-door Cabinet meetings. In Senate v. Ermita, the concept of presidential communications privilege is fully discussed. EHcaAIAs may be gleaned from the above discussion, the claim of executive privilege is highly recognized in cases where the subject of inquiry relates to a power textually committed by the Constitution to the President, such as the area of military and foreign relations. Under our Constitution, the President is the repository of the commander-in-chief, appointing, pardoning, and diplomatic powers. Consistent with the doctrine of separation of powers, the information relating to these powers may enjoy greater confidentiality than others.

The above cases, especially, Nixon, In Re Sealed Case and Judicial Watch, somehow provide the elements of presidential communications privilege, to wit:1) The protected communication must relate to a "quintessential and non-delegable presidential power."2) The communication must be authored or "solicited and received" by a close advisor of the President or the President himself. The judicial test is that an advisor must be in "operational proximity" with the President. 3) The presidential communications privilege remains a qualified privilege that may be overcome by a showing of adequate need, such that the information sought "likely contains important evidence" and by the unavailability of the information elsewhere by an appropriate investigating authority. In the case at bar, Executive Secretary Ermita premised his claim of executive privilege on the ground that the communications elicited by the three (3) questions "fall under conversation and correspondence between the President and public officials" necessary in "her executive and policy decision-making process" and, that "the information sought to be disclosed might impair our diplomatic as well as economic relations with the People's Republic of China." Simply put, the bases

Page 14: ARTICLE VII the executive department

are presidential communications privilege and executive privilege on matters relating to diplomacy or foreign relations.Using the above elements, we are convinced that, indeed, the communications elicited by the three (3) questions are covered by the presidential communications privilege. First, the communications relate to a "quintessential and non-delegable power" of the President, i.e. the power to enter into an executive agreement with other countries. This authority of the President to enter into executive agreements without the concurrence of the Legislature has traditionally been recognized in Philippine jurisprudence. Second, the communications are "received" by a close advisor of the President. Under the "operational proximity" test, petitioner can be considered a close advisor, being a member of President Arroyo's cabinet. And third, there is no adequate showing of a compelling need that would justify the limitation of the privilege and of the unavailability of the information elsewhere by an appropriate investigating authority.

The third element deserves a lengthy discussion.The foregoing is consistent with the earlier case of Nixon vs. Sirica, where it was held that

presidential communications privilege are presumptively privileged and that the presumption can be overcome only by mere showing of public need by the branch seeking access to conversations. The courts are enjoined to resolve the competing interests of the political branches of the government "in the manner that preserves the essential functions of each Branch." Here, the record is bereft of any categorical explanation from respondent Committees to show a compelling or critical need for the answers to the three (3) questions in the enactment of a law. Instead, the questions veer more towards the exercise of the legislative oversight function under Section 22 of Article VI rather than Section 21 of the same Article. Senate v. Ermita ruled that the "the oversight function of Congress may be facilitated by compulsory process only to the extent that it is performed in pursuit of legislation." It is conceded that it is difficult to draw the line between an inquiry in aid of legislation and an inquiry in the exercise of oversight function of Congress. In this regard, much will depend on the content of the questions and the manner the inquiry is conducted.

In this regard, Senate v. Ermita stressed that the validity of the claim of executive privilege depends not only on the ground invoked but, also, the procedural setting or the context in which the claim is made.

The respondent Committees should cautiously tread into the investigation of matters which may present a conflict of interest that may provide a ground to inhibit the Senators participating in the inquiry if later on an impeachment proceeding is initiated on the same subject matter of the present Senate inquiry. Pertinently, in Senate Select Committee on Presidential Campaign Activities v. Nixon, it was held that since an impeachment proceeding had been initiated by a House Committee, the Senate Select Committee's immediate oversight need for five presidential tapes, should give way to the House Judiciary Committee which has the constitutional authority to inquire into presidential impeachment. The Court expounded on this issue in this wise:

It is true, of course, that the Executive cannot, any more than the other branches of government, invoke a general confidentiality privilege to shield its officials and employees from investigations by the proper governmental institutions into possible criminal wrongdoing. The Congress learned this as to its own privileges in Gravel v. United States, as did the judicial branch, in a sense, in Clark v. United States, and the executive branch itself in Nixon v. Sirica. But under Nixon v. Sirica, the showing required to overcome the presumption favoring confidentiality turned, not on

Page 15: ARTICLE VII the executive department

the nature of the presidential conduct that the subpoenaed material might reveal, but, instead, on the nature and appropriateness of the function in the performance of which the material was sought, and the degree to which the material was necessary to its fulfillment. Here also our task requires and our decision implies no judgment whatever concerning possible presidential involvement in culpable activity. On the contrary, we think the sufficiency of the Committee's showing must depend solely on whether the subpoenaed evidence is demonstrably critical to the responsible fulfillment of the Committee's functions.

Jurisprudence teaches that for the claim to be properly invoked, there must be a formal claim of privilege, lodged by the head of the department which has control over the matter." A formal and proper claim of executive privilege requires a "precise and certain reason" for preserving their confidentiality.

The Letter dated November 17, 2007 of Executive Secretary Ermita satisfies the requirement. It serves as the formal claim of privilege. There he expressly states that "this Office is constrained to invoke the settled doctrine of executive privilege as refined in Senate v. Ermita, and has advised Secretary Neri accordingly." Obviously, he is referring to the Office of the President. That is more than enough compliance. In Senate v. Ermita, a less categorical letter was even adjudged to be sufficient.With regard to the existence of "precise and certain reason", we find the grounds relied upon by Executive Secretary Ermita specific enough so as not "to leave respondent Committees in the dark on how the requested information could be classified as privileged." The case of Senate v. Ermita only requires that an allegation be made "whether the information demanded involves military or diplomatic secrets, closed-door Cabinet meetings, etc." The particular ground must only be specified. The enumeration is not even intended to be comprehensive." The following statement of grounds satisfies the requirement:

The context in which executive privilege is being invoked is that the information sought to be disclosed might impair our diplomatic as well as economic relations with the People's Republic of China. Given the confidential nature in which these information were conveyed to the President, he cannot provide the Committee any further details of these conversations, without disclosing the very thing the privilege is designed to protect.At any rate, as held further in Senate v. Ermita, the Congress must not require the executive

to state the reasons for the claim with such particularity as to compel disclosure of the information which the privilege is meant to protect. This is a matter of respect to a coordinate and co-equal department.

A. PROHIBITIONS (Sec 13)

EN BANC [G.R. No. 85468. September 7, 1989.] QUINTIN S. DOROMAL vs. SANDIGANBAYAN

Facts:In October 1987, Special Prosecution Officer II, Dionisio A. Caoili, conducted a preliminary

investigation of the charge against the petitioner, Quintin S. Doromal, a former Commissioner of the Presidential Commission on Good Government (PCGG), for violation of the Anti-Graft and

Page 16: ARTICLE VII the executive department

Corrupt Practices Act (RA 3019), Sec. 3(h), in connection with his shareholdings and position as president and director of the Doromal International Trading Corporation (DITC) which submitted bids to supply P61 million worth of electronic, electrical, automotive, mechanical and airconditioning equipment to the Department of Education, Culture and Sports (or DECS) and the National Manpower and Youth Council (or NMYC), which act is absolutely prohibited by Section 13 of Article VII of the Constitution.

Issue:Whether the said act violates the constitution.

Held:Yes. Section 13, Article VII of the 1987 Constitution provides that "the President, Vice-

President, the members of the Cabinet and their deputies or assistants shall not . . . during (their) tenure, . . . directly or indirectly . . . participate in any business." The constitutional ban is similar to the prohibition in the Civil Service Law (PD No. 807, Sec. 36, subpar. 24) that "pursuit of private business . . . without the permission required by Civil Service Rules and Regulations" shall be a ground for disciplinary action against any officer or employee in the civil service.

Author’s Note: This case is more of a Criminal Procedure than a Political Law case.

EN BANC[G.R. No. 83896. February 22, 1991.]CIVIL LIBERTIES UNION vs. THE EXECUTIVE SECRETARY

Facts:The petitioners sought a declaration of the unconstitutionality of Executive Order No. 284

issued by President Corazon C. Aquino on July 25, 1987. The pertinent provisions of the assailed Executive Order are:

"SECTION 1. Even if allowed by law or by the ordinary functions of his position, a member of the Cabinet, undersecretary or assistant secretary or other appointive officials of the Executive Department may, in addition to his primary position, hold not more than two positions in the government and government corporations and receive the corresponding compensation therefor; Provided, that this limitation shall not apply to ad hoc bodies or committees, or to boards, councils or bodies of which the President is the Chairman."SECTION 2. If a member of the cabinet, undersecretary or assistant secretary or other appointive official of the Executive Department holds more positions than what is allowed in Section 1 hereof, they (sic) must relinquish the excess position in favor of the subordinate official who is next in rank, but in no case shall any official hold more than two positions other than his primary position.

Page 17: ARTICLE VII the executive department

"SECTION 3. In order to fully protect the interest of the government in government-owned or controlled corporations, at least one-third (1/3) of the members of the boards of such corporation should either be a secretary, or undersecretary, or assistant secretary."

Petitioners maintain that this Executive Order which, in effect, allows members of the Cabinet, their undersecretaries and assistant secretaries to hold other government offices or positions in addition to their primary positions, albeit subject to the limitation therein imposed, runs counter to Section 13, Article VII of the 1987 Constitution.

In sum, the constitutionality of Executive Order No. 284 is being challenged by petitioners on the principal submission that it adds exceptions to Section 13, Article VII other than those provided in the Constitution. According to petitioners, by virtue of the phrase "unless otherwise provided in this Constitution," the only exceptions against holding any other office or employment in Government are those provided in the Constitution, namely: (1) The Vice-President may be appointed as a Member of the Cabinet under Section 3, par. (2), Article VII thereof; and (2) the Secretary of Justice is an ex-officio member of the Judicial and Bar Council by virtue of Section 8 (1), Article VIII.

Public respondents, on the other hand, maintain that the phrase "unless otherwise provided in the Constitution" in Section 13, Article VII makes reference to Section 7, par. (2), Article IX-B insofar as the appointive officials mentioned therein are concerned.

Issue:Does the prohibition in Section 13, Article VII of the 1987 Constitution insofar as Cabinet

members, their deputies or assistants are concerned admit of the broad exceptions made for appointive officials in general under Section 7, par. (2), Article IX-B which, for easy reference is quoted anew, thus: "Unless otherwise allowed by law or by the primary functions of his position, no appointive official shall hold any other office or employment in the Government or any subdivision, agency or instrumentality thereof, including government-owned or controlled corporation or their subsidiaries."

Held:No. A foolproof yardstick in constitutional construction is the intention underlying the

provision under consideration. Thus, it has been held that the Court in construing a Constitution should bear in mind the object sought to be accomplished by its adoption, and the evils, if any, sought to be prevented or remedied. A doubtful provision will be examined in the light of the history of the times, and the condition and circumstances under which the Constitution was framed. The object is to ascertain the reason which induced the framers of the Constitution to enact the particular provision and the purpose sought to be accomplished thereby, in order to construe the whole as to make the words consonant to that reason and calculated to effect that purpose.

The blatant betrayal of public trust evolved into one of the serious causes of discontent with the Marcos regime. It was therefore quite inevitable and in consonance with the overwhelming sentiment of the people that the 1986 Constitutional Commission, convened as it was after the people successfully unseated former President Marcos, should draft into its proposed Constitution

Page 18: ARTICLE VII the executive department

the provisions under consideration which are envisioned to remedy, if not correct, the evils that flow from the holding of multiple governmental offices and employment. In fact, as keenly observed by Mr. Justice Isagani A. Cruz during the deliberations in these cases, one of the strongest selling points of the 1987 Constitution during the campaign for its ratification was the assurance given by its proponents that the scandalous practice of Cabinet members holding multiple positions in the government and collecting unconscionably excessive compensation therefrom would be discontinued.

But what is indeed significant is the fact that although Section 7, Article IX-B already contains a blanket prohibition against the holding of multiple offices or employment in the government subsuming both elective and appointive public officials, the Constitutional Commission should see it fit to formulate another provision, Sec. 13, Article VII, specifically prohibiting the President, Vice-President, members of the Cabinet, their deputies and assistants from holding any other office or employment during their tenure, unless otherwise provided in the Constitution itself.

Evidently, from this move as well as in the different phraseologies of the constitutional provisions in question, the intent of the framers of the Constitution was to impose a stricter prohibition on the President and his official family in so far as holding other offices or employment in the government or elsewhere is concerned.

Moreover, such intent is underscored by a comparison of Section 13, Article VII with other provisions of the Constitution on the disqualifications of certain public officials or employees from holding other offices or employment. Under Section 13, Article VII, "(N)o Senator or Member of the House of Representatives may hold any other office or employment in the Government. . .". Under Section 5(4), Article XVI, "(N)o member of the armed forces in the active service shall, at any time, be appointed in any capacity to a civilian position in the Government, including government-owned or controlled corporations or any of their subsidiaries." Even Section 7 (2), Article IX-B, relied upon by respondents provides "(U)nless otherwise allowed by law or by the primary functions of his position, no appointive official shall hold any other office or employment in the Government."It is quite notable that in all these provisions on disqualifications to hold other office or employment, the prohibition pertains to an office or employment in the government and government-owned or controlled corporations or their subsidiaries. In striking contrast is the wording of Section 13, Article VII which states that "(T)he President, Vice-President, the Members of the Cabinet, and their deputies or assistants shall not, unless otherwise provided in this Constitution, hold any other office or employment during their tenure." In the latter provision, the disqualification is absolute, not being qualified by the phrase "in the Government." The prohibition imposed on the President and his official family is therefore all-embracing and covers both public and private office or employment. cdll

Going further into Section 13, Article VII, the second sentence provides: "They shall not, during said tenure, directly or indirectly, practice any other profession, participate in any business, or be financially interested in any contract with, or in any franchise, or special privilege granted by the Government or any subdivision, agency or instrumentality thereof, including government-owned or controlled corporations or their subsidiaries." These sweeping, all-embracing prohibitions imposed on the President and his official family, which prohibitions are not similarly imposed on other public officials or employees such as the Members of Congress, members of the civil service in general and members of the armed forces, are proof of the intent of the 1987 Constitution to

Page 19: ARTICLE VII the executive department

treat the President and his official family as a class by itself and to impose upon said class stricter prohibitions.

Thus, while all other appointive officials in the civil service are allowed to hold other office or employment in the government during their tenure when such is allowed by law or by the primary functions of their positions, members of the Cabinet, their deputies and assistants may do so only when expressly authorized by the Constitution itself. In other words, Section 7, Article IX-B is meant to lay down the general rule applicable to all elective and appointive public officials and employees, while Section 13, Article VII is meant to be the exception applicable only to the President, the Vice-President, Members of the Cabinet, their deputies and assistants.

This being the case, the qualifying phrase "unless otherwise provided in this Constitution" in Section 13, Article VII cannot possibly refer to the broad exceptions provided under Section 7, Article IX-B of the 1987 Constitution. To construe said qualifying phrase as respondents would have us do, would render nugatory and meaningless the manifest intent and purpose of the framers of the Constitution to impose a stricter prohibition on the President, Vice-President, Members of the Cabinet, their deputies and assistants with respect to holding other offices or employment in the government during their tenure. Respondents' interpretation that Section 13 of Article VII admits of the exceptions found in Section 7, par. (2) of Article IX-B would obliterate the distinction so carefully set by the framers of the Constitution as to when the high-ranking officials of the Executive Branch from the President to Assistant Secretary, on the one hand, and the generality of civil servants from the rank immediately below Assistant Secretary downwards, on the other, may hold any other office or position in the government during their tenure.

The prohibition against holding dual or multiple offices or employment under Section 13, Article VII of the Constitution must not, however, be construed as applying to posts occupied by the Executive officials specified therein without additional compensation in an ex-officio capacity as provided by law and as required 22 by the primary functions of said officials' office. The reason is that these posts do no comprise "any other office" within the contemplation of the constitutional prohibition but are properly an imposition of additional duties and functions on said officials. 23 To characterize these posts otherwise would lead to absurd consequences, among which are: The President of the Philippines cannot chair the National Security Council reorganized under Executive Order No. 115 (December 24, 1986). Neither can the Vice-President, the Executive Secretary, and the Secretaries of National Defense, Justice, Labor and Employment and Local Government sit in this Council, which would then have no reason to exist for lack of a chairperson and members. The respective undersecretaries and assistant secretaries, would also be prohibited.

The Secretary of Labor and Employment cannot chair the Board of Trustees of the National Manpower and Youth Council (NMYC) or the Philippine Overseas Employment Administration (POEA), both of which are attached to his department for policy coordination and guidance. Neither can his Undersecretaries and Assistant Secretaries chair these agencies.

The Secretaries of Finance and Budget cannot sit in the Monetary Board. 24 Neither can their respective undersecretaries and assistant secretaries. The Central Bank Governor would then be assisted by lower ranking employees in providing policy direction in the areas of money, banking and credit. 25 Indeed, the framers of our Constitution could not have intended such absurd consequences. A Constitution, viewed as a continuously operative charter of government, is not to be interpreted as

Page 20: ARTICLE VII the executive department

demanding the impossible or the impracticable; and unreasonable or absurd consequences, if possible, should be avoided.

To reiterate, the prohibition under Section 13, Article VII is not to be interpreted as covering positions held without additional compensation in ex-officio capacities as provided by law and as required by the primary functions of the concerned official's office. The term ex-officio means "from office; by virtue of office." It refers to an "authority derived from official character merely, not expressly conferred upon the individual character, but rather annexed to the official position." Ex-officio likewise denotes an "act done in an official character, or as a consequence of office, and without any other appointment or authority than that conferred by the office." 27 An ex-officio member of a board is one who is a member by virtue of his title to a certain office, and without further warrant or appointment.

Mandating additional duties and functions to the President, Vice-President, Cabinet Members, their deputies or assistants which are not inconsistent with those already prescribed by their offices or appointments by virtue of their special knowledge, expertise and skill in their respective executive offices is a practice long-recognized in many jurisdictions. It is a practice justified by the demands of efficiency, policy direction, continuity and coordination among the different offices in the Executive Branch in the discharge of its multifarious tasks of executing and implementing laws affecting national interest and general welfare and delivering basic services to the people. It is consistent with the power vested on the President and his alter egos, the Cabinet members, to have control of all the executive departments, bureaus and offices and to ensure that the laws are faithfully executed. 35 Without these additional duties and functions being assigned to the President and his official family to sit in the governing bodies or boards of governmental agencies or instrumentalities in an ex-officio capacity as provided by law and as required by their primary functions, they would be deprived of the means for control and supervision, thereby resulting in an unwieldy and confused bureaucracy.

It bears repeating though that in order that such additional duties or functions may not transgress the prohibition embodied in Section 13, Article VII of the 1987 Constitution, such additional duties or functions must be required by the primary functions of the official concerned, who is to perform the same in an ex-officio capacity as provided by law, without receiving any additional compensation therefor.

The ex-officio position being actually and in legal contemplation part of the principal office, it follows that the official concerned has no right to receive additional compensation for his services in the said position. The reason is that these services are already paid for and covered by the compensation attached to his principal office. It should be obvious that if, say, the Secretary of Finance attends a meeting of the Monetary Board as an ex-officio member thereof, he is actually and in legal contemplation performing the primary function of his principal office in defining policy in monetary and banking matters, which come under the jurisdiction of his department. For such attendance, therefore, he is not entitled to collect any extra compensation, whether it be in the form of a per diem or an honorarium or an allowance, or some other such euphemism. By whatever name it is designated, such additional compensation is prohibited by the Constitution.

In the light of the construction given to Section 13, Article VII in relation to Section 7, par. (2), Article IX-B of the 1987 Constitution, Executive Order No. 284 dated July 23, 1987 is unconstitutional. Ostensibly restricting the number of positions that Cabinet members,

Page 21: ARTICLE VII the executive department

undersecretaries or assistant secretaries may hold in addition to their primary position to not more than two (2) positions in the government and government corporations, Executive Order No. 284 actually allows them to hold multiple offices or employment in direct contravention of the express mandate of Section 13, Article VII of the 1987 Constitution prohibiting them from doing so, unless otherwise provided in the 1987 Constitution itself.

Compare with provisions against other officials:Art VI, Sec. 13Art. IX, A, Sec. 2Art. IX, B, Sec. 7Art. VIII, Sec. 12

Exceptions to the Rule:Vice-President – Art VII, Sec. 3, par 2Secretary of Justice – Art VIII, Sec 8 (1)Ex officio positions

B. SUCCESSION(A)At the beginnin gof the term

Art VII, Sec. 7, 10(B) During the term

Art VII, Sec 8, 10(C) Temporary Disability

Art. VII, Sec 2-3

[G.R. No. 146738. April 3, 2001.]JOSEPH E. ESTRADA vs. GLORIA MACAPAGAL-ARROYO

Facts:Petitioner postulates that respondent Arroyo as Vice President has no power to adjudge the

inability of the petitioner to discharge the powers and duties of the presidency. His significant submittal is that "Congress has the ultimate authority under the Constitution to determine whether the President is incapable of performing his functions in the manner provided for in section 11 of article VII."

Issue:Whether Estrada was just temporarily incapable of discharging his functions as President;

hence did not resign.

Held:No. An examination of section 11, Article VII is in order. It provides:

Page 22: ARTICLE VII the executive department

"SEC. 11. Whenever the President transmits to the President of the Senate and the Speaker of the House of Representatives his written declaration that he is unable to discharge the powers and duties of his office, and until he transmits to them a written declaration to the contrary, such powers and duties shall be discharged by the Vice-President as Acting President.

Whenever a majority of all the Members of the Cabinet transmit to the President of the Senate and to the Speaker of the House of Representatives their written declaration that the President is unable to discharge the powers and duties of his office, the Vice-President shall immediately assume the powers and duties of the office as Acting President.

Thereafter, when the President transmits to the President of the Senate and to the Speaker of the House of Representatives his written declaration that no inability exists, he shall reassume the powers and duties of his office. Meanwhile, should a majority of all the Members of the Cabinet transmit within five days to the President of the Senate and to the Speaker of the House of Representatives their written declaration that the President is unable to discharge the powers and duties of his office, the Congress shall decide the issue. For that purpose, the Congress shall convene, if it is not in session, within forty-eight hours, in accordance with its rules and without need of call.

If the Congress, within ten days after receipt of the last written declaration, or, if not in session, within twelve days after it is required to assemble, determines by a two-thirds vote of both Houses, voting separately, that the President is unable to discharge the powers and duties of his office, the Vice-President shall act as President; otherwise, the President shall continue exercising the powers and duties of his office."

That is the law. Now, the operative facts:1. Petitioner, on January 20, 2001, sent the above letter claiming inability to the Senate

President and Speaker of the House; 2. Unaware of the letter, respondent Arroyo took her oath of office as President on January 20,

2001 at about 12:30 p.m.; 3. Despite receipt of the letter, the House of Representatives passed on January 24, 2001 House

Resolution No. 175; On the same date, the House of the Representatives passed House Resolution No. 176, expressing its support to the assumption of the office of the president by Arroyo.

On February 7, 2001, the House of the Representatives passed House Resolution No. 178

which states: "RESOLUTION CONFIRMING PRESIDENT GLORIA MACAPAGAL-ARROYO'S NOMINATION OF SENATOR TEOFISTO T. GUINGONA, JR. AS VICE PRESIDENT OF THE REPUBLIC OF THE PHILIPPINES

4. Also, despite receipt of petitioner's letter claiming inability, some twelve (12) members of the Senate signed

On February 7, the Senate also passed Senate Resolution No. 82 which states: "RESOLUTION CONFIRMING PRESIDENT GLORIA MACAPAGAL ARROYO'S NOMINATION OF SEM. TEOFISTO T. GUINGONA, JR. AS VICE PRESIDENT OF THE REPUBLIC OF THE PHILIPPINES

On the same date, February 7, the Senate likewise passed Senate Resolution No. 83 which states: "RESOLUTION RECOGNIZING THAT THE IMPEACHMENT COURT IS FUNCTUS OFFICIO

Page 23: ARTICLE VII the executive department

5. On February 8, the Senate also passed Resolution No. 84 "certifying to the existence of vacancy in the Senate and calling on the COMELEC to fill up such vacancy through election to be held simultaneously with the regular election on May 14, 2001 and the Senatorial candidate garnering the thirteenth (13th) highest number of votes shall serve only for the unexpired term of Senator Teofisto T. Guingona, Jr.

6. Both houses of Congress started sending bills to be signed into law by respondent Arroyo as President.

7. Despite the lapse of time and still without any functioning Cabinet, without any recognition from any sector of government, and without any support from the Armed Forces of the Philippines and the Philippine National Police, the petitioner continues to claim that his inability to govern is only momentary.

What leaps to the eye from these irrefutable facts is that both houses of Congress have recognized respondent Arroyo as the President. Implicitly clear in that recognition is the premise that the inability of petitioner Estrada. Is no longer temporary. Congress has clearly rejected petitioner's claim of inability.

The question is whether this Court has jurisdiction to review the claim of temporary inability of petitioner Estrada and thereafter revise the decision of both Houses of Congress recognizing respondent Arroyo as president of the Philippines. Following Tañada v. Cuenco, we hold that this Court cannot exercise its judicial power or this is an issue "in regard to which full discretionary authority has been delegated to the Legislative xxx branch of the government." Or to use the language in Baker vs. Carr,103 there is a "textually demonstrable or a lack of judicially discoverable and manageable standards for resolving it." Clearly, the Court cannot pass upon petitioner's claim of inability to discharge the power and duties of the presidency. The question is political in nature and addressed solely to Congress by constitutional fiat. It is a political issue, which cannot be decided by this Court without transgressing the principle of separation of powers.

In fine, even if the petitioner can prove that he did not resign, still, he cannot successfully claim that he is a President on leave on the ground that he is merely unable to govern temporarily. That claim has been laid to rest by Congress and the decision that respondent Arroyo is the de jure, president made by a co-equal branch of government cannot be reviewed by this Court.

VII. POWERS AND FUNCTIONS OF THE PRESIDENT

A. EXECUTIVE POWER (Art VII, Secs. 1 and 17)

EN BANC [G.R. No. 88211. September 15, 1989.] FERDINAND E. MARCOS vs. HONORABLE RAUL MANGLAPUS

Facts:

Page 24: ARTICLE VII the executive department

Mr. Marcos, in his deathbed, has signified his wish to return to the Philippines to die. But Mrs. Aquino, considering the dire consequences to the nation of his return at a time when the stability of government is threatened from various directions and the economy is just beginning to rise and move forward, has stood firmly on the decision to bar the return of Mr. Marcos and his family.

Issue:Whether or not, in the exercise of the powers granted by the Constitution, the President may

prohibit the Marcoses from returning to the Philippines.

Held:Yes. As the Supreme Court in Ocampo v. Cabangis [15 Phil. 626 (1910)] pointed out "a grant

of the legislative power means a grant of all legislative power; and a grant of the judicial power means a grant of all the judicial power which may be exercised under the government." [At 631-632.] If this can be said of the legislative power which is exercised by two chambers with a combined membership of more than two hundred members and of the judicial power which is vested in a hierarchy of courts, it can equally be said of the executive power which is vested in one official — the President.

Consideration of tradition and the development of presidential power under the different constitutions are essential for a complete understanding of the extent of and limitations to the President's powers under the 1987 Constitution. Although the 1987 Constitution imposes limitations on the exercise of specific powers of the President, it maintains intact what is traditionally considered as within the scope of "executive power." Corollarily, the powers of the President cannot be said to be limited only to the specific powers enumerated in the Constitution. In other words, executive power is more than the sum of specific powers so enumerated.

The power involved is the President's residual power to protect the general welfare of the people. It is founded on the duty of the President, as steward of the people. To paraphrase Theodore Roosevelt, it is not only the power of the President but also his duty to do anything not forbidden by the Constitution or the laws that the needs of the nation demand. The President is not only clothed with extraordinary powers in times of emergency, but is also tasked with attending to the day-to-day problems of maintaining peace and order and ensuring domestic tranquillity in times when no foreign foe appears on the horizon. Wide discretion, within the bounds of law, in fulfilling presidential duties in times of peace is not in any way diminished by the relative want of an emergency specified in the commander-in-chief provision.

The request or demand of the Marcoses to be allowed to return to the Philippines cannot be considered in the light solely of the constitutional provisions guaranteeing liberty of abode and the right to travel, subject to certain exceptions, or of case law which clearly never contemplated situations even remotely similar to the present one. It must be treated as a matter that is appropriately addressed to those residual unstated powers of the President which are implicit in and correlative to the paramount duty residing in that office to safeguard and protect general welfare. In that context, such request or demand should submit to the exercise of a broader discretion on the part of the President to determine whether it must be granted or denied.

Page 25: ARTICLE VII the executive department

EN BANC [G.R. No. L-21897. October 22, 1963.]RAMON A. GONZALES vs. RUFINO G. HECHANOVA

Facts:On September 22, 1963, respondent Executive Secretary authorized the importation of

67,000 tons of foreign rice to be purchased from private sources, and created a rice procurement committee composed of the other respondents herein for the implementation of said proposed importation. Thereupon, or on September 25, 1963, herein petitioner, Ramon A. Gonzales — a rice planter, and president of the Iloilo Palay and Corn Planters Association, whose members are, likewise, engaged in the production of rice and corn — filed the petition herein, averring that, in making or attempting to make said importation of foreign rice, the aforementioned respondents "are acting without jurisdiction or in excess of jurisdiction", because Republic Act No. 3452 — which allegedly repeals or amends Republic Act No. 2207 — explicitly prohibits the importation of rice and corn by "the Rice and Corn Administration or any other government agency." On the other hand, Respondents question the sufficiency of petitioner's cause of action upon the theory that the proposed importation in question is not governed by Republic Act Nos. 2207 and 3452, but was authorized by the President as commander-in-chief "for military stock pile purposes" in the exercise of his alleged authority under Section 2 of Commonwealth Act No. 1; 8 that in cases of necessity, the President "or his subordinates may take such preventive measure for the restoration of good order and maintenance of peace"; and that, as Commander-in-Chief of our armed forces, "the President . . . is duty-bound to prepare for the challenge of threats of war or emergency without waiting for any special authority."

Issue:Whether the president can enter into executive agreements in violation of a legislative

enactment.

Held:No. Respondents' trend of thought, that, if an executive officer believes that compliance with

a certain statute will not benefit the people he is at liberty to disregard it, must be rejected — we still live under a rule of law.

Although the President may, under the American constitutional system, enter into executive agreements without previous legislative authority, he may not, by executive agreement, enter into a transaction which is prohibited by statutes enacted prior thereto.

Under the Constitution, the main function of the Executive is to enforce laws enacted by Congress. The former may not interfere in the performance of the legislative powers of the latter, except in the exercise of the veto power. He may not defeat legislative enactments that have acquired the status of laws, by indirectly repealing the same through an executive agreement providing for the performance of the very act prohibited by said laws.[G.R. No. 149724. August 19, 2003.]DENR vs. DENR REGION 12 EMPLOYEES

Facts:

Page 26: ARTICLE VII the executive department

Pursuant to DENR Adm. Order No. 99-14, a Memorandum was issued directing the immediate transfer of the DENR XII Regional Offices from Cotabato City to Koronadal, South Cotabato. The respondents questioned the authority of the DENR Secretary to reorganize the DENR because they claimed that the power to reorganize is “executive” in nature.

Issue:Whether the DENR Secretary has the authority to reorganize the DENR.

Held:It is apropos to reiterate the elementary doctrine of qualified political agency, thus:Under this doctrine, which recognizes the establishment of a single executive, all executive

and administrative organizations are adjuncts of the Executive Department, the heads of the various executive departments are assistants and agents of the Chief Executive, and, except in cases where the Chief Executive is required by the Constitution or law to act in person or the exigencies of the situation demand that he act personally, the multifarious executive and administrative functions of the Chief Executive are performed by and through the executive departments, and the acts of the Secretaries of such departments, performed and promulgated in the regular course of business, are, unless disapproved or reprobated by the Chief Executive, presumptively the acts of the Chief Executive. This doctrine is corollary to the control power of the President as provided for under Article VII, Section 17 of the 1987 Constitution.

Applying the doctrine of qualified political agency, the power of the President to reorganize the National Government may validly be delegated to his cabinet members exercising control over a particular executive department. Thus, in DOTC Secretary v. Mabalot, 21 we held that the President — through his duly constituted political agent and alter ego, the DOTC Secretary — may legally and validly decree the reorganization of the Department, particularly the establishment of DOTC-CAR as the LTFRB Regional Office at the Cordillera Administrative Region, with the concomitant transfer and performance of public functions and responsibilities appurtenant to a regional office of the LTFRB.

Similarly, in the case at bar, the DENR Secretary can validly reorganize the DENR by ordering the transfer of the DENR XII Regional Offices from Cotabato City to Koronadal, South Cotabato. The exercise of this authority by the DENR Secretary, as an alter ego, is presumed to be the acts of the President for the latter had not expressly repudiated the same.

B. POWER OF CONTROL (Sec. 17)

[G.R. No. L-7708. May 30, 1955.]JOSE MONDANO vs. FERNANDO SILVOSA

Facts:The petitioner is the duly elected and qualified mayor of the municipality of Mainit, province

of Surigao. On 27 February 1954 Consolacion Vda. de Mosende filed a sworn complaint with the Presidential Complaints and Action Committee accusing him of (1) rape committed on her daughter

Page 27: ARTICLE VII the executive department

Caridad Mosende; and (2) concubinage for cohabiting with her daughter in a place other than the conjugal dwelling. On 6 March the Assistant Executive Secretary indorsed the complaint to the respondent provincial governor for immediate investigation, appropriate action and report. On 10 April the petitioner appeared before the provincial governor in obedience to his summons and was served with a copy of the complaint filed by the provincial governor with the provincial board. On the same day, the provincial governor issued Administrative Order No. 8 suspending the petitioner from office. Thereafter, the Provincial Board proceeded to hear the charges preferred against the petitioner over his objection.

Issue: Whether the Assistant Executive Secretary, as agent of the Chief Executive, can exercise

control over local governments, most specifically a town mayor in this case.

Held:No. The department head as agent of the President has direct control and supervision over

all bureaus and offices under his jurisdiction as provided for in section 79(c) of the Revised Administrative Code, but he does not have the same control of local governments as that exercised by him over bureaus and offices under his jurisdiction. Likewise, his authority to order the investigation of any act or conduct of any person in the person in the service of any bureau of office under his department is confined to bureaus under his jurisdiction and does not extend to local governments over the President exercises only general supervision as may be provided by law (section 10, paragraph 1, Article VII of the Constitution). If the provisions of section 79(c) of the Revised Administrative Code are to be construed as conferring upon the corresponding department head direct control, direction, and supervision over all local governments and that for that reason he may order the investigation of an official of a local government for malfeasance in office, such interpretation would be contrary to the provisions of paragraph 1, section 10, article VII, of the Constitution.

In administrative law supervision means overseeing or the power or authority of an officer to see that subordinate officers perform their duties. If the latter fail or neglect to fulfill them the former may take such action or step as prescribed by law to make them perform these duties. Control, on the other hand, means the power of an officer to alter or modify or nullify or set aside what a subordinate officer had done in the performance of his duties and to substitute the judgment of the former for that of the latter.

The Congress has expressly and specifically lodged the provincial supervision over municipal officials in the provincial governor who is authorized to "receive and investigate complaints made under oath against municipal officers for neglect of duty, oppression, corruption or other form of maladministration of office, and conviction by final judgment of any crime involving moral turpitude." 2 And if the charges are serious, "he shall submit written charges touching the matter to the provincial board, furnishing a copy of such charges to the accused either personally or by registered mail, and he may in such case suspend the officer (not being the municipal treasurer) pending action by the board, if in his opinion the charge be one affecting the official integrity of the officer in question." 3 Section 86 of the Revised Administrative Code adds nothing to the power of supervision to be exercised by the Department Head over the administration of . . . municipalities . .

Page 28: ARTICLE VII the executive department

. If it be construed that it does and such additional power is the same authority as that vested in the Department Head by section 79 (c) of the Revised Administrative Code, then such additional power must be deemed to have been abrogated by section 10(1), Article VII, of the Constitution.

EN BANC [G.R. No. 46570. April 21, 1939.]JOSE D. VILLENA vs. THE SECRETARY OF THE INTERIOR

Facts:It appears that the Division of Investigation of the Department of Justice, upon the request of

the Secretary of the Interior, conducted an inquiry into the conduct of the petitioner, mayor of Makati, Rizal, as a result of which the latter was found to have committed bribery, extortion, malicious abuse of authority and unauthorized practice of the law profession. The respondent, therefore, on February 8, 1939, recommended to the President of the Philippines the suspension of the petitioner to prevent possible coercion of witnesses, which recommendation was granted, according to the answer of the Solicitor-General of March 20, 1939, verbally by the President on the same day. The Secretary of the Interior suspended the petitioner from office on February 9, 1939, and then and thereafter wired the Provincial Governor of Rizal with instruction that the petitioner be advised accordingly.

Villena contended that the Secretary of the Interior has no jurisdiction or authority to suspend and much less to prefer by himself administrative charges against the petitioner and decide also by himself the merits of the charges as the power to suspend municipal elective officials and to try and punish them for misconduct in office or dereliction of duty is lodged in some other agencies of the government. As to him, the Secretary of the Interior, by suspending him, has exercised control over local governments when that power has been taken away from the President of the Philippines by the Constitution

Issue:Whether the Secretary of Interior has authority to investigate and to suspend Villena,

pending the investigation of the charges against the latter.

Held:Yes. Section 79 (C) of the Administrative Code provides as follows:

"The Department Head shall have direct control, direction, and supervision over all bureaus and offices under his jurisdiction and may, any provision of existing law to the contrary notwithstanding, repeal or modify the decisions of the chiefs of said bureaus or offices when advisable in the public interest.

"The Department Head may order the investigation of any act or conduct of any person in the service of any bureau or office under his department and in connection therewith may appoint a committee or designate an official or person who shall conduct such investigations, and such committee, official, or person may summon, witness by subpoena and subpoena duces tecum, administer cath and take testimony relevant to the investigation."

Page 29: ARTICLE VII the executive department

The above section speaks, it is true, of direct control, direction, and supervision over bureaus and offices under the jurisdiction of the Secretary of the Interior, but this section should be interpreted in relation to section 86 of the same Code which grants to the Department of the Interior "executive supervision over the administration of provinces, municipalities, chartered cities and other local political subdivisions." In the case of Planas vs. Gil (37 Off. Gaz., 1228), we observed that "Supervision is not a meaningless thing. It is an active power. It is certainly not without limitation, but it at least implies authority to inquire into facts and conditions in order to render the power real and effective. If supervision is to be conscientious and rational, and not automatic and brutal, it must be founded upon a knowledge of actual facts and conditions disclosed after careful study and investigation." The principle there enunciated is applicable with equal force to the present case.We hold, therefore, that the Secretary of the Interior is invested with authority to order the investigation of the charges against the petitioner and to appoint a special investigator for that purpose. As regards the challenged power of the Secretary of the Interior to decree the suspension of the herein petitioner pending an administrative investigation of the charges against him, the question, it may be admitted, is not free from difficulties. There is no clear and express grant of power to the secretary to suspend a mayor of a municipality who is under investigation. On the contrary, the power appears lodged in the provincial governor by section 2188 of the Administrative Code which provides that "The provincial governor shall receive and investigate complaints made under oath against municipal officers for neglect of duty, oppression, corruption or other form of maladministration of office, and conviction by final judgment of any crime involving moral turpitude. For minor delinquency he may reprimand the offender; and if a more severe punishment seems to be desirable he shall submit written charges touching the matter to the provincial board, furnishing a copy of such charges to the accused either personally or by registered mail, and he may in such case suspend the officer (not being the municipal treasurer) pending action by the board, if in his opinion the charge be one affecting the official integrity of the officer in question. Where suspension is thus effected, the written charges against the officer shall be filed with the board within five days." The fact, however, that the power of suspension is expressly granted by section 2188 of the Administrative Code to the provincial governor does not mean that the grant is necessarily exclusive and precludes the Secretary of the Interior from exercising a similar power. For instance, counsel for the petitioner admitted in the oral argument that the President of the Philippines may himself suspend the petitioner from office in virtue of his greater power of removal (sec. 2191, as amended, Administrative Code) to be exercised conformably to law. Indeed, if the President could, in the manner prescribed by law, remove a municipal official, it would be a legal incongruity if he were to be devoid of the lesser power of suspension. And the incongruity would be more patent if, possessed of the power both to suspend and to remove a provincial official (sec. 2078, Administrative Code), the President were to be without the power to suspend a municipal official. Here is, parenthetically, an instance where, as counsel for petitioner admitted, the power to suspend a municipal official is not exclusive. Upon the other hand, it may be argued with some degree of plausibility that, if the Secretary of the Interior is, as we have hereinabove concluded, empowered to investigate the charges against the petitioner and to appoint a special investigator for that purpose, preventive suspension may be a means by

Page 30: ARTICLE VII the executive department

which to carry into effect a fair and impartial investigation. This is a point, however, which, for the reason hereinafter indicated, we do not have to decide.

In the deliberation of this case it has also been suggested that, admitting that the President of the Philippines is invested with the authority to suspend the petitioner, and it appearing that he had verbally approved or at least acquiesced in the action taken by the Secretary of the Interior, the suspension of the petitioner should be sustained on the principle of approval or ratification of the act of the Secretary of the Interior by the President of the Philippines. There is, to be sure, more weight in this argument than in the suggested generalization of section 37 of Act No. 4007. Withal, at first blush, the argument of ratification may seem plausible under the circumstances, it should be observed that there are certain prerogative acts which, by their very nature, cannot be validated by subsequent approval or ratification by the President. There are certain constitutional powers and prerogatives of the Chief Executive of the Nation which must be exercised by him in person and no amount of approval or ratification will validate the exercise of any of those powers by any other person. Such, for instance, i9 his power to suspend the writ of habeas corpus and proclaim martial law (par. 3, sec. 11, Art. VII) and the exercise by him of the benign prerogative of mercy (par. 6, sec. 11, idem). Upon the other hand, doubt is entertained be some members of the court whether the statement made by the Secretary to the President in the latter's behalf and by his authority that the President had no objection to the suspension of the petitioner could be accepted as an affirmative exercise of the power of suspension in this case, or that the verbal approval by the President of the suspension alleged in a pleading presented in this case by the Solicitor-General could be considered as a sufficient ratification in law.

After serious reflection, we have decided to sustain the contention of the government in this case on the broad proposition, albeit not suggested, that under the presidential type of government which we have adopted and considering the departmental organization established and continued in force by paragraph 1, section 12, Article VII, of our Constitution, all executive and administrative organizations are adjuncts of the Executive Department, the heads of the various executive departments are assistants and agents of the Chief Executive, and, except in cases where the Chief Executive is required by the Constitution or the law to act in person or the exigencies of the situation demand that he act personally, the multifarious executive and administrative functions of the Chief Executive are performed by and through the executive departments, and the acts of the secretaries of such departments, performed and promulgated in the regular course of business, are, unless disapproved or reprobated by the Chief Executive, presumptively the acts of the Chief Executive.

With reference to the Executive Department of the government, there is one purpose which is crystal-clear and is readily visible without the projection of judicial searchlight, and that is, the establishment of a single, not plural, Executive. The first section of Article VII of the Constitution, dealing with the Executive Department, begins with the enunciation of the principle that "The executive power shall be vested in a President of the Philippines." This means that the President of the Philippines is the Executive of the Government of the Philippines, and no other. The heads of the executive departments occupy political positions and hold office in an advisory capacity, and, in the language of Thomas Jefferson, "should be of the President's bosom confidence" (7 Writings, Ford ed., 498), and, in the language of Attorney-General Cushing (7 Op., Attorney-General, 453), "are subject to the direction of the President." Without minimizing the importance of the heads of

Page 31: ARTICLE VII the executive department

the various departments, their personality is in reality but the projection of that of the President. Stated otherwise, and as forcibly characterized by Chief Justice Taft of the Supreme Court of the United States, "each head of a department is, and must be, the President's alter ego in the matters of that department where the President is required by law to exercise authority"

Secretaries of departments, of course, exercise certain powers under the law but the law cannot impair or in any way affect the constitutional power of control and direction of the President. As a matter of executive policy, they may be granted departmental autonomy as to certain matters but this is by mere concession of the executive, in the absence of valid legislation in the particular field. If the President, then, is the authority in the Executive Department, he assumes the corresponding responsibility. The head of a department is a man of his confidence; he controls and directs his acts; he appoints him and can remove him at pleasure; he is the executive, not any of his secretaries. It is therefore logical that he, the President, should be answerable for the acts of administration of the entire executive Department before his own conscience no less than before that undefined power of public opinion which, in the language of Danie, Webster, is the last repository of popular government. these are the necessary corollaries of the American presidential type of government, and if there is any defect, it is attributable to the system itself. We cannot modify the system unless we modify the Constitution, and we cannot modify the Constitution by any subtle process of judicial interpretation or construction.

EN BANC [G.R. No. L-58184. October 30, 1981.]FREE TELEPHONE WORKERS UNION vs. THE HONORABLE MINISTER OF LABOR

Facts:Petitioner filed with the Ministry of Labor a notice of strike for unfair labor practices allegedly

committed by private respondent company inviolation of their existing collective bargaining agreement, particularly the unilateral and arbitrary implementation of a Code of Conduct to the detriment and interest of its members. Several conciliation meetings called by the Ministry followed. Thereafter, the Ministry of Labor pursuant to law, certified the labor dispute to the NLRC for compulsory arbitration and the holding of any strike at private respondent establishment was enjoined. Hearing was subsequently conducted whereas private respondent agreed to the indefinite preventive suspension of the provisions of the Code of Conduct, the principal cause of the controversy. In a petition for certiorari before the Supreme Court, petitioner union submits that Batas Pambansa Blg. 130 insofar as it amends Article 264 of the Labor Code delegating to the Minister of Labor the power and discretion to assume jurisdiction and/or certify strikes for compulsory arbitration to the NLRC, and in effect make or unmake the law on free collective bargaining, is an undue delegation of legislative powers and is contrary to the assurance of the State to the workers' right to self organization and collective bargaining. Such power, according to petitioner union, is within the competence of the President who can best determine national interests when a strike is in progress.

Issue: Whether there is invalid delegation in this case.

Page 32: ARTICLE VII the executive department

Author’s Note: The crux of this case is the validity of delegation of powers. The president’s power of control over the members of the Cabinet is discussed only to support the claim that the Philippines still maintained a presidential, not parliamentary, form of government during the effectivity of the 1973 Constitution under Marcos.

Held:Yes. (The author omitted the discussion on delegation of power by reason of its irrelevance.)As the only one whose constituency is national it is the President who, by virtue of his

election by the entire electorate, has an indisputable claim to speak for the country as a whole. Moreover, it is he who explicitly granted the greater power of control of such ministries. He continues to be the Executive, the amplitude and scope of the functions entrusted to him in the formulation of policy and its execution leading to the apt observation by Laski that there is not one aspect of which that does not affect the lives of all. The Prime Minister can be of valuable assistance indeed to the President in the discharge of his awesome responsibility, but it is the latter who is vested with powers, aptly characterized by Justice Laurel in Planas v. Gil, 67 Phil. 62 (1939) as broad and extraordinary (being) expected to govern with a firm and steady hand without vexation or embarrassing interference and much less dictation from any source.

Under the presidential system, all executive and administrative organizations are adjuncts of the Executive Department, the heads of the various executive departments are assistants and agents of the Chief Executive, and. except in cases where the Chief Executive is required by the Constitution or the law to act in person or the exigencies of the situation demand that he act personally, the multifarious executive and administrative functions of the Chief Executive are performed by and through the executive departments, and the acts of the secretaries of such departments, performed and promulgated in the regular course of business, are, unless disapproved or reprobated by the Chief Executive, presumptively the acts of the Chief Executive.

C. POWER OF GENERAL SUPERVISION OVER LGUsArt. X, Sec 4, 16

EN BANC RODOLFO T. GANZON vs. THE HONORABLE COURT OF APPEALSC. POWER OF GENERAL SUPERVISION OVER LOCAL GOVERNMENTS

Art. X, Sec. 4. The President shall exercise general supervision over local governments. Ganzon vs CAThe grant of mere supervisory power over local governments and autonomous regions is in line with the policy of the State to promote the autonomy of local governments and autonomous regions. There can be no real local autonomy while the National Government controls the local governments.

Ganzon vs CA 200 SCRA 271

Page 33: ARTICLE VII the executive department

The petitioners question the power of the President, acting through the Secretary of Local Government, to suspend and/or remove local officials.

It is the considered opinion of the Court that notwithstanding the change in the constitution, the charter did not intend to divest the legislature of its right -- or the President of her prerogative as conferred by existing legislation to provide administrative sanctions against local officials. The omission of "as may be provided by law" (Sec. 4, Art. X) signifies nothing more than to underscore local governments' autonomy from Congress and to break Congress' "control" over local govt. affairs. The Constitution did not, however, intend for the sake of local autonomy, to deprive the legislature of all authority over municipal corporations, in particular, concerning discipline.

Petitioners are under the impression that the Constitution has left the President mere supervisory powers, which supposedly excludes the power of investigation, and denied her control, which allegedly embraces disciplinary authority. This is a mistaken impression because legally "supervision" is not incompatible with disciplinary authority. "Control" has been defined as the power of an officer to alter, modify or nullify or set aside what a subordinate officer had done in the performance of his duties and to substitute the judgment of the former for that of the latter. "Supervision" on the other hand means overseeing or the power or authority of an officer to see that subordinate officers perform their duties.

D. POWER OF APPOINTMENTSARMIENTO V MISONPetitioners brought this suit for prohibition in their capacity as taxpayers, members of the Bar and law professors, to enjoin respondent Commissioner of Customs from performing his functions on the ground that his appointment, w/o confirmation by the CA, is unconstitutional.

HELD: Art. VII, Sec. 16, as orginally proposed by the Committe on Executive Power of the 1986 Con Com read:

Sec. 16. The President shall nominate and, with the consent of a Commission on Appointment, shall appoint the heads of executive departments and bureaus, ambassadors, other public ministers and consuls, or officers of the armed forces from the rank of colonel or naval captain and all other officers of the Government whose appointments are not otherwise provided for by law, and those whom he may be authorized by law to appoint. The Congress may by law vest the appointment of inferior officers in the President alone, in the courts, or in the heads of departments.

Page 34: ARTICLE VII the executive department

However, on motion of Comm. Foz, 2 changes were approved in the text of the provision. The first was to delete the phrase "and bureaus," and the second was to place a period (.) after the word "captain" and substitute the phrase "and all" w/ the phrase "HE SHALL ALSO APPOINT ANY." The first amendment was intended to exempt the appointment of bureau directors from the requirement of confirmation on the ground that this position is low and to require confirmation would subject bureau directors to political influence. On the other hand, the 2nd amendment was intended to subject to confirmation only those mentioned in the frist sentence, namely:

The heads of the exec. depts, ambassadors, other public ministers and consuls, officers of the armed forces from the rank of colonel or naval captain, and other officers whose appointments are vested in him in the Consti, i.e.,

(1) Regular members of the Judicial and Bar Council [Art. VIII, Sec. 8(2)]

(2) Chairman and Commissioners of the Civil Service Commission [Art. IX-B, Sec. 1 (2)];

(3) Chairman and Commissioners of the COMELEC [Art. IX-C, Sec. 1 (2)];

(4) Chairman and Commissioners of the COA [Art. IX-D, Sec. 1 (2)];

(5) Members of the regional consultative commission (Art. X, Sec. 18.)

The rest of the appointments mentioned in sec. 16 are not subject to confirmation. These are: (1) all other officers of the Govt whose appointments are not otherwise provided for by law; (2) those whom the Pres. may be authorized by law to appoint; and (3) officers lower in rank whose appointments Congress may by law vest in the Pres. alone.

Quintos-Deles vs. Commission on Appointments 177 SCRA 259

F: Petitioner and 3 others were appointed Sectoral Representatives by the President pursuant to Article VII, Section 16, par. 2 and Article XVIII, Section 7 of the Constitution. However, petitioner and the 3 other sectoral representatives- appointees were not able to take their oaths and discharge their duties as members of Congress due to the opposition of some congressmen-members of the Commission on Appointments (CA), who insisted that sectoral representatives must first be confirmed by the CA before they can take their oaths and/or assume office as members of the House of Representatives.

Page 35: ARTICLE VII the executive department

ISSUE: Whether the Constitution requires confirmation by the CA in the appointment of sectoral representatives to the House of Representatives.

HELD: YES. Section 16, Article VII of the Constitution provides that: "The president shall nominate and, with the consent of the Commission on Appointments, appoint the heads of the executive departments, ambassadors, other public ministers and consuls, or officers of the armed forces from the rank of colonel or naval captain, and other officers whose appointments are vested in him in this Constitution x x x".

In SARMIENTO VS. MISON (1987), the SC construed Sec. 16, Art. VII of the Constitution to mean that only appointments to offices mentioned in the first sentence of the said Section 16, Art. VII require confirmation by the CA.

Since the seats reserved for sectoral representatives in par. 2, Sec. 5, Art. VI may be filled by appointment by the President by express provision of Sec. 7, Art. XVIII of the Constitution, it is indubitable that sectoral representatives to the House of Representatives are among the "other officers whose appointments are vested in the President in this Constitution", referred to in the first sentence of Sec. 16, Art. VII whose appointments are subject to confirmation by the CA.(SARMIENTO VS. MISON)

Implicit in the invocation of par. 2, Section 16, Art. VII as authority for the appointment of petitioner is, the recognition by the President as appointing authority that petitioner's appointment requires confirmation by the CA. Under Par. 2, Sec. 16, Art VII, appointments made by the President pursuant thereto "shall be effective only until disapproval by the CA or until the next adjournment of the Congress." If indeed appointments of sectoral representatives need no confirmation, the President need not make any reference to the constitutional provisions above-quoted in appointing the petitioner. As a matter of fact, the President had expressly submitted petitioner's appointment for confirmation by the CA. Considering that Congress had adjourned without resp. CA having acted on petitioner's appointment, said appointment/nomination had become moot and academic pursuant to Sec. 23 of the Rules of resp. CA and "unless resubmitted shall not again be considered by the Commission." Adapted.

Calderon v. Carale (208 SCRA 254)

Page 36: ARTICLE VII the executive department

F: In March 1989, RA 6715 (Herrera-Veloso Law), amending the Labor Code, was approved. It provides in Sec. 13 thereof as follows:

"xxx

The Chairman, the Division Presiding Commissioners and other Commissioners shall be appointed by the President, subject to confirmation by the CA. xxx"

Pursuant to said law, President Aquino appointed the Chairman (B. CARALE) and Commissioners of the NLRC. The appointments stated that the appointees may qualify and enter upon the performances of the duties of the office.

The present petition for prohibition questions the constitutionality and legality of the permanent appointments extended by the President to the respondents Chairman and Members of the NLRC, without submitting the same to the Commission on Appointments for confirmation pursuant to Art. 215 of the Labor Code as amended by RA 6715.

Petitioners insists on a mandatory compliance w/ RA 6715 which has in its favor the presumption of validity. RA 6715 is not, according to the petitioner, an encroachment on the appointing power of the executive contained in Sec. 16 of Art. VII of the Constitution.

The Solicitor General contends, on the other hand, that RA 6715 transgresses Sec. 16, Art. VII by expanding the confirmation powers of the Commission on Appointments without constitutional basis.

ISSUES: (1) W/N Congress may, by law, require confirmation by the Commission on Appointments of appointments extended by the President to government officers additional to those expressly mentioned in the first sentence of Sec. 16, Art. VII of the Constitution. (NO)

HELD: The controversy in the case is focused on Sec. 16, Art. VII of the 1987 Constitution w/c provides:

"Sec. 16. The President shall nominate and, with the consent of the Commission on Appointments, appoint the heads of the executive departments, ambassadors, other public ministers and consuls, or officers of the armed forces from the rank of colonel or naval captain, and other officers whose appointments are vested in him in this Constitution. He shall also appoint all other officers of the Government whose appointments are not otherwise provided for by law, and those whom he may be authorized by law to appoint. The Congress may, by law, vest the

Page 37: ARTICLE VII the executive department

appointment of other officers lower in rank in the President alone, in the courts, or in the heads of departments, agencies, commissions, or boards." xxx

There are four groups of officers whom the President shall appoint. These four groups are:

First, the heads of the executive departments, ambassadors, other public ministers and consuls, officers of the armed forces from the rank of colonel or naval captain, and other officers whose appointments are vested in him in this Constitution;

Second, all other officers of the Government whose appointments are not otherwise provided for law;

Third, those whom the President may be authorized by law to appoint;

Fourth, officers lower in rank whose appointments the Congress may by law vest in the President alone.

The second sentence of Sec. 16, Art. VII refers to all other officers of the government whose appointments are not otherwise provided for by law and those whom the President may be author-ized by law to appoint.

Indubitably, the NLRC Chairman and Commissioners fall within the second sentence of Sec. 16, Art. VII, more specifically under "those whom he (the President) may be authorized by law to ap-point." Undeniably, the Chairman and Members of the NLRC are not among the officers mentioned in the first sentence of Sec. 16 whose appointments requires confirmation by the CA.

To the extent that RA 6715 requires confirmation by the CA of the appointments of respondents Chairman and Members of the NLRC, it is unconstitutional because:

1. it amends by legislation, the first sentence of Sec. 16, Art. VII of the Constitution by adding thereto appointments requiring confirmation by the CA; and

Page 38: ARTICLE VII the executive department

2. it amends by legislation, the second sentence of Sec. 16, Art. VII, by imposing the confirmation of the CA on appointments w/c are otherwise entrusted only with the President.

Deciding on what laws to pass is a legislative prerogative. Determining their constitutionality is a judicial function.

Supreme Court decisions applying or interpreting the Constitution shall form part of the legal system of the Philippines. No doctrine or principle of law laid down by the Court in a decision rendered en banc or in division may be modified or reversed except by the Court sitting en banc. Adapted.

Notes: From the rulings in Sarmiento III v. Mison, 156 S 549), Bautista v. Salonga, 172 S 160, and Deles v. Constitutional Commission, 177 S 259, these doctrines are deducible:

1. Confirmation by the CA is required only for presidential appointees as mentioned in the first sentence of Sec. 16, Art. VII, including, those officers whose appointments are expressly vested by the Constitution itself in the president (like sectoral representatives to Congress and members of the constitutional commissions of Audit, Civil Service and Election).

2. Confirmation is not required when the President appoints other government officers whose appointments are not otherwise provided for by law or those officers whom he may be authorized by law to appoint (like the Chairman and Members of the Com. on Human Rights). Also, as observed in Sarmiento v. Mison, when Congress creates inferior offices but omits to provide for appointment thereto, or provides in an unconstitutional manner for such appointments, the officers are considered as among those whose appointments are not otherwise provided for by law.

(2) W/N legislation can expand a constitutional provision after the Supreme Court has interpreted it.

Page 39: ARTICLE VII the executive department

In Endencia and Jugo v. David, 93 Phil. 699, the Court held:

"We have already said that the Legislature under our form of government is assigned the task and the power to make and enact laws, but not to interpret them. This is more true with regard to the interpretation of the Constitution, which is not within the sphere of the Legislative Department. If the Legislature may declare what a law means, or what a specific portion of the Constitution means, especially after the courts have in actual case ascertained its meaning by interpretation and applied it in a decision, this would surely cause confusion and instability in judicial processes and court decisions. Under such a system, a final court determination of a case based on judicial interpretation of the law or of the Constitution may be undermined or even annulled by a subsequent and different interpretation of the law or of the Constitution by the Legislative Department. That would neither be wise nor desirable, besides being clearly violative of the fundamental principles of our constitutional system of government, particularly those governing the separation of powers."

The function of the Court in passing upon an act of Congress is to " lay the article of the Constitution which is invoked beside the statute which is challenged and to decide whether the latter squares with the former" and to announce its considered judgment upon the question."

It cannot be overlooked that Sec. 16, Art. VII of the 1987 Constitution was deliberately intended by the framers to be a departure from the system embodied in the 1935 Constitution where the CA exercised the power of confirmation over almost all presidential appointments, leading to many cases of abuse of such power of confirmation. Adapted.

E. PARDONING POWERF. MILITARY POWERS

Writ of habeas Data

THE RULE ON THE WRIT OF HABEAS DATA

SECTION 1. Habeas Data. - The writ of habeas data is a remedy available to any person whose right to privacy in life, liberty or security is violated or threatened by an unlawful act or omission of a public official or employee, or of a private individual or entity engaged in the gathering, collecting

Page 40: ARTICLE VII the executive department

or storing of data or information regarding the person, family, home and correspondence of the aggrieved party.

SEC. 2. Who May File. - Any aggrieved party may file a petition for the writ of habeas data. However, in cases of extralegal killings and enforced disappearances, the petition may be filed by:

(a) Any member of the immediate family of the aggrieved party, namely: the spouse, children and parents; or

(b) Any ascendant, descendant or collateral relative of the aggrieved party within the fourth civil degree of consanguinity or affinity, in default of those mentioned in the preceding paragraph; or

SEC. 3. Where to File. - The petition may be filed with the Regional Trial Court where the petitioner or respondent resides, or that which has jurisdiction over the place where the data or information is gathered, collected or stored, at the option of the petitioner.

The petition may also be filed with the Supreme Court or the Court of Appeals or the Sandiganbayan when the action concerns public data files of government offices.

SEC. 4. Where Returnable; Enforceable. - When the writ is issued by a Regional Trial Court or any judge thereof, it shall be returnable before such court or judge.

When issued by the Court of Appeals or the Sandiganbayan or any of its justices, it may be returnable before such court or any justice thereof, or to any Regional Trial Court of the place where the petitioner or respondent resides, or that which has jurisdiction over the place where the data or information is gathered, collected or stored.

When issued by the Supreme Court or any of its justices, it may be returnable before such Court or any justice thereof, or before the Court of Appeals or the Sandiganbayan or any of its justices, or to

Page 41: ARTICLE VII the executive department

any Regional Trial Court of the place where the petitioner or respondent resides, or that which has jurisdiction over the place where the data or information is gathered, collected or stored.

The writ of habeas data shall be enforceable anywhere in the Philippines.

Sec. 5. Docket Fees. - No docket and other lawful fees shall be required from an indigent petitioner. The petition of the indigent shall be docked and acted upon immediately, without prejudice to subsequent submission of proof of indigency not later than fifteen (15) days from the filing of the petition.

SEC. 6. Petition. - A verified written petition for a writ of habeas data should contain:

(a) The personal circumstances of the petitioner and the respondent;

(b) The manner the right to privacy is violated or threatened and how it affects the right to life, liberty or security of the aggrieved party;

(c) The actions and recourses taken by the petitioner to secure the data or information;

(d) The location of the files, registers or databases, the government office, and the person in charge, in possession or in control of the data or information, if known;

(e) The reliefs prayed for, which may include the updating, rectification, suppression or destruction of the database or information or files kept by the respondent.

In case of threats, the relief may include a prayer for an order enjoining the act complained of; and

Page 42: ARTICLE VII the executive department

(f) Such other relevant reliefs as are just and equitable.

SEC. 7. Issuance of the Writ. - Upon the filing of the petition, the court, justice or judge shall immediately order the issuance of the writ if on its face it ought to issue. The clerk of court shall issue the writ under the seal of the court and cause it to be served within three (3) days from the issuance; or, in case of urgent necessity, the justice or judge may issue the writ under his or her own hand, and may deputize any officer or person serve it.

The writ shall also set the date and time for summary hearing of the petition which shall not be later than ten (10) work days from the date of its issuance.

SEC. 8. Penalty for Refusing to Issue or Serve the Writ. - A clerk of court who refuses to issue the writ after its allowance, or a deputized person who refuses to serve the same, shall be punished by the court, justice or judge for contempt without prejudice to other disciplinary actions.

SEC. 9. How the Writ is Served. - The writ shall be served upon the respondent by a judicial officer or by a person deputized by the court, justice or judge who shall retain a copy on which to make a return of service. In case the writ cannot be served personally on the respondent, the rules on substituted service shall apply.

SEC. 10. Return; Contents. - The respondent shall file a verified written return together with supporting affidavits within five (5) working days from service of the writ, which period may be reasonably extended by the Court for justifiable reasons. The return shall, among other things, contain the following:

(a) The lawful defenses such as national security, state secrets, privileged communications, confidentiality of the source of information of media and others;

(b) In case of respondent in charge, in possession or in control of the data or information subject of the petition;

Page 43: ARTICLE VII the executive department

(i) a disclosure of the data or information about the petitioner, the nature of such data or information, and the purpose for its collection;

(ii) the steps or actions taken by the respondent to ensure the security and confidentiality of the data or information; and,

(iii) the currency and accuracy of the data or information held; and,

(c) Other allegations relevant to the resolution of the proceeding.

A general denial of the allegations in the petition shall not be allowed.

SEC. 11. Contempt. - The court, justice or judge may punish with imprisonment or fine a respondent who commits contempt by making a false return, or refusing to make a return; or any person who otherwise disobeys or resist a lawful process or order of the court.

SEC. 12. When Defenses May be Heard in Chambers. - A hearing in chambers may be conducted where the respondent invokes the defense that the release of the data or information in question shall compromise national security or state secrets, or when the data or information cannot be divulged to the public due to its nature or privileged character.

Sec. 13. Prohibited Pleadings and Motions. - The following pleadings and motions are prohibited:

(a) Motion to dismiss;

(b) Motion for extension of time to file return, opposition, affidavit, position paper and other pleadings;

Page 44: ARTICLE VII the executive department

(c) Dilatory motion for postponement;

(d) Motion for a bill of particulars;

(e) Counterclaim or cross-claim;

(f) Third-party complaint;

(g) Reply;

(h) Motion to declare respondent in default;

(i) Intervention;

(j) Memorandum;

(k) Motion for reconsideration of interlocutory orders or interim relief orders; and

(l) Petition for certiorari, mandamus or prohibition against any interlocutory order.

SEC. 14. Return; Filing. - In case the respondent fails to file a return, the court, justice or judge shall proceed to hear the petition ex parte, granting the petitioner such relief as the petition may warrant unless the court in its discretion requires the petitioner to submit evidence.

Page 45: ARTICLE VII the executive department

SEC. 15. Summary Hearing. - The hearing on the petition shall be summary. However, the court, justice or judge may call for a preliminary conference to simplify the issues and determine the possibility of obtaining stipulations and admissions from the parties.

SEC. 16. Judgment. - The court shall render judgment within ten (10) days from the time the petition is submitted for decision. If the allegations in the petition are proven by substantial evidence, the court shall enjoin the act complained of, or order the deletion, destruction, or rectification of the erroneous data or information and grant other relevant reliefs as may be just and equitable; otherwise, the privilege of the writ shall be denied.

Upon its finality, the judgment shall be enforced by the sheriff or any lawful officers as may be designated by the court, justice or judge within five (5) working days.

SEC. 17. Return of Service. - The officer who executed the final judgment shall, within three (3) days from its enforcement, make a verified return to the court. The return shall contain a full statement of the proceedings under the writ and a complete inventory of the database or information, or documents and articles inspected, updated, rectified, or deleted, with copies served on the petitioner and the respondent.

The officer shall state in the return how the judgment was enforced and complied with by the respondent, as well as all objections of the parties regarding the manner and regularity of the service of the writ.

SEC. 18. Hearing on Officer’s Return. - The court shall set the return for hearing with due notice to the parties and act accordingly.

SEC. 19. Appeal. - Any party may appeal from the final judgment or order to the Supreme Court under Rule 45. The appeal may raise questions of fact or law or both.

The period of appeal shall be five (5) working days from the date of notice of the judgment or final order.

Page 46: ARTICLE VII the executive department

The appeal shall be given the same priority as in habeas corpus and amparo cases.

SEC. 20. Institution of Separate Actions. - The filing of a petition for the writ of habeas data shall not preclude the filing of separate criminal, civil or administrative actions.

SEC. 21. Consolidation. - When a criminal action is filed subsequent to the filing of a petition for the writ, the latter shall be consolidated with the criminal action.

When a criminal action and a separate civil action are filed subsequent to a petition for a writ of habeas data, the petition shall be consolidated with the criminal action.

After consolidation, the procedure under this Rule shall continue to govern the disposition of the reliefs in the petition.

SEC. 22. Effect of Filing of a Criminal Action. - When a criminal action has been commenced, no separate petition for the writ shall be filed. The relief under the writ shall be available to an aggrieved party by motion in the criminal case.

The procedure under this Rule shall govern the disposition of the reliefs available under the writ of habeas data.

SEC. 23. Substantive Rights. - This Rule shall not diminish, increase or modify substantive rights.

SEC. 24. Suppletory Application of the Rules of Court. - The Rules of Court shall apply suppletorily insofar as it is not inconsistent with this Rule.

Page 47: ARTICLE VII the executive department

SEC. 25. Effectivity. - This Rule shall take effect on February 2, 2008, following its publication in three (3) newspapers of general circulation.

[PUBLISHED IN THE MANILA BULLETIN, THE PHILIPPINE STAR AND THE PHILIPPINE DAILY INQUIRER ON 25 JANUARY 2008]

Writ of AmparoSection 1. Petition. - The petition for a writ of amparo is a remedy available to any person whose right to life, liberty and security is violated or threatened with violation by an unlawful act or omission of a public official or employee, or of a private individual or entity.

The writ shall cover extralegal killings and enforced disappearances or threats thereof.

Sec. 2. Who May File. - The petition may be filed by the aggrieved party or by any qualified person or entity in the following order:

1. Any member of the immediate family, namely: the spouse, children and parents of the aggrieved party;

2. Any ascendant, descendant or collateral relative of the aggrieved party within the fourth civil degree of consanguinity or affinity, in default of those mentioned in the preceding paragraph; or

3. Any concerned citizen, organization, association or institution, if there is no known member of the immediate family or relative of the aggrieved party.

The filing of a petition by the aggrieved party suspends the right of all other authorized parties to file similar petitions. Likewise, the filing of the petition by an authorized party on behalf of the aggrieved party suspends the right of all others, observing the order established herein.

Sec. 3. Where to File. - The petition may be filed on any day and at any time with the Regional Trial Court of the place where the threat, act or omission was committed or any of its elements occurred, or with the Sandiganbayan, the Court of Appeals, the Supreme Court, or any justice of such courts. The writ shall be enforceable anywhere in the Philippines.

When issued by a Regional Trial Court or any judge thereof, the writ shall be returnable before such court or judge.

When issued by the Sandiganbayan or the Court of Appeals or any of their justices, it may be returnable before such court or any justice thereof, or to any Regional Trial Court of the place

Page 48: ARTICLE VII the executive department

where the threat, act or omission was committed or any of its elements occurred.

When issued by the Supreme Court or any of its justices, it may be returnable before such Court or any justice thereof, or before the Sandiganbayan or the Court of Appeals or any of their justices, or to any Regional Trial Court of the place where the threat, act or omission was committed or any of its elements occurred.

Sec. 4. No Docket Fees. - The petitioner shall be exempted from the payment of the docket and other lawful fees when filing the petition. The court, justice or judge shall docket the petition and act upon it immediately.

Sec. 5. Contents of Petition. - The petition shall be signed and verified and shall allege the following:

1. The personal circumstances of the petitioner;

2. The name and personal circumstances of the respondent responsible for the threat, act or omission, or, if the name is unknown or uncertain, the respondent may be described by an assumed appellation;

3. The right to life, liberty and security of the aggrieved party violated or threatened with violation by an unlawful act or omission of the respondent, and how such threat or violation is committed with the attendant circumstances detailed in supporting affidavits;

4. The investigation conducted, if any, specifying the names, personal circumstances, and addresses of the investigating authority or individuals, as well as the manner and conduct of the investigation, together with any report;

5. The actions and recourses taken by the petitioner to determine the fate or whereabouts of the aggrieved party and the identity of the person responsible for the threat, act or omission; and

6. The relief prayed for.

The petition may include a general prayer for other just and equitable reliefs.

Sec. 6. Issuance of the Writ. - Upon the filing of the petition, the court, justice or judge shall immediately order the issuance of the writ if on its face it ought to issue. The clerk of court shall issue the writ under the seal of the court; or in case of urgent necessity, the justice or the judge may issue the writ under his or her own hand, and may deputize any officer or person to serve it.

The writ shall also set the date and time for summary hearing of the petition which shall not be later than seven (7) days from the date of its issuance.

Sec. 7. Penalty for Refusing to Issue or Serve the Writ. - A clerk of court who refuses to issue the

Page 49: ARTICLE VII the executive department

writ after its allowance, or a deputized person who refuses to serve the same, shall be punished by the court, justice or judge for contempt without prejudice to other disciplinary actions.

Sec. 8. How the Writ is Served. - The writ shall be served upon the respondent by a judicial officer or by a person deputized by the court, justice or judge who shall retain a copy on which to make a return of service. In case the writ cannot be served personally on the respondent, the rules on substituted service shall apply.

Sec. 9. Return; Contents. - Within seventy-two (72) hours after service of the writ, the respondent shall file a verified written return together with supporting affidavits which shall, among other things, contain the following:

1. The lawful defenses to show that the respondent did not violate or threaten with violation the right to life, liberty and security of the aggrieved party, through any act or omission;

2. The steps or actions taken by the respondent to determine the fate or whereabouts of the aggrieved party and the person or persons responsible for the threat, act or omission;

3. All relevant information in the possession of the respondent pertaining to the threat, act or omission against the aggrieved party; and

4. If the respondent is a public official or employee, the return shall further state the actions that have been or will still be taken:

1. to verify the identity of the aggrieved party;

2. to recover and preserve evidence related to the death or disappearance of the person identified in the petition which may aid in the prosecution of the person or persons responsible;

3. to identify witnesses and obtain statements from them concerning the death or disappearance;

4. to determine the cause, manner, location and time of death or disappearance as well as any pattern or practice that may have brought about the death or disappearance;

5. to identify and apprehend the person or persons involved in the death or disappearance; and

6. to bring the suspected offenders before a competent court.

The return shall also state other matters relevant to the investigation, its resolution and the prosecution of the case.

Page 50: ARTICLE VII the executive department

A general denial of the allegations in the petition shall not be allowed.

Sec. 10. Defenses not Pleaded Deemed Waived. - All defenses shall be raised in the return, otherwise, they shall be deemed waived.

Sec. 11. Prohibited Pleadings and Motions. - The following pleadings and motions are prohibited:

1. Motion to dismiss;

2. Motion for extension of time to file return, opposition, affidavit, position paper and other pleadings;

3. Dilatory motion for postponement;

4. Motion for a bill of particulars;

5. Counterclaim or cross-claim;

6. Third-party complaint;

7. Reply;

8. Motion to declare respondent in default;

9. Intervention;

10. Memorandum;

11. Motion for reconsideration of interlocutory orders or interim relief orders; and

12. Petition for certiorari, mandamus or prohibition against any interlocutory order.

Sec. 12. Effect of Failure to File Return. - In case the respondent fails to file a return, the court, justice or judge shall proceed to hear the petition ex parte.

Sec. 13. Summary Hearing. - The hearing on the petition shall be summary. However, the court, justice or judge may call for a preliminary conference to simplify the issues and determine the possibility of obtaining stipulations and admissions from the parties.

The hearing shall be from day to day until completed and given the same priority as petitions for habeas corpus.

Sec. 14. Interim Reliefs. - Upon filing of the petition or at anytime before final judgment, the court,

Page 51: ARTICLE VII the executive department

justice or judge may grant any of the following reliefs:

(a) Temporary Protection Order. - The court, justice or judge, upon motion or motu proprio, may order that the petitioner or the aggrieved party and any member of the immediate family be protected in a government agency or by an accredited person or private institution capable of keeping and securing their safety. If the petitioner is an organization, association or institution referred to in Section 3(c) of this Rule, the protection may be extended to the officers involved.

The Supreme Court shall accredit the persons and private institutions that shall extend temporary protection to the petitioner or the aggrieved party and any member of the immediate family, in accordance with guidelines which it shall issue.

The accredited persons and private institutions shall comply with the rules and conditions that may be imposed by the court, justice or judge.

(b) Inspection Order. - The court, justice or judge, upon verified motion and after due hearing, may order any person in possession or control of a designated land or other property, to permit entry for the purpose of inspecting, measuring, surveying, or photographing the property or any relevant object or operation thereon.

The motion shall state in detail the place or places to be inspected. It shall be supported by affidavits or testimonies of witnesses having personal knowledge of the enforced disappearance or whereabouts of the aggrieved party.

If the motion is opposed on the ground of national security or of the privileged nature of the information, the court, justice or judge may conduct a hearing in chambers to determine the merit of the opposition.

The movant must show that the inspection order is necessary to establish the right of the aggrieved party alleged to be threatened or violated.

The inspection order shall specify the person or persons authorized to make the inspection and the date, time, place and manner of making the inspection and may prescribe other conditions to protect the constitutional rights of all parties. The order shall expire five (5) days after the date of its issuance, unless extended for justifiable reasons.

(c) Production Order. - The court, justice or judge, upon verified motion and after due hearing, may order any person in possession, custody or control of any designated documents, papers, books, accounts, letters, photographs, objects or tangible things, or objects in digitized or electronic form, which constitute or contain evidence relevant to the petition or the return, to produce and permit their inspection, copying or photographing by or on behalf of the movant.

Page 52: ARTICLE VII the executive department

The motion may be opposed on the ground of national security or of the privileged nature of the information, in which case the court, justice or judge may conduct a hearing in chambers to determine the merit of the opposition.

The court, justice or judge shall prescribe other conditions to protect the constitutional rights of all the parties.

(d) Witness Protection Order. - The court, justice or judge, upon motion or motu proprio, may refer the witnesses to the Department of Justice for admission to the Witness Protection, Security and Benefit Program, pursuant to Republic Act No. 6981.

The court, justice or judge may also refer the witnesses to other government agencies, or to accredited persons or private institutions capable of keeping and securing their safety.

Sec. 15. Availability of Interim Reliefs to Respondent. - Upon verified motion of the respondent and after due hearing, the court, justice or judge may issue an inspection order or production order under paragraphs (b) and (c) of the preceding section.

A motion for inspection order under this section shall be supported by affidavits or testimonies of witnesses having personal knowledge of the defenses of the respondent.

Sec. 16. Contempt. - The court, justice or judge may order the respondent who refuses to make a return, or who makes a false return, or any person who otherwise disobeys or resists a lawful process or order of the court to be punished for contempt. The contemnor may be imprisoned or imposed a fine.

Sec. 17. Burden of Proof and Standard of Diligence Required. - The parties shall establish their claims by substantial evidence.

The respondent who is a private individual or entity must prove that ordinary diligence as required by applicable laws, rules and regulations was observed in the performance of duty.

The respondent who is a public official or employee must prove that extraordinary diligence as required by applicable laws, rules and regulations was observed in the performance of duty.

The respondent public official or employee cannot invoke the presumption that official duty has been regularly performed to evade responsibility or liability.

Sec. 18. Judgment. - The court shall render judgment within ten (10) days from the time the petition is submitted for decision. If the allegations in the petition are proven by substantial evidence, the court shall grant the privilege of the writ and such reliefs as may be proper and appropriate; otherwise, the privilege shall be denied.

Page 53: ARTICLE VII the executive department

Sec. 19. Appeal. - Any party may appeal from the final judgment or order to the Supreme Court under Rule 45. The appeal may raise questions of fact or law or both.

The period of appeal shall be five (5) working days from the date of notice of the adverse judgment.

The appeal shall be given the same priority as in habeas corpus cases.

Sec. 20. Archiving and Revival of Cases. - The court shall not dismiss the petition, but shall archive it, if upon its determination it cannot proceed for a valid cause such as the failure of petitioner or witnesses to appear due to threats on their lives.

A periodic review of the archived cases shall be made by the amparo court that shall, motu proprio or upon motion by any party, order their revival when ready for further proceedings. The petition shall be dismissed with prejudice upon failure to prosecute the case after the lapse of two (2) years from notice to the petitioner of the order archiving the case.

The clerks of court shall submit to the Office of the Court Administrator a consolidated list of archived cases under this Rule not later than the first week of January of every year.

Sec. 21. Institution of Separate Actions. - This Rule shall not preclude the filing of separate criminal, civil or administrative actions.

Sec. 22. Effect of Filing of a Criminal Action. - When a criminal action has been commenced, no separate petition for the writ shall be filed. The reliefs under the writ shall be available by motion in the criminal case.

The procedure under this Rule shall govern the disposition of the reliefs available under the writ of amparo.

Sec. 23. Consolidation. - When a criminal action is filed subsequent to the filing of a petition for the writ, the latter shall be consolidated with the criminal action.

When a criminal action and a separate civil action are filed subsequent to a petition for a writ of amparo, the latter shall be consolidated with the criminal action.

After consolidation, the procedure under this Rule shall continue to apply to the disposition of the reliefs in the petition.

Sec. 24. Substantive Rights. - This Rule shall not diminish, increase or modify substantive rights recognized and protected by the Constitution.

Sec. 25. Suppletory Application of the Rules of Court. - The Rules of Court shall apply suppletorily insofar as it is not inconsistent with this Rule.

Page 54: ARTICLE VII the executive department

Sec. 26. Applicability to Pending Cases. - This Rule shall govern cases involving extralegal killings and enforced disappearances or threats thereof pending in the trial and appellate courts.

Sec. 27. Effectivity. - This Rule shall take effect on October 24, 2007, following its publication in three (3) newspapers of general circulation.

G. EMERGENCY POWERS Art. VI, Sec. 23. (1) The Congress, by a vote of two-thirds of both Houses in joint sessions assembled, voting separately, shall have the sole power to declare the existence of a state of war.

(2) In times of war or other national emergency, the Congress may, by law, authorize the President, for a limited period and subject to such restrictions as it may prescribe, to exercise powers necessary and proper to carry out a declared national policy. Unless sooner withdrawn by resolution of the Congress, such powers shall cease upon the next adjournment thereof.

H. CONTRACTING AND GUARANTEEING FOREIGN LOANSArt. VII, Sec. 21. No treaty or international agreement shall be valid and effective unless concurred in by at least two-thirds of all the members of the Senate.

I. POWER OVER FOREIGN AFFAIRSJ. POWER OVER LEGISLATION

Facts:The petitioners take common issue on the power of the President (acting through the

Secretary of Local Government), to suspend and/or remove local officials. The petitions of Mayor Ganzon originated from a series of administrative complaints, ten in number, filed against him by various city officials sometime in 1988, on various charges, among them, abuse of authority, oppression, grave misconduct, disgraceful and immoral conduct, intimidation, culpable violation of the Constitution, and arbitrary detention.

Issue:Whether or not the Secretary of Local Government, as the President's alter ego, can suspend

and or remove local officials under the power of supervision of the President.

Held:Yes. The 1987 Constitution provides in Art. X, Sec. 4 that "[T]he President of the Philippines

shall exercise general supervision over local governments." It modifies a counterpart provision appearing in the 1935 Constitution, Art. VII, Sec. 10(1), stating that "[T]he President shall . . . exercise general supervision over all local governments as may be provided by law." It is the considered opinion of the Court that notwithstanding the change in the constitutional language, the Charter did not intend to divest the legislature of its right — or the President of her prerogative as conferred by existing legislation — to provide administrative sanctions against local officials. It is

Page 55: ARTICLE VII the executive department

our opinion that the omission (of "as may be provided by law") signifies nothing more than to underscore local governments' autonomy from Congress and to break Congress' "control" over local government affairs. The Constitution did not, however, intend, for the sake of local autonomy, to deprive the legislature of all authority over municipal corporations, in particular, concerning discipline. The change in constitutional language did not exempt local governments from legislative regulation provided regulation is consistent with the fundamental premise of autonomy.

Since local governments remain accountable to the national authority, the latter may, by law, and in the manner set forth therein, impose disciplinary action against local officials. In the case at bar, the Secretary of Local Government, the President's alter ego, in consonance with the specific legal provisions of Batas Blg. 337, the existing Local Government Code, can suspend petitioner Mayor of Iloilo City (G.R. Nos. 93252 and 95245) and petitioner member of the Sangguniang Panglunsod.

The petitioners are under the impression that the Constitution has left the President mere supervisory powers, which supposedly excludes the power of investigation, and denied her control, which allegedly embraces disciplinary authority. It is a mistaken impression because legally, "supervision" is not incompatible with disciplinary authority as this Court has held.

"Supervision" is not incompatible with disciplinary authority. As this Court held in Ganzon vs. Cayanan, 104 Phil. 484, "in administration law supervision means overseeing or the power or authority of an officer to see that subordinate officers perform their duties. If the latter fail or neglect to fulfill them the former may take such action or step as prescribed by law to make them perform their duties."

Thus, in those case that this Court denied the President the power (to suspend remove) it was not because we did not think that the President can not exercise it on account of his limited power, but because the law lodged the power elsewhere. But in those cases in which the law gave him the power, the Court, as in Ganzon v. Kayanan, found little difficulty in sustaining him.

Now, autonomy is either decentralization of administration or decentralization of power. There is decentralization of administration when the central government delegates administrative powers to political subdivisions in order to broaden the base of government power and in the process to make local governments "more responsive and accountable," and "ensure their fullest development as self-reliant communities and make them more effective partners in the pursuit of national development and social progress." At the same time, it relieves the central government of the burden of managing local affairs and enables it to concentrate on national concerns. The President exercises "general supervision" over them, but only to "ensure that local affairs are administered according to law." He has no control over their acts in the sense that he can substitute their judgments with his own.

[G.R. No. 125350. December 3, 2002.]HON. RTC JUDGES MERCEDES G. DADOLE vs. COMMISSION ON AUDIT

Facts:In 1986, petitioners as RTC and MTC judges stationed in Mandaue City received a monthly

allowance of P1,260 each pursuant to the yearly appropriation ordinance. Eventually, in 1991, it

Page 56: ARTICLE VII the executive department

was increased to P1,500 for each judge. However, on March 15, 1994, the Department of Budget and Management (DBM) issued Local Budget Circular No. 55 (LBC 55) which provides that the additional monthly allowances to be given by a local government unit should not exceed P1,000 in provinces and cities and P700 in municipalities. Acting on the said DBM directive, the Mandaue City Auditor issued notices of disallowance to herein petitioners in excess of the amount authorized by LBC 55. Thus, petitioners filed with the Office of the City Auditor a protest. However, it was treated as a motion for reconsideration and was endorsed to the Commission on Audit (COA) Regional Office No. 7. In turn, the COA Regional Office referred the said motion to their Head Office with recommendation that the same should be denied. Accordingly, it was denied by the COA. Hence, petitioners filed the instant petition. They argued, among others, that LBC 55 is void for infringing on the local autonomy of Mandaue City by dictating a uniform amount that a local government unit can disburse as additional allowances to judges stationed therein.

Petitioner judges argue that LBC 55 is void for infringing on the local autonomy of Mandaue City by dictating a uniform amount that a local government unit can disburse as additional allowances to judges stationed therein. They maintain that said circular is not supported by any law and therefore goes beyond the supervisory powers of the President.

Issue:Whether LBC 55 of the DBM is void for going beyond the supervisory powers of the President

Held:No. We recognize that, although our Constitution 6 guarantees autonomy to local

government units, the exercise of local autonomy remains subject to the power of control by Congress and the power of supervision by the President. Section 4 of Article X of the 1987 Philippine Constitution provides that:

Sec. 4. The President of the Philippines shall exercise general supervision over local governments. . . .

In Pimentel vs. Aguirre, 7 we defined the supervisory power of the President and distinguished it from the power of control exercised by Congress. Thus:

This provision (Section 4 of Article X of the 1987 Philippine Constitution) has been interpreted to exclude the power of control. In Mondano v. Silvosa, i[5] the Court contrasted the President's power of supervision over local government officials with that of his power of control over executive officials of the national government. It was emphasized that the two terms — supervision and control — differed in meaning and extent. The Court distinguished them as follows:

". . . In administrative law, supervision means overseeing or the power or authority of an officer to see that subordinate officers perform their duties. If the latter fail or neglect to fulfill them, the former may take such action or step as prescribed by law to make them perform their duties. Control, on the other hand, means the power of an officer to alter or modify or nullify or set aside what a subordinate officer ha[s] done in the performance of his duties and to substitute the judgment of the former for that of the latter." Ii

In Taule v. Santos, we further stated that the Chief Executive wielded no more authority than that of checking whether local governments or their officials were performing their duties as

Page 57: ARTICLE VII the executive department

provided by the fundamental law and by statutes. He cannot interfere with local governments, so long as they act within the scope of their authority. "Supervisory power, when contrasted with control, is the power of mere oversight over an inferior body; it does not include any restraining authority over such body," iv[8] we said.

In a more recent case, Drilon v. Lim, the difference between control and supervision was further delineated. Officers in control lay down the rules in the performance or accomplishment of act. If these rules are not followed, they may, in their discretion, order the act undone or redone by their subordinates or even decide to do it themselves. On the other hand, supervision does not cover such authority. Supervising officials merely see to it that the rules are followed, but they themselves do not lay down such rules, nor do they have the discretion to modify or replace them. If the rules are not observed, they may order the work done or redone, but only to conform to such rules. They may not prescribe their own manner of execution of the act. They have no discretion on this matter except to see to it that the rules are followed.

Under our present system of government, executive power is vested in the President. The members of the Cabinet and other executive officials are merely alter egos. As such, they are subject to the power of control of the President, at whose will and behest they can be removed from office; or their actions and decisions changed, suspended or reversed. In contrast, the heads of political subdivisions are elected by the people. Their sovereign powers emanate from the electorate, to whom they are directly accountable. By constitutional fiat, they are subject to the President's supervision only, not control, so long as their acts are exercised within the sphere of their legitimate powers. By the same token, the President may not withhold or alter any authority or power given them by the Constitution and the law.

Clearly then, the President can only interfere in the affairs and activities of a local government unit if he or she finds that the latter has acted contrary to law. This is the scope of the President's supervisory powers over local government units. Hence, the President or any of his or her alter egos cannot interfere in local affairs as long as the concerned local government unit acts within the parameters of the law and the Constitution. Any directive therefore by the President or any of his or her alter egos seeking to alter the wisdom of a law-conforming judgment on local affairs of a local government unit is a patent nullity because it violates the principle of local autonomy and separation of powers of the executive and legislative departments in governing municipal corporations.

Does LBC 55 go beyond the law it seeks to implement? Yes.LBC 55 provides that the additional monthly allowances to be given by a local government

unit should not exceed P1,000 in provinces and cities and P700 in municipalities. Section 458, par. (a)(1)(xi), of RA 7160, the law that supposedly serves as the legal basis of LBC 55, allows the grant of additional allowances "when the finances of the city government allow." The said provision does not authorize setting a definite maximum limit to the additional allowances granted to judges. Thus, we need not belabor the point that the finances of a city government may allow the grant of additional allowances higher than P1,000 if the revenues of the said city government exceed its annual expenditures. Thus, to illustrate, a city government with locally generated annual revenues of P40 million and expenditures of P35 million can afford to grant allowances of more than P1,000 each to, say, ten judges inasmuch as the finances of the city can afford it.

Page 58: ARTICLE VII the executive department

Setting a uniform amount for the grant of additional allowances is an inappropriate way of enforcing the criterion found in Section 458, par. (a)(1)(xi), of RA 7160. The DBM over-stepped its power of supervision over local government units by imposing a prohibition that did not correspond with the law it sought to implement. In other words, the prohibitory nature of the circular had no legal basis.

1. POWER OF APPOINTMENTd.1. Basis

d.2. with concurrence of COAArt VII, Sec. 16

EN BANC[G.R. No. 139554. July 21, 2006.]ARMITA B. RUFINO vs. BALTAZAR N. ENDRIGA

Facts:Presidential Decree No. 15 (PD 15) created the Cultural Center of the Philippines (CCP) for the

primary purpose of propagating arts and culture in the Philippines. PD 15 created a Board of Trustees ("Board") to govern the CCP. The controversy revolves on who between the contending groups, Rufino and Endriga groups, both claiming as the rightful trustees of the CCP Board, has the legal right to hold office. The resolution of the issue boils down to the constitutionality of the provision of PD 15 on the manner of filling vacancies in the Board. During the term of President Fidel V. Ramos, the CCP Board included Endriga, Lagdameo, Sison, Potenciano, Fernandez, Lenora A. Cabili ("Cabili"), and Manuel T. Mañosa ("Mañosa"). On 22 December 1998, then President Joseph E. Estrada appointed seven new trustees to the CCP Board for a term of four years to replace the Endriga group (by the Rufino Group) as well as two other incumbent trustees.

Endriga questioned President Estrada's appointment of seven new members to the CCP Board. The Endriga group alleged that under Section 6(b) of PD 15, vacancies in the CCP Board "shall be filled by election by a vote of a majority of the trustees held at the next regular meeting . . . ." In case "only one trustee survive[s], the vacancies shall be filled by the surviving trustee acting in consultation with the ranking officers of the [CCP]." The Endriga group claimed that it is only when the CCP Board is entirely vacant may the President of the Philippines fill such vacancies, acting in consultation with the ranking officers of the CCP. The Endriga group asserted that when former President Estrada appointed the Rufino group, only one seat was vacant due to the expiration of Mañosa's term. It maintained that under the CCP Charter, the trustees' fixed four-year term could only be terminated "by reason of resignation, incapacity, death, or other cause."

On the other hand, Rufino group asserted that the law could only delegate to the CCP Board the power to appoint officers lower in rank than the trustees of the Board. The law may not validly confer on the CCP trustees the authority to appoint or elect their fellow trustees, for the latter

Page 59: ARTICLE VII the executive department

would be officers of equal rank and not of lower rank. Section 6(b) of PD 15 authorizing the CCP trustees to elect their fellow trustees should be declared unconstitutional being repugnant to Section 16, Article VII of the 1987 Constitution allowing the appointment only of "officers lower in rank" than the appointing power.

Issue:Whether Section 6(b) of PD 15 is constitutional.

Held:No. The source of the President's power to appoint, as well as the Legislature's authority to

delegate the power to appoint, is found in Section 16, Article VII of the 1987 Constitution.The power to appoint is the prerogative of the President, except in those instances when the

Constitution provides otherwise. Usurpation of this fundamentally Executive power by the Legislative and Judicial branches violates the system of separation of powers that inheres in our democratic republican government.

Under Section 16, Article VII of the 1987 Constitution, the President appoints three groups of officers. The first group refers to the heads of the Executive departments, ambassadors, other public ministers and consuls, officers of the armed forces from the rank of colonel or naval captain, and other officers whose appointments are vested in the President by the Constitution. The second group refers to those whom the President may be authorized by law to appoint. The third group refers to all other officers of the Government whose appointments are not otherwise provided by law.

Under the same Section 16, there is a fourth group of lower-ranked officers whose appointments Congress may by law vest in the heads of departments, agencies, commissions, or boards. The present case involves the interpretation of Section 16, Article VII of the 1987 Constitution with respect to the appointment of this fourth group of officers.The President appoints the first group of officers with the consent of the Commission on Appointments. The President appoints the second and third groups of officers without the consent of the Commission on Appointments. The President appoints the third group of officers if the law is silent on who is the appointing power, or if the law authorizing the head of a department, agency, commission, or board to appoint is declared unconstitutional. Thus, if Section 6(b) and (c) of PD 15 is found unconstitutional, the President shall appoint the trustees of the CCP Board because the trustees fall under the third group of officers.

The framers of the 1987 Constitution clearly intended that Congress could by law vest the appointment of lower-ranked officers in the heads of departments, agencies, commissions, or boards. The deliberations 26 of the 1986 Constitutional Commission explain this intent beyond any doubt.

The framers of the 1987 Constitution changed the qualifying word "inferior" to the less disparaging phrase "lower in rank" purely for style. However, the clear intent remained that these inferior or lower in rank officers are the subordinates of the heads of departments, agencies, commissions, or boards who are vested by law with the power to appoint. The express language of the Constitution and the clear intent of its framers point to only one conclusion — the officers whom the heads of departments, agencies, commissions, or boards may appoint must be of lower rank than those vested by law with the power to appoint.

Page 60: ARTICLE VII the executive department

Further, Section 16, Article VII of the 1987 Constitution authorizes Congress to vest "in the heads of departments, agencies, commissions, or boards" the power to appoint lower-ranked officers. Section 16 provides:

The Congress may, by law, vest the appointment of other officers lower in rank in the President alone, in the courts, or in the heads of departments, agencies, commissions, or boards.

In a department in the Executive branch, the head is the Secretary. The law may not authorize the Undersecretary, acting as such Undersecretary, to appoint lower-ranked officers in the Executive department. In an agency, the power is vested in the head of the agency for it would be preposterous to vest it in the agency itself. In a commission, the head is the chairperson of the commission. In a board, the head is also the chairperson of the board. In the last three situations, the law may not also authorize officers other than the heads of the agency, commission, or board to appoint lower-ranked officers.

The grant of the power to appoint to the heads of agencies, commissions, or boards is a matter of legislative grace. Congress has the discretion to grant to, or withhold from, the heads of agencies, commissions, or boards the power to appoint lower-ranked officers. If it so grants, Congress may impose certain conditions for the exercise of such legislative delegation, like requiring the recommendation of subordinate officers or the concurrence of the other members of the commission or board.

This is in contrast to the President's power to appoint which is a self-executing power vested by the Constitution itself and thus not subject to legislative limitations or conditions. The power to appoint conferred directly by the Constitution on the Supreme Court en banc and on the Constitutional Commissions 30 is also self-executing and not subject to legislative limitations or conditions.

The 1987 Constitution speaks of vesting the power to appoint "in the courts, or in the heads of departments, agencies, commissions, or boards." This is consistent with Section 5(6), Article VIII of the 1987 Constitution which states that the "Supreme Court shall . . . [a]ppoint all officials and employees of the Judiciary in accordance with the Civil Service Law," making the Supreme Court en banc the appointing power. In sharp contrast, when the 1987 Constitution speaks of the power to appoint lower-ranked officers in the Executive branch, it vests the power "in the heads of departments, agencies, commissions, or boards."

The CCP, being governed by a board, is not an agency but a board for purposes of Section 16, Article VII of the 1987 Constitution. Section 6(b) and (c) of PD 15 is thus irreconcilably inconsistent with Section 16, Article VII of the 1987 Constitution. Section 6(b) and (c) of PD 15 empowers the remaining trustees of the CCP Board to fill vacancies in the CCP Board, allowing them to elect their fellow trustees. On the other hand, Section 16, Article VII of the 1987 Constitution allows heads of departments, agencies, commissions, or boards to appoint only "officers lower in rank" than such "heads of departments, agencies, commissions, or boards." This excludes a situation where the appointing officer appoints an officer equal in rank as him. Thus, insofar as it authorizes the trustees of the CCP Board to elect their co-trustees, Section 6(b) and (c) of PD 15 is unconstitutional because it violates Section 16, Article VII of the 1987 Constitution.

Page 61: ARTICLE VII the executive department

It does not matter that Section 6(b) of PD 15 empowers the remaining trustees to "elect" and not "appoint" their fellow trustees for the effect is the same, which is to fill vacancies in the CCP Board. A statute cannot circumvent the constitutional limitations on the power to appoint by filling vacancies in a public office through election by the co-workers in that office. Such manner of filling vacancies in a public office has no constitutional basis.

Further, Section 6(b) and (c) of PD 15 makes the CCP trustees the independent appointing power of their fellow trustees. The creation of an independent appointing power inherently conflicts with the President's power to appoint. This inherent conflict has spawned recurring controversies in the appointment of CCP trustees every time a new President assumes office.

EN BANC[G.R. No. 79974. December 17, 1987.]ULPIANO P. SARMIENTO III AND JUANITO G. ARCILLA vs. SALVADOR MISON

Facts:The petitioners questioned the appointment of Mison as Commissioner of the Bureau of

customs as it was done without the confirmation of the Commission on Appointments.

Issue:Whether the appointment is valid.

Held:Yes. It is readily apparent that under the provisions of the 1987 Constitution, just quoted,

there are four (4) groups of officers whom the President shall appoint. These four (4) groups, to which we will hereafter refer from time to time, are:

First, the heads of the executive departments, ambassadors, other public ministers and consuls, officers of the armed forces from the rank of colonel or naval captain, and other officers whose appointments are vested in him in this Constitution;

Second, all other officers of the Government whose appointments are not otherwise provided for by law;

Third, those whom the President may be authorized by law to appoint; Fourth, officers lower in rank 4 whose appointments the Congress may by law vest in the

President alone.

The first group of officers is clearly appointed with the consent of the Commission on Appointments. Appointments of such officers are initiated by nomination and, if the nomination is confirmed by the Commission on Appointments, the President appoints. The second, third and fourth groups of officers are the present bone of contention.in the 1935 Constitution, almost all presidential appointments required the consent (confirmation) of the Commission on Appointments. It is now a sad part of our political history that the power of

Page 62: ARTICLE VII the executive department

confirmation by the Commission on Appointments, under the 1935 Constitution, transformed that commission, many times, into a venue of "horse-trading" and similar malpractices.

On the other hand, the 1973 Constitution, consistent with the authoritarian pattern in which it was molded and remolded by successive amendments, placed the absolute power of appointment in the President with hardly any check on the part of the legislature.Given the above two in extremes, one, in the 1935 Constitution and the other, in the 1973 Constitution, it is not difficult for the Court to state that the framers of the 1987 Constitution and the people adopting it, struck a "middle ground" by requiring the consent (confirmation) of the Commission on Appointments for the first group of appointments and leaving to the President, without such confirmation, the appointment of other officers, i.e., those in the second and third groups as well as those in the fourth group, i.e., officers of lower rank.

As a result of the innovations introduced in Sec. 16, Article VII of the 1987 Constitution, there are officers whose appointments require no confirmation of the Commission on Appointments, even if such officers may be higher in rank, compared to some officers whose appointments have to be confirmed by the Commission on Appointments under the first sentence of the same Sec. 16, Art. VII.But these contrasts, while initially impressive, merely underscore the purposive intention and deliberate judgment of the framers of the 1987 Constitution that, except as to those officers whose appointments require the consent of the Commission on Appointments by express mandate of the first sentence in Sec., 16, Art. VII, appointments of other officers are left to the President without need of confirmation by the Commission on Appointments. This conclusion is inevitable, if we are to presume, as we must, that the framers of the 1987 Constitution were knowledgeable of what they were doing and of the foreseable effects thereof.

Besides, the power to appoint is fundamentally executive or presidential in character. Limitations on or qualifications of such power should be strictly construed against them. Such limitations or qualifications must be clearly stated in order to be recognized. But, it is only in the first sentence of Sec. 16, Art. VII where it is clearly stated that appointments by the President to the positions therein enumerated require the consent of the Commission on Appointments.

It is evident that the position of Commissioner of the Bureau of Customs (a bureau head) is not one of those within the first group of appointments where the consent of the Commission on Appointments is required. As a matter of fact, as already pointed out, while the 1935 Constitution includes "heads of bureaus" among those officers whose appointments need the consent of the Commission on Appointments, the 1987 Constitution, on the other hand, deliberately excluded the position of "heads of bureaus" from appointments that need the consent (confirmation) of the Commission on Appointments.

Moreover, the President is expressly authorized by law to appoint the Commissioner of the Bureau of Customs. The original text of Sec. 601 of Republic Act No. 1937, otherwise known as the Tariff and Customs Code of the Philippines, which was enacted by the Congress of the Philippines on 22 June 1967.

After the effectivity of the 1987 Constitution, however, Rep. Act No. 1937 and PD No. 34 have to be read in harmony with Sec. 16, Art. VII, with the result that, while the appointment of the Commissioner of the Bureau of Customs is one that devolves on the President, as an appointment

Page 63: ARTICLE VII the executive department

he is authorized by law to make, such appointment, however, no longer needs the confirmation of the Commission on Appointments.

[G.R. No. 86439. April 13, 1989.]MARY CONCEPCION BAUTISTA vs. SENATOR JOVITO R. SALONGA

Facts:On 27 August 1987, the President of the Philippines designated herein petitioner Mary

Concepcion Bautista as "Acting Chairman, Commission on Human Rights." Realizing perhaps the need for a permanent chairman and members of the Commission on Human Rights, befitting an independent office, as mandated by the Constitution, the President of the Philippines on 17 December 1988 extended to petitioner Bautista a permanent appointment as Chairman of the Commission. On 22 December 1988, before the Chief Justice of this Court, Hon. Marcelo B. Fernan, petitioner Bautista took her oath of office by virtue of her appointment as Chairman of the Commission on Human Rights.

On 9 January 1989, petitioner Bautista received a letter from the Secretary of the Commission on Appointments requesting her to submit to the Commission certain information and documents as required by its rules in connection with the confirmation of her appointment as Chairman of the Commission on Human Rights. On 26 January 1989, the Commission on Appointments disapproved petitioner Bautista's "ad interim appointment" as Chairperson of the Commission on Human Rights in view of her refusal to submit to the jurisdiction of the Commission on Appointments. the President had designated PCHR Commissioner Hesiquio R. Mallillin as "Acting Chairman of the Commission" pending the resolution of Bautista's case.

Issue:Whether or not the appointment by the President of the Chairman of the Commission on

Human Rights (CHR), an "independent office" created by the 1987 Constitution, is to be made with or without the confirmation of the Commission on Appointments.

Held:Since the position of Chairman of the Commission on Human Rights is not among the

positions mentioned in the first sentence of Sec. 16, Art. VII of the 1987 Constitution, appointments to which are to be made with the confirmation of the Commission on Appointments, it follows that the appointment by the President of the Chairman of the CHR is to be made without the review or participation of the Commission on Appointments.

To be more precise, the appointment of the Chairman and Members of the Commission on Human Rights is not specifically provided for in the Constitution itself, unlike the Chairmen and Members of the Civil Service Commission, the Commission on Elections and the Commission on Audit, whose appointments are expressly vested by the Constitution in the President with the consent of the Commission on Appointment.

The President appoints the Chairman and Members of the Commission on Human Rights pursuant to the second sentence in Section 16, Art. VII, that is, without the confirmation of the

Page 64: ARTICLE VII the executive department

Commission on Appointments because they are among the officers of government "whom he (the President) may be authorized by law to appoint." And Section 2(c), Executive Order No. 163, 5 May 1987, authorizes the President to appoint the Chairman and Members of the Commission on Human Rights.As disclosed by the records, and as previously adverted to, it is clear that petitioner Bautista was extended by Her Excellency, the President a permanent appointment as Chairman of the Commission on Human Rights on 17 December 1988. Before this date, she was merely the "Acting Chairman" of the Commission. Bautista's appointment on 17 December 1988 is an appointment that was for the President solely to make, i.e., not an appointment to be submitted for review and confirmation (or rejection) by the Commission on Appointments. This is in accordance with Sec. 16, Art. VII of the 1987 Constitution and the doctrine in Mison which is here reiterated.

The threshold question that has really come to the fore is whether the President, subsequent to her act of 17 December 1988, and after petitioner Bautista had qualified for the office to which she had been appointed, by taking the oath of office and actually assuming and discharging the functions and duties thereof, could extend another appointment to the petitioner on 14 January 1989, an "ad interim appointment" as termed by the respondent Commission on Appointments or any other kind of appointment to the same office of Chairman of the Commission on Human Rights that called for confirmation by the Commission on Appointments.

The Court, with all due respect to both the Executive and Legislative Departments of government, and after careful deliberation, is constrained to hold and rule in the negative. When Her Excellency, the President converted petitioner Bautista's designation as Acting Chairman to a permanent appointment as Chairman of the Commission on Human Rights on 17 December 1988, significantly she advised Bautista (in the same appointment letter) that, by virtue of such appointment, she could qualify and enter upon the performance of the duties of the office (of Chairman of the Commission on Human Rights). All that remained for Bautista to do was to reject or accept the appointment. Obviously, she accepted the appointment by taking her oath of office before the Chief Justice of the Supreme Court, Hon. Marcelo B. Fernan and assuming immediately thereafter the functions and duties of the Chairman of the Commission on Human Rights. Bautista's appointment therefore on 17 December 1988 as Chairman of the Commission on Human Rights was a completed act on the part of the President.

Respondent Commission vigorously contends that, granting that petitioner's appointment as Chairman of the Commission on Human Rights is one that, under Sec. 16, Art. VII of the Constitution, as interpreted in the Mison case, is solely for the President to make, yet, it is within the president's prerogative to voluntarily submit such appointment to the Commission on Appointment for confirmation. The mischief in this contention, as the Court perceives it, lies in the suggestion that the President (with Congress agreeing) may, from time to time move power boundaries, in the Constitution differently from where they are placed by the Constitution.

Nor can the Commission on Appointments, by the actual exercise of its constitutionally delimited power to review presidential appointments, create power to confirm appointments that the Constitution has reserved to the President alone. Stated differently, when the appointment is one that the Constitution mandates is for the President to make without the participation of the Commission on Appointments, the executive's voluntary act of submitting such appointment to the

Page 65: ARTICLE VII the executive department

Commission on Appointments and the latter's act of confirming or rejecting the same, are done without or in excess of jurisdiction.

Nor can respondents impressively contend that the new appointment or re-appointment on 14 January 1989 was an ad interim appointment, because, under the Constitutional design, ad interim appointments do not apply to appointments solely for the President to make, i.e., without the participation of the Commission on Appointments. Ad interim appointments, by their very nature under the 1987 Constitution, extend only to appointments where the review of the Commission on Appointments is needed. That is why ad interim appointments are to remain valid until disapproval by the Commission on Appointments or until the next adjournment of Congress; but appointments that are for the President solely to make, that is, without the participation of the Commission on Appointments, can not be ad interim appointments.

EN BANC[G.R. No. 83216. September 4, 1989.]TERESITA QUINTOS-DELES vs CIMMISSION ON CONSTITUTIONAL COMMISSIONS

Facts:On April 6, 1988, petitioner and three others were appointed Sectoral Representatives by the

President pursuant to Article VII, Section 16, paragraph 2 and Article XVIII, Section 7 of the Constitution. However, petitioner and the three other sectoral representatives-appointees were not able to take their oaths and discharge their duties as members of Congress due to the opposition of some congressmen-members of the Commission on Appointments, who insisted that sectoral representatives must first be confirmed by the respondent Commission before they could take their oaths and/or assume office as members of the House of Representatives.

Issue:Whether the Constitution requires the appointment of sectoral representatives to the House

of Representatives to be confirmed by the Commission on Appointments.

Held:In Sarmiento vs. Mison, et al. (156 SCRA 549 [1987]), we construed Section 16, Article VII of

the Constitution to mean that only appointments to offices mentioned in the first sentence of the said Section 16, Article VII require confirmation by the Commission on Appointments. The ruling in Mison was reiterated in the recent case of Mary Concepcion Bautista vs. Sen. Jovito Salonga, et al. (G.R. No. 86439, promulgated on April 13, 1989). Since the seats reserved for sectoral representatives in paragraph 2, Section 5, Art. VI may be filled by appointment by the President by express provision of Section 7, Art. XVIII of the Constitution, it is indubitable that sectoral representatives to the House of Representatives are among the "other officers whose appointments are vested in the President in this Constitution," referred to in the first sentence of Section 16, Art. VII whose appointments are subject to confirmation by the Commission on Appointments.

Page 66: ARTICLE VII the executive department

There are appointments vested in the President in the Constitution which, by express mandate of the Constitution, require no confirmation such as appointments of members of the Supreme Court and judges of lower courts (Sec. 9, Art. VIII) and the Ombudsman and his deputies (Sec. 9, Art. XI). No such exemption from confirmation had been extended to appointments of sectoral representatives in the Constitution.

Petitioner's appointment was furthermore made pursuant to Art. VII, Section 16, paragraph 2 which provides:

"SEC 16. . . .

The President shall have the power to make appointments during the recess of the Congress, whether voluntary or compulsory, but such appointments shall be effective only until disapproval by the Commission on Appointments or until the next adjournment of the Congress."

The reference to paragraph 2, Section 16 of Article VII as additional authority for the appointment of petitioner is of vital significance to the case at bar. The records show that petitioner's appointment was made on April 6, 1988 or while Congress was in recess (March 26, 1988 to April 17, 1988); hence, the reference to the said paragraph 2 of Section 16, Art. VII in the appointment extended to her.

Implicit in the invocation of paragraph 2, Section 16, Art. VII as authority for the appointment of petitioner is, the recognition by the President as appointing authority that petitioner's appointment requires confirmation by the Commission on Appointments. Under paragraph 2, Section 16, Art. VII, appointments made by the President pursuant thereto "shall be effective only until disapproval by the Commission on Appointments or until the next adjournment of the Congress." If indeed appointments of sectoral representatives need no confirmation, the President need not make any reference to the constitutional provisions above-quoted in appointing the petitioner. As a matter of fact, the President in a letter dated April 11, 1989 had expressly submitted petitioner's appointment for confirmation by the Commission on Appointments.

The provisions of Executive Order No. 198 do not deal with the manner of appointment of sectoral representatives. Executive Order No. 198 confines itself to specifying the sectors to be represented, their number, and the nomination of such sectoral representatives. The power of the President to appoint sectoral representatives remains directly derived from Section 7, Article XVIII of the Constitution which is quoted in the second "Whereas" clause of Executive Order No. 198. Petitioner Deles' appointment was issued not by virtue of Executive Order No. 198 but pursuant to Art. VII, Section 16, paragraph 2 and Art. XVIII, Section 7 of the Constitution which require submission to the confirmation process.

EN BANC[G.R. No. 91636. April 23, 1992.]PETER JOHN D. CALDERON vs. BARTOLOME CARALE

Facts:

Page 67: ARTICLE VII the executive department

Sometime in March 1989, RA 6715 (Herrera-Veloso Law), amending the Labor Code (PD 442) was approved. It provides in Section 13 thereof as follows:"xxx xxx xxx

The Chairman, the Division Presiding Commissioners and other Commissioners shall all be appointed by the President, subject to confirmation by the Commission on Appointments. Appointments to any vacancy shall come from the nominees of the sector which nominated the predecessor. The Executive Labor Arbiters and Labor Arbiters shall also be appointed by the President, upon recommendation of the Secretary of Labor and Employment, and shall be subject to the Civil Service Law, rules and regulations."

Pursuant to said law (RA 6715), President Aquino appointed the Chairman and Commissioners of the NLRC representing the public, workers and employers sectors.

This petition for prohibition questions the constitutionality and legality of the permanent appointments extended by the President of the Philippines to the respondents Chairman and Members of the National Labor Relations Commission (NLRC), without submitting the same to the Commission on Appointments for confirmation pursuant to Art. 215 of the Labor Code as amended by said RA 6715.

Issue:Whether RA 6715 is constitutional insofar as it requires confirmation by the Commission on

Appointments the appointment of the Chairman and Commissioners of the NLRC by the president . whether or not Congress may, by law, require confirmation by the Commission on Appointments of appointments extended by the President to government officers additional to those expressly mentioned in the first sentence of Sec. 16, Art. VII of the Constitution whose appointments require confirmation by the Commission on Appointments.

Held: No. From the cases of Sarmiento III vs. Mison (156 SCRA 549); Mary Concepcion Bautista v.

Salonga (172 SCRA 160), and Teresita Quintos Deles, et al. v. the Commission on Constitutional Commissions, et al (177 SCRA 259), these doctrines are deducible: 1. Confirmation by the Commission on Appointments is required only for presidential appointees mentioned in the first sentence of Section 16, Article VII, including, those officers whose appointments are expressly vested by the Constitution itself in the president (like sectoral representatives to Congress and members of the constitutional commissions of Audit, Civil Service and Election). 2. Confirmation is not required when the President appoints other government officers whose appointments are not otherwise provided for by law or those officers whom he may be authorized by law to appoint (like the Chairman and Members of the Commission on Human Rights). Also, as observed in Mison, when Congress creates inferior offices but omits to provide for appointment thereto, or provides in an unconstitutional manner for such appointments, the officers are considered as among those whose appointments are not otherwise provided for by law.

The second sentence of Sec. 16, Art. VII refers to all other officers of the government whose appointment are not otherwise provided for by law and those whom the President may be authorized by law to appoint. Indubitably, the NLRC Chairman and Commissioners fall within the second sentence of Section 16, Article VII of the Constitution, more specifically under the "third

Page 68: ARTICLE VII the executive department

groups" of appointees referred to in Mison, i.e. those whom the President may be authorized by law to appoint. Undeniably, the Chairman and Members of the NLRC are not among the officers mentioned in the first sentence of Section 16, Article VII whose appointments requires confirmation by the Commission on Appointments. To the extent that RA 6715 requires confirmation by the Commission on Appointments of the appointments of respondents Chairman and Members of the National Labor Relations Commission, it is unconstitutional because: 1) it amends by legislation, the first sentence of Sec. 16, Art. VII of the Constitution by adding thereto appointments requiring confirmation by the Commission on Appointments; and 2) it amends by legislation the second sentence of Sec. 16, Art. VII of the Constitution, by imposing the confirmation of the Commission on Appointments on appointments which are otherwise entrusted only with the President. Deciding on what law to pass is a legislative prerogative. Determining their constitutionality is a judicial function. The Court respects the laudable intention of the legislature. Regretfully, however, the constitutional infirmity of Sec. 13 of RA 6715 amending Art. 215 of the Labor Code, insofar as it requires confirmation of the Commission on Appointments over appointments of the Chairman and Members of the National Labor Relations Commission (NLRC) is, as we see it, beyond redemption if we are to render fealty to the mandate of the Constitution in Sec. 16, Art. VII thereof.

(a) Heads of departments(b) Ambassadors, public ministers and consuls(c) Officers of AFP from colonel and naval captain(d) Chairman and members of the constitutional commission(e) Regular members of JBC (Art VIII, Sec. 8 [2])(f) Sectoral (Art XVIII, Sec. 7)

d.3 Upon Recommendation of JBC(a) members of SC and all other courts (Art VIII, Sec 9)(b) ombudsman and deputies

d.4 Appointment of VP as cabinet member (Sec 3)

d.5 Appointments solely by president (Sec 16)(a) those whose appointments are not otherwise provided by law(b) those whom he may be authorized by law to appoint

d.6 Limitations to appointing powerd.6.1. Art VII, Secs 13 and 15

EN BANC

Page 69: ARTICLE VII the executive department

[G.R. No. L-19313. January 19, 1962.]DOMINADOR R. AYTONA vs. ANDRES V. CASTILLO

Facts:On December 29, 1961, then President Carlos P. Garcia appointed Dominador R. Aytona as ad

interim Governor of the Central Bank. On the same day, the latter took the corresponding oath.

On December 30, 1961, at noon, President-elect Diosdado Macapagal assumed office; and on December 31, 1961, he issued Administrative Order No. 2 recalling, withdrawing, and cancelling all ad interim appointments made by President Garcia after December 13, 1961 (date when he, Macapagal, had been proclaimed elected by the Congress). On January 1, 1962, President Macapagal appointed Andres V. Castillo as ad interim Governor of the Central Bank, and the latter qualified immediately.

Issue:Whether the new President had power to issue the order of cancellation of the ad interim

appointments made by the past President, even after the appointees had already qualified.

Held:As a rule, once an appointment is issued, it cannot be reconsidered specially where the

appointee has qualified. On the other hand, the authorities admit of exceptional circumstances justifying revocation such as when mass ad-interim appointments (350) issued in the last hours of an outgoing Chief Executive are to be considered by the Commission on Appointments that is different from that be submitted by an incoming Chief Executive who may not wholly approve of the selections especially if it is doubtful that the outgoing President exercised double care in extending such appointments.

After the proclamation of the election of an incoming Chief Executive, the outgoing Chief Executive is no more than a "care- taker" administration. He is duty bound to prepare for the orderly transfer of authority to the incoming President and he should not do acts which, he ought to know, would embarrass or obstruct the policies, of his successor. It is not for him to use his powers as incumbent President to continue the political warfare that had ended or to avail himself of presidential prerogatives to serve partisan purposes.

The filling up of vacancies in important positions, if few, and so spaced as to afford some assurance of deliberate action and careful consideration of the need for the appointment and the appointee's qualifications may be undoubtedly permitted. But the issuance of 350 appointments in one night and the planned induction of almost all of them a few hours before the inauguration of the new President may, with some reason, be regarded by the latter as an abuse of presidential prerogatives.

When the President makes ad-interim appointments, he exercises a special prerogative and is bound to be prudent to insure approval of his selection either by previous consultation with the members of the Commission or by thereafter explaining to them the reason for such selection. Where the Commission on Appointments that will consider the appointees is different from that existing at the time of the appointment and where the names are to be submitted by his successor

Page 70: ARTICLE VII the executive department

who may not wholly approved of the selections, the President should be doubly careful in extending such appointments.

EN BANC[G.R. No. L-21776. February 28, 1964.]NICANOR G. JORGE vs. JOVENCIO Q. MAYOR

Facts:Petitioner, Nicanor G. Jorge, is a career official in the Bureau of Lands. He started working

there as a Junior Computer in the course of 38 years service, from February 1, 1922 to October 31, 1960, and attained the position of Acting Director, through regular and successive promotions, in accordance with civil service rules. On June 17, 1961, he was designated Acting Director of the same Bureau, and on December 13, 1961 was appointed by President Carlos Garcia ad interim Director. He qualified by taking the oath of office on the 23rd December of 1961. This appointment was on December 26, 1961, transmitted to the Commission on Appointments, and on May 14, 1962, petitioner's ad interim appointment as Director of Lands was confirmed by the Commission.

Petitioner discharged the duties as Director until on November 14, 1962 he received a letter from Benjamin Gozon, then Secretary of Agriculture and Natural Resources of the Macapagal Administration, informing him that pursuant to a letter from the Assistant Executive Secretary Bernal, served on petitioner on November 13, his appointment was among those revoked by Administrative Order No. 2 of President Diosdado Macapagal; that the position of Director of Lands was considered vacant.

Issue: Whether the Jorge appointment is similarly situated as those in Aytona case and therefore invalid.

Held:Petitioner Jorge's ad interim appointment is dated December 13, 1961, but there is no

evidence on record that it was made and released after the joint session of Congress that ended on the same day. It is a matter of contemporary history, of which this Court may take judicial cognizance, that the session ended late in the night of December 13, 1961, and, therefore, after regular office hours. In the absence of competent evidence to the contrary, it is to be presumed that the appointment of Jorge was made before the close of office hours, that being the regular course of business. The appointment, therefore, was not included in, nor intended to be covered by, Administrative Order No. 2, and the same stands unrevoked. Consequently, it was validly confirmed by the Commission on Appointments, and thereafter, the office never became vacant.

It is an error to consider petitioner's case as within the purview of our ruling in the Aytona vs. Castillo case (L-19313, Jan. 20, 1962). If in that case this Court refused to interfere with the application of the Chief Executive's Administrative Order No. 2, it was because the circumstances of the appointments therein involved rendered it doubtful whether the appointees' equitable rights could be invoked,"considering the rush conditional appointments, hurried maneuvers and other

Page 71: ARTICLE VII the executive department

happenings detracting from that degree of good faith, morality and propriety which form the basic foundation of claims to equitable relief."

There is certainly no parity between the appointment of petitioner in December 13, 1961 and the confused scramble for appointments in and during the days immediately preceding the inauguration of the present administration. For aught that appears on the record before us, the appointment of petitioner Jorge was the only one made in that day, and there is nothing to show that it was not —"so spaced as to afford some assurance of deliberate action and careful consideration of the need for the appointment and the appointee's qualifications."That could be validly made even by an outgoing President under the Aytona ruling.

EN BANC[G.R. No. L-19981. February 29, 1964.]GODOFREDO QUIMSING vs. EDUARDO TAJANGLANGIT

Facts:On May 20, 1960, Godofredo Quimsing was designated Acting Chief of Police of Iloilo City. On

December 20, 1961, and while such incumbent of the office, he was extended by then President Garcia an ad-interim appointment to the same position. Quimsing took his oath of office before the City Mayor of Iloilo on December 28, 1961, and continued discharging the functions of Chief of Police of said City.

At the session of the Commission on Appointments on May 16, 1962, the appointment of Quimsing, among others was confirmed. On the following day, however, at the session of said body, Senator Puyat moved for the reconsideration of all the appointments previously confirmed, manifesting at the same time that said "motion for reconsideration be laid on the table."

On June 11, 1962, President Macapagal designated Eduardo Tajanglangit as acting Chief of Police of Iloilo City and the latter took his oath and tried to discharge the functions of the office on June 13, 1962. Respondent Tajanglangit, in his answer, claimed among others, that petitioner's ad-interim appointment was a nullity in view of the President's Administrative Order No. 2, withdrawing, cancelling, or recalling ad-interim appointments extended after December 13, 1961; and that the alleged confirmation of petitioner's ad-interim appointment by the Commission on Appointments did not also produce any effect, because the same had been the subject of a motion for reconsideration and no further action has been taken on said appointment until the present time.

Issue:Whether the Quimsing appointment is similarly situated as those rejected in the Aytona case,

hence illegal.

Held:No. In the various cases decided by this Court after the Aytona v. Castillo case, 1 the matter

of the validity of appointments made after December 13, 1961 by former President Garcia was

Page 72: ARTICLE VII the executive department

considered not in the light of the said Administrative Order No. 2 (which was never upheld by this Court), but on the basis of the nature, character and merit of the individual appointments and the particular circumstances surrounding the same. In other words, this Court did not declare that all the ad-interim appointments made by the outgoing President after December 13, 1961 are invalid by the mere fact that the same were extended after said date, nor they automatically come within the category of the, "midnight" appointments, the validity of which were doubted and which gave rise to the ruling in the Aytona case cited by respondent.

In the present case, petitioner Quimsing admittedly had been occupying the position in controversy, in an acting capacity since May 20,1960, and discharging the functions thereof. Clearly, it cannot be said that the ad-interim appointment extended to him on December 20, 1961, by virtue of which he took his oath of office on December 28, 1961 was one of those hurried designations that brought about the "scramble" on the 29th and 30th of December, 1961, where the outgoing Chief Executive perhaps did not have the opportunity to consider the merits and qualifications of the hundreds of nominees to the positions to which they were respectively being appointed. The ad-interim appointment of petitioner, whose qualification is not in dispute and the regularity of which is not questioned except for the fact that it was made only on December 20, 1961, can not be considered as among those "midnight" appointments the validity of which this Court declared to be, at least, doubtful to entitle the appointees to the equitable relief of quo warranto".

d.6.2. Interim or recess appointmentsArt VI, Sec 19Art VII, Sec 16, par 2

[G.R. No. L-25577. March 15, 1966.]ONOFRE P. GUEVARA vs. RAOUL M. INOCENTES

Facts:Petitioner was extended an ad interim appointment as undersecretary of Labor by the former

Executive on November 18, 1965, having taken his oath of office on November 25 of the same year, and considering that the ad interim appointment for the same position extended to respondent by the incumbent Executive on January 23, 1966 is invalid in spite of Memorandum Circular No. 8 issued by the latter on the same date declaring all ad interim appointments made by the former Executive as having lapsed with the adjournment of the special session of Congress at about midnight of January 22, 1966

Petitioner argued that his position is subsisting because the term “session” in Article VII, section 10, subsection 4 of the Constitution refers to a regular session and not a special session; and because the commission on appointment has not yet been constituted.

Issue:Whether the ad interim appointment of the petitioner has already expired.

Held:

Page 73: ARTICLE VII the executive department

Yes. After due deliberation, the Court resolved that the ad interim appointment extended to petitioner on November 18, 1965 by the former Executive lapsed when the special session of Congress adjourned sine die at about midnight of January 22, 1966, as embodied in our resolution dated February 16, 1966.Petitioner's theory that the first mode of termination consisting in the disapproval by the Commission on Appointments should be inseparably related with the clause "until the next adjournment of Congress" in the sense that the Commission has to be first organized in order that the last mode may operate is untenable considering that the latter is not dependent upon, nor influenced in any manner by, the operation of the former. As already stated, the two modes of termination are completely separate from and independent of each other. If the framers of the Constitution had intended to make the operation of the second clause dependent upon the prior constitution of the Commission on Appointments they should have so stated in clear terms considering that the first clause implies a positive act of the Commission while the second an entirely separate and independent act of Congress. Indeed, the theory of petitioner, if carried to its logical conclusion, may result into the anomaly that, should Congress be controlled by a party not inclined to organize said Commission, or should there arise a group which for reasons of its own indulges in obstructionism, the Commission on Appointments contemplated in the Constitution is never organized as a consequence of the action of either, any appointment made during the recess of Congress would never run the test of legislative scrutiny and would thereby then be always considered permanent even if it is extended ad interim, a result which, to be sure, was never intended by the framers of our Constitution. It thus becomes imperative that we avoid such absurd result.

It is true that the phrases "until the next adjournment of the Congress" does not make any reference to any specific session of the Congress — whether regular or special — but such silence is of no moment, for it is a well-known maxim in statutory construction that when the law does not distinguish, the courts should not distinguish. Consequently, it is safe to conclude that the framers of the Constitution in employing merely the word adjournment as a mode of terminating an appointment made during the recess of Congress had in mind either the regular or special session, and not simply the regular one.

The power to appoint is inherently an executive function while the power to confirm or reject appointments belongs to the legislative department, the latter power having been conferred as a check on the former. This power to check may be exercised through the members of both Houses in the Commission on Appointments. But although the Commission on Appointments is provided for in the Constitution, its organization requires congressional action, and once organized, by express provision of the Constitution, it "shall meet only while Congress is in session." Consequently, if for any reason Congress adjourns a regular or special session without organizing the Commission on Appointments, Congress should be deemed to have impliedly exercised said power to check by allowing the ad interim appointment to lapse as provided for in the Constitution.

EN BANC[G.R. No. 131136. February 28, 2001.]CONRADO L. DE RAMA vs. THE COURT OF APPEALS

Page 74: ARTICLE VII the executive department

Facts:Upon assumption as Mayor of Pagbilao, Quezon, Conrado L. de Rama wrote a letter to the

Civil Service Commission (or CSC), seeking the recall of the appointments of fourteen municipal employees on the ground that those were "midnight" appointments of the former mayor in violation of Article VII, Section 15 of the Constitution. While the matter was pending before the CSC, Elsa Marino, Morell Ayala and Flordeliza Oriazel filed with the CSC a claim for payment of their salaries which were withheld pursuant to Office Order No. 95-01, issued by de Rama wherein their appointments as permanent employees were recalled.

Issue:Whether the “midnight appointment” ban against the president applies to local chiefs.

Held:No. The only reason advanced by the petitioner to justify the recall was that these were

"midnight appointments." The CSC correctly ruled, however, that the constitutional prohibition on so-called "midnight appointments," specifically those made within two (2) months immediately prior to the next presidential elections, applies only to the President or Acting President.

It has been held that upon the issuance of an appointment and the appointee's assumption of the position in the civil service, "he acquires a legal right which cannot be taken away either by revocation of the appointment or by removal except for cause and with previous notice and hearing." Moreover, it is well-settled that the person assuming a position in the civil service under a completed appointment acquires a legal, not just an equitable, right to the position. This right is protected not only by statute, but by the Constitution as well, which right cannot be taken away by either revocation of the appointment, or by removal, unless there is valid cause to do so, provided that there is previous notice and hearing.

A thorough perusal of the records reveal that the CSC's ruling is supported by the evidence and the law. The fourteen (14) employees were duly appointed following two meetings of the Personnel Selection Board held on May 31 and June 26, 1995. There is no showing that any of the private respondents were not qualified for the positions they were appointed to. Moreover, their appointments were duly attested to by the Head of the CSC field office at Lucena City. By virtue thereof, they had already assumed their appointive positions even before petitioner himself assumed his elected position as town mayor. Consequently, their appointments took effect immediately and cannot be unilaterally revoked or recalled by petitioner.

EN BANC[G.R. No. 149036. April 2, 2002.]MA. J. ANGELINA G. MATIBAG vs. ALFREDO L. BENIPAYO

Facts:

Page 75: ARTICLE VII the executive department

Petitioner questioned the constitutionality of the appointment and the right to hold office of respondents Alfredo L. Benipayo, as Chairman of the Commission on Elections, and Resurreccion Z. Borra and Florentino A. Tuason, Jr., as COMELEC Commissioners. The first ad interim appointment of the respondents were by-passed by Commission on Appointment (failed to act) and such appointment was later on renewed by the president. Petitioner claimed that the ad interim appointments and reappointments of the respondents violated the constitutional provisions on the independence of the COMELEC, as well as on the prohibitions on temporary appointments and reappointments of its Chairman and members under Section 1 (2), Article IX-C of the Constitution.

Issues:1. What is the nature of ad interim appointment, permanent or temporary? 2. Does the renewal of the ad interim appointment when the same was by-passed by the CA

constitute reappointment, hence violative of the constitution?

Held:First Issue: It is permanent.Second Issue: It does not constitute renewal.

Discussion:An ad interim appointment is a permanent appointment because it takes effect immediately and can no longer be withdrawn by the President once the appointee has qualified into office. The fact that it is subject to confirmation by the Commission on Appointments does not alter its permanent character. The Constitution itself makes an ad interim appointment permanent in character by making it effective until disapproved by the Commission on Appointments or until the next adjournment of Congress. The second paragraph of Section 16, Article VII of the Constitution provides as follows: "The President shall have the power to make appointments during the recess of the Congress, whether voluntary or compulsory, but such appointments shall be effective only until disapproval by the Commission on Appointments or until the next adjournment of the Congress." Thus, the ad interim appointment remains effective until such disapproval or next adjournment, signifying that it can no longer be withdrawn or revoked by the President. The fear that the President can withdraw or revoke at any time and for any reason an ad interim appointment is utterly without basis.

The Constitution imposes no condition on the effectivity of an ad interim appointment, and thus an ad interim appointment takes effect immediately. The appointee can at once assume office and exercise, as a de jure officer, all the powers pertaining to the office. In Pacete vs. Secretary of the Commission on Appointments, this Court elaborated on the nature of an ad interim appointment as follows: "A distinction is thus made between the exercise of such presidential prerogative requiring confirmation by the Commission on Appointments when Congress is in session and when it is in recess. In the former, the President nominates, and only upon the consent of the Commission on Appointments may the person thus named assume office. It is not so with reference to ad interim appointments. It takes effect at once. The individual chosen may thus qualify and perform his function without loss of time. His title to such office is complete. In the

Page 76: ARTICLE VII the executive department

language of the Constitution, the appointment is effective 'until disapproval by the Commission on Appointments or until the next adjournment of the Congress."

Petitioner cites Black's Law Dictionary which defines the term "ad interim" to mean "in the meantime" or "for the time being." Hence, petitioner argues that an ad interim appointment is undoubtedly temporary in character. This argument is not new and was answered by this Court in Pamantasan ng Lungsod ng Maynila vs. Intermediate Appellate Court, where we explained that: ". . . From the arguments, it is easy to see why the petitioner should experience difficulty in understanding the situation. Private respondent had been extended several 'ad interim' appointments which petitioner mistakenly understands as appointments temporary in nature. Perhaps, it is the literal translation of the word 'ad interim' which creates such belief. The term is defined by Black to mean "in the meantime" or "for the time being." Thus, an officer ad interim is one appointed to fill a vacancy, or to discharge the duties of the office during the absence or temporary incapacity of its regular incumbent (Black's Law Dictionary, Revised Fourth Edition, 1978). But such is not the meaning nor the use intended in the context of Philippine law. In referring to Dr. Esteban's appointments, the term is not descriptive of the nature of the appointments given to him. Rather, it is used to denote the manner in which said appointments were made, that is, done by the President of the Pamantasan in the meantime, while the Board of Regents, which is originally vested by the University Charter with the power of appointment, is unable to act. . . .." Thus, the term "ad interim appointment," as used in letters of appointment signed by the President, means a permanent appointment made by the President in the meantime that Congress is in recess. It does not mean a temporary appointment that can be withdrawn or revoked at any time. The term, although not found in the text of the Constitution, has acquired a definite legal meaning under Philippine jurisprudence. The Court had again occasion to explain the nature of an ad interim appointment in the more recent case of Marohombsar vs. Court of Appeals, where the Court stated: "We have already mentioned that an ad interim appointment is not descriptive of the nature of the appointment, that is, it is not indicative of whether the appointment is temporary or in an acting capacity, rather it denotes the manner in which the appointment was made.

An ad interim appointment can be terminated for two causes specified in the Constitution. The first cause is the disapproval of his ad interim appointment by the Commission on Appointments. The second cause is the adjournment of Congress without the Commission on Appointments acting on his appointment. These two causes are resolutory conditions expressly imposed by the Constitution on all ad interim appointments. These resolutory conditions constitute, in effect, a Sword of Damocles over the heads of ad interim appointees. No one, however, can complain because it is the Constitution itself that places the Sword of Damocles over the heads of the ad interim appointees.

While an ad interim appointment is permanent and irrevocable except as provided by law, an appointment or designation in a temporary or acting capacity can be withdrawn or revoked at the pleasure of the appointing power. A temporary or acting appointee does not enjoy any security of tenure, no matter how briefly. This is the kind of appointment that the Constitution prohibits the President from making to the three independent constitutional commissions, including the COMELEC. Thus, in Brillantes vs. Yorac, this Court struck down as unconstitutional the designation by then President Corazon Aquino of Associate Commissioner Haydee Yorac as Acting Chairperson

Page 77: ARTICLE VII the executive department

of the COMELEC. This Court ruled that: "A designation as Acting Chairman is by its very terms essentially temporary and therefore revocable at will. No cause need be established to justify its revocation. Assuming its validity, the designation of the respondent as Acting Chairman of the Commission on Elections may be withdrawn by the President of the Philippines at any time and for whatever reason she sees fit. It is doubtful if the respondent, having accepted such designation, will not be estopped from challenging its withdrawal. . . . The Constitution provides for many safeguards to the independence of the Commission on Elections, foremost among which is the security of tenure of its members. That guarantee is not available to the respondent as Acting Chairman of the Commission on Elections by designation of the President of the Philippines."

Earlier, in Nacionalista Party vs. Bautista, a case decided under the 1935 Constitution, which did not have a provision prohibiting temporary or acting appointments to the COMELEC, this Court nevertheless declared unconstitutional the designation of the Solicitor General as acting member of the COMELEC. This Court ruled that the designation of an acting Commissioner would undermine the independence of the COMELEC and hence violate the Constitution. We declared then: "It would be more in keeping with the intent, purpose and aim of the framers of the Constitution to appoint a permanent Commissioner than to designate one to act temporarily." In the instant case, the President did in fact appoint permanent Commissioners to fill the vacancies in the COMELEC, subject only to confirmation by the Commission on Appointments. Benipayo, Borra and Tuason were extended permanent appointments during the recess of Congress. They were not appointed or designated in a temporary or acting capacity, unlike Commissioner Haydee Yorac in Brillantes vs. Yorac and Solicitor General Felix Bautista in Nacionalista Party vs. Bautista. The ad interim appointments of Benipayo, Borra and Tuason are expressly allowed by the Constitution which authorizes the President, during the recess of Congress, to make appointments that take effect immediately.

While the Constitution mandates that the COMELEC "shall be independent," this provision should be harmonized with the President's power to extend ad interim appointments. To hold that the independence of the COMELEC requires the Commission on Appointments to first confirm ad interim appointees before the appointees can assume office will negate the President's power to make ad interim appointments. This is contrary to the rule on statutory construction to give meaning and effect to every provision of the law. It will also run counter to the clear intent of the framers of the Constitution.

The reinstatement in the present Constitution of the ad interim appointing power of the President was for the purpose of avoiding interruptions in vital government services that otherwise would result from prolonged vacancies in government offices, including the three constitutional commissions. In his concurring opinion in Guevara vs. Inocentes, decided under the 1935 Constitution, Justice Roberto Concepcion, Jr. explained the rationale behind ad interim appointments in this manner: "Now, why is the lifetime of ad interim appointments so limited? Because, if they expired before the session of Congress, the evil sought to be avoided — interruption in the discharge of essential functions — may take place. Because the same evil would result if the appointments ceased to be effective during the session of Congress and before its adjournment. Upon the other hand, once Congress has adjourned, the evil aforementioned may easily be conjured by the issuance of other ad interim appointments or reappointments." Indeed, the timely application of the last sentence of Section 16, Article VII of the Constitution barely

Page 78: ARTICLE VII the executive department

avoided the interruption of essential government services in the May 2001 national elections. Following the decision of this Court in Gaminde vs. Commission on Appointments, promulgated on December 13, 2000, the terms of office of constitutional officers first appointed under the Constitution would have to be counted starting February 2, 1987, the date of ratification of the Constitution, regardless of the date of their actual appointment. By this reckoning, the terms of office of three Commissioners of the COMELEC, including the Chairman, would end on February 2, 2001.

Evidently, the exercise by the President in the instant case of her constitutional power to make ad interim appointments prevented the occurrence of the very evil sought to be avoided by the second paragraph of Section 16, Article VII of the Constitution. This power to make ad interim appointments is lodged in the President to be exercised by her in her sound judgment. Under the second paragraph of Section 16, Article VII of the Constitution, the President can choose either of two modes in appointing officials who are subject to confirmation by the Commission on Appointments. First, while Congress is in session, the President may nominate the prospective appointee, and pending consent of the Commission on Appointments, the nominee cannot qualify and assume office. Second, during the recess of Congress, the President may extend an ad interim appointment which allows the appointee to immediately qualify and assume office.

There is no dispute that an ad interim appointee disapproved by the Commission on Appointments can no longer be extended a new appointment. The disapproval is a final decision of the Commission on Appointments in the exercise of its checking power on the appointing authority of the President. The disapproval is a decision on the merits, being a refusal by the Commission on Appointments to give its consent after deliberating on the qualifications of the appointee. Since the Constitution does not provide for any appeal from such decision, the disapproval is final and binding on the appointee as well as on the appointing power. In this instance, the President can no longer renew the appointment not because of the constitutional prohibition on reappointment, but because of a final decision by the Commission on Appointments to withhold its consent to the appointment.

An ad interim appointment that is by-passed because of lack of time or failure of the Commission on Appointments to organize is another matter. A by-passed appointment is one that has not been finally acted upon on the merits by the Commission on Appointments at the close of the session of Congress. There is no final decision by the Commission on Appointments to give or withhold its consent to the appointment as required by the Constitution. Absent such decision, the President is free to renew the ad interim appointment of a by-passed appointee. This is recognized in Section 17 of the Rules of the Commission on Appointments, which provides as follows: "Section 17. Unacted Nominations or Appointments Returned to the President. Nominations or appointments submitted by the President of the Philippines which are not finally acted upon at the close of the session of Congress shall be returned to the President and, unless new nominations or appointments are made, shall not again be considered by the Commission." Hence, under the Rules of the Commission on Appointments, a by-passed appointment can be considered again if the President renews the appointment.

It is well settled in this jurisdiction that the President can renew the ad interim appointments of by-passed appointees. Justice Roberto Concepcion, Jr. lucidly explained in his concurring opinion in Guevara vs. Inocentes why by-passed ad interim appointees could be

Page 79: ARTICLE VII the executive department

extended new appointments, thus: "In short, an ad interim appointment ceases to be effective upon disapproval by the Commission, because the incumbent can not continue holding office over the positive objection of the Commission. It ceases, also, upon "the next adjournment of the Congress," simply because the President may then issue new appointments — not because of implied disapproval of the Commission deduced from its inaction during the session of Congress, for, under the Constitution, the Commission may affect adversely the interim appointments only by action, never by omission. If the adjournment of Congress were an implied disapproval of ad interim appointments made prior thereto, then the President could no longer appoint those so by-passed by the Commission. But, the fact is that the President may reappoint them, thus clearly indicating that the reason for said termination of the ad interim appointments is not the disapproval thereof allegedly inferred from said omission of the Commission, but the circumstance that upon said adjournment of the Congress, the President is free to make ad interim appointments or reappointments."

The prohibition on reappointment in Section 1 (2), Article IX-C of the Constitution applies neither to disapproved nor by-passed ad interim appointments. A disapproved ad interim appointment cannot be revived by another ad interim appointment because the disapproval is final under Section 16, Article VII of the Constitution, and not because a reappointment is prohibited under Section 1(2), Article IX-C of the Constitution. A by-passed ad interim appointment can be revived by a new ad interim appointment because there is no final disapproval under Section 16, Article VII of the Constitution, and such new appointment will not result in the appointee serving beyond the fixed term of seven years.

The phrase "without reappointment" applies only to one who has been appointed by the President and confirmed by the Commission on Appointments, whether or not such person completes his term of office. There must be a confirmation by the Commission on Appointments of the previous appointment before the prohibition on reappointment can apply. To hold otherwise will lead to absurdities and negate the President's power to make ad interim appointments. In the great majority of cases, the Commission on Appointments usually fails to act, for lack of time, on the ad interim appointments first issued to appointees. If such ad interim appointments can no longer be renewed, the President will certainly hesitate to make ad interim appointments because most of her appointees will effectively be disapproved by mere inaction of the Commission on Appointments. This will nullify the constitutional power of the President to make ad interim appointments, a power intended to avoid disruptions in vital government services. This Court cannot subscribe to a proposition that will wreak havoc on vital government services.

The ad interim appointments and subsequent renewals of appointments of Benipayo, Borra and Tuason do not violate the prohibition on reappointments because there were no previous appointments that were confirmed by the Commission on Appointments. A reappointment presupposes a previous confirmed appointment. The same ad interim appointments and renewals of appointments will also not breach the seven-year term limit because all the appointments and renewals of appointments of Benipayo, Borra and Tuason are for a fixed term expiring on February 2, 2008. Any delay in their confirmation will not extend the expiry date of their terms of office. Consequently, there is no danger whatsoever that the renewal of the ad interim appointments of these three respondents will result in any of the evils intended to be exorcised by the twin prohibitions in the Constitution. The continuing renewal of the ad interim appointment of these

Page 80: ARTICLE VII the executive department

three respondents, for so long as their terms of office expire on February 2, 2008, does not violate the prohibition on reappointments in Section 1 (2), Article IX-C of the Constitution.

d.6.3. Temporary DesignationsAdm. Code of 1987, Book III, Sec 17

d.6.4. Limitations on Appointing Power of Acting President (Secs 14-15)

2. PARDONING POWER (Sec 19)

Art IX, C, Sec. 5

e.1. pardon distinguished from probation

[G.R. No. 45685. December 22, 1937.]THE PEOPLE OF THE PHILIPPINES vs. JOSE O. VERAFacts:

e.2. pardon distibguished from parole

EN BANC[G.R. No. 76872. July 23, 1987.]WILFREDO TORRES Y SUMULONG vs. HON. NEPTALI A. GONZALES

Facts:Petitioner was convicted by the Court of First Instance of Manila of the crime of estafa. On 18

April 1979, a conditional pardon was granted to the petitioner by the President of the Philippines on condition that petitioner would "not again violate any of the penal laws of the Philippines. Should this condition be violated, he will be proceeded against in the manner prescribed by law." Petitioner accepted the conditional pardon and was consequently released from confinement.

On 21 May 1986, the Board of Pardons and Parole (the "Board") resolved to recommend to the President the cancellation of the conditional pardon granted to the petitioner. In making its recommendation to the President, the Board relied upon the decisions of this Court in Tesoro vs. Director of Prisons (68 Phil. 154 [1939]) and Espuelas vs. Provincial Warden of Bohol (108 Phil. 356 [1960]). The evidence before the Board showed that on 22 March 1982 and 24 June 1982, petitioner had been charged with twenty counts of estafa in Criminal Cases Nos. Q-19672 and Q-

Page 81: ARTICLE VII the executive department

20756, which cases were then (on 21 May 1986) pending trial before the Regional Trial Court of Rizal (Quezon City). The record before the Board also showed that on 26 June 1985, petitioner had been convicted by the Regional Trial Court of Rizal (Quezon City) of the crime of sedition in Criminal Case No. Q-22926: this conviction was then pending appeal before the Intermediate Appellate Court. The Board also had before it a letter report dated 14 January 1986 from the National Bureau of Investigation ("NBI"), addressed to the Board, on the petitioner. Per this letter, the records of the NBI showed that a long list of charges had been brought against the petitioner during the last twenty years for a wide assortment of crimes including estafa, other forms of swindling, grave threats, grave coercion, illegal possession of firearms, ammunition and explosives, malicious mischief, violation of Batas Pambansa Blg. 22, and violation of Presidential Decree No. 772 (interfering with police functions). Some of these charges were identified in the NBI report as having been dismissed. The NBI report did not purport to be a status report on each of the charges there listed and identified.

On 10 October 1986, the respondent Minister of Justice issued "by authority of the President" an Order of Arrest and Recommitment against petitioner. The petitioner was accordingly arrested and confined in Muntinlupa to serve the unexpired portion of his sentence.

Petitioner now impugns the validity of the Order of Arrest and Recommitment. He claims that he did not violate his conditional pardon since he has not been convicted by final judgment of the twenty (20) counts of estafa charged in Criminal Cases Nos. Q-19672 and Q-20756 nor of the crime of sedition in Criminal Case No. Q-22926 3 Petitioner also contends that he was not given an opportunity to be heard before he was arrested and recommitted to prison, and accordingly claims he has been deprived of his rights under the due process clause of the Constitution.

Issue:Whether the pending charges suffice to constitute breach of the conditional pardon.

Held:No. It may be emphasized that what is involved in the instant case is not the prosecution of

the parolee for a subsequent offense in the regular course of administration of the criminal law. What is involved is rather the ascertainment of whether the convict has breached his undertaking that he would "not again violate any of the penal laws of the Philippines" for purposes of reimposition upon him of the remitted portion of his original sentence. The consequences that we here deal with are the consequences of an ascertained breach of the conditions of a pardon. A convict granted conditional pardon, like the petitioner herein, who is recommitted must of course be convicted by final judgment of a court of the subsequent crime or crimes with which he was charged before the criminal penalty for such subsequent offense(s) can be imposed upon him. Again, since Article 159 of the Revised Penal Code defines a distinct, substantive, felony, the parolee or convict who is regarded as having violated the provisions thereof must be charged, prosecuted and convicted by final judgment before he can be made to suffer the penalty prescribed in Article 159.

Succinctly put, in proceeding against a convict who has been conditionally pardoned and who is alleged to have breached the conditions of his pardon, the Executive Department has two options: (i) to proceed against him under Section 64 (i) of the Revised Administrative Code; or (ii) to proceed against him under Article 159 of the Revised Penal Code which imposes the penalty of

Page 82: ARTICLE VII the executive department

prision correccional, minimum period, upon a convict who "having been granted conditional pardon by the Chief Executive, shall violate any of the conditions of such pardon." Here, the President has chosen to proceed against the petitioner under Section 64 (i) of the Revised Administrative Code. That choice is an exercise of the President's executive prerogative and is not subject to judicial scrutiny.

e.3. pardon distinguished from amnesty

EN BANC[G.R. No. L-1278. January 21, 1949.]LORETO BARRIOQUINTO and NORBERTO JIMENEZ vs. ENRIQUE A. FERNANDEZ

Facts:Petitioners Norberto Jimenez and Loreto Barrioquinto were charged with the crime of

murder. As the latter had not yet been arrested the case proceeded against the former, and after trial the Court of First Instance of Zamboanga sentenced Jimenez to life imprisonment. Before the period for perfecting an appeal had expired, the defendant Jimenez became aware of the Proclamation No. 8, dated September 7, 1946, which grants amnesty in favor of all persons who may be charged with an act penalized under the Revised Penal Code in furtherance of the resistance to the enemy or against persons aiding in the war efforts of the enemy, and committed during the period from December 8, 1941, to the date when each particular area of the Philippines where the offense was actually committed was liberated from enemy control and occupation, and said Jimenez decided to submit his case to the Guerrilla Amnesty Commission presided by the respondents herein, and the other petitioner Loreto Barrioquinto, who had then been already apprehended, did the same.

The Amnesty Commission denied to extend the benefits of the Amnesty on the ground that Jimenez and Barrioquinto did not admit their commission of the crime charged.

Issue:Whether the petitioners can invoke the benefits of the amnesty regardless of whether there

is prior admission of the offense charged.

Held:Yes. Pardon is granted by the Chief Executive and as such it is a private act which must be

pleaded and proved by the person pardoned, because the courts take no notice thereof; while amnesty by Proclamation of the Chief Executive with the concurrence of Congress, and it is a public act of which the courts should take judicial notice. Pardon is granted to one after conviction; while amnesty is granted to classes of persons or communities who may be guilty of political offenses, generally before or after the institution of the criminal prosecution and sometimes after conviction.

Page 83: ARTICLE VII the executive department

Pardon looks forward and relieves the offender from the consequences of an offense of which he has been convicted, that is, it abolishes or forgives the punishment, and for that reason it does "nor work the restoration of the rights to hold public office, or the right of suffrage, unless such rights be expressly restored by the terms of the pardon," and it "in no case exempt the culprit from the payment of the civil indemnity imposed upon him by the sentence" (article 36, Revised Penal Code). While amnesty looks backward and abolishes and puts into oblivion the offense itself, it so overlooks and obliterates the offense with which he is charged that the person released by amnesty stands before the law precisely as though he had committed no offense.

In view of the foregoing, we are of the opinion and so hold that, in order to entitle a person to the benefits of the Amnesty Proclamation of September 7, 1946, it is not necessary that he should, as a condition precedent or sine qua non, admit having committed the criminal act or offense with which he is charged, and allege the amnesty as a defense; it is sufficient that the evidence, either of the complainant or the accused, shows that the offense committed comes within the terms of said Amnesty Proclamation. Hence, it is not correct to say that "invocation of the benefits of amnesty is in the nature of a plea of confession and avoidance." Although the accused does not confess the imputation against him, he may be declared by the courts or the Amnesty Commissions entitled to the benefits of the amnesty. For, whether or not he admits or confesses having committed the offense with which he is charged, the Commissions should, if necessary or requested by the interested party, conduct summary hearing of the witnesses both for the complainants and the accused, on whether he has committed the offense in furtherance of the resistance to the enemy, or against persons aiding in the war efforts of the enemy, and decide whether he is entitled to the benefits of amnesty and to be "regarded as a patriot or hero who have rendered invaluable services to the nation," or not, in accordance with the terms of the Amnesty Proclamation. Since the Amnesty Proclamation is a public act, the courts as well as the Amnesty Commissions created thereby should take notice of the terms of said Proclamation and apply the benefits granted therein to cases coming within their province or jurisdiction, whether pleaded or claimed by the person charged with such offenses or not, if the evidence presented shows that the accused is entitled to said benefits.

e.4. effect of pardon

EN BANC[G.R. No. 78239. February 9, 1989.]SALVACION A. MONSANTO vs. FULGENCIO S. FACTORAN, JR.

Facts:Petitioner (then assistant treasurer of Calbayog City)had been convicted of the complex

crime of estafa thru falsification of public documents and sentenced to imprisonment of four years, two months and one day of prision correccional as minimum, to ten years and one day of prision mayor as maximum. The penalty of prision mayor carries the accessory penalties of temporary

Page 84: ARTICLE VII the executive department

absolute disqualification and perpetual special disqualification from the right of suffrage, enforceable during the term of the principal penalty. He was granted pardon. By reason of said pardon, petitioner wrote the Calbayog City treasurer requesting that she be restored to her former post as assistant city treasurer since the same was still vacant.

Petitioner's letter-request was referred to the Ministry of Finance for resolution in view of the provision of the Local Government Code transferring the power of appointment of treasurers from the city governments to the said Ministry. In its 4th Indorsement dated March 1, 1985, the Finance Ministry ruled that petitioner may be reinstated to her position without the necessity of a new appointment not earlier than the date she was extended the absolute pardon. It also directed the city treasurer to see to it that the amount of P4,892.50 which the Sandiganbayan had required to be indemnified in favor of the government as well as the costs of the litigation, be satisfied.

Seeking reconsideration of the foregoing ruling, petitioner wrote the Ministry on April 17, 1985 stressing that the full pardon bestowed on her has wiped out the crime which implies that her service in the government has never been interrupted and therefore the date of her reinstatement should correspond to the date of her preventive suspension which is August 1, 1982; that she is entitled to backpay for the entire period of her suspension; and that she should not be required to pay the proportionate share of the amount of P4,892.50.

Issue:Whether or not a public officer, who has been granted an absolute pardon by the Chief

Executive, is entitled to reinstatement to her former position without need of a new appointment.

Held:No. Temporary absolute disqualification bars the convict from public office or employment,

such disqualification to last during the term of the sentence. Even if the offender be pardoned, as to the principal penalty, the accessory penalties remain unless the same have been expressly remitted by the pardon. The penalty of prision correccional carries, as one of its accessory penalties, suspension from public office.

Pardon is defined as "an act of grace, proceeding from the power entrusted with the execution of the laws, which exempts the individual, on whom it is bestowed, from the punishment the law inflicts for a crime he has committed. It is the private, though official act of the executive magistrate, delivered to the individual for whose benefit it is intended, and not communicated officially to the Court . . . A pardon is a deed, to the validity of which delivery is essential, and delivery is not complete without acceptance.

The modern trend of authorities now rejects the unduly broad language of the Garland case [4 Wall, 333 18 L.ED. 366] (reputed to be perhaps the most extreme statement which has been made on the effects of a pardon). To our mind, this is the more realistic approach. While a pardon has generally been regarded as blotting out the existence of guilt so that in the eye of the law the offender is as innocent as though he never Committed the offense, it does not operate for all purposes. The very essence of a pardon is forgiveness or remission of guilt. Pardon implies guilt. It does not erase the fact of the commission of the crime and the conviction thereof. It does not wash out the moral stain. It involves forgiveness and not forgetfulness. The better considered cases

Page 85: ARTICLE VII the executive department

regard full pardon (at least one not based on the offender's innocence) as relieving the party from all the punitive consequences of his criminal act, including the disqualifications or disabilities based on the finding of guilt. But it relieves him from nothing more. "To say, however, that the offender is a 'new man', and 'as innocent as if he had never committed the offense;' is to ignore the difference between the crime and the criminal. A person adjudged guilty of an offense is a convicted criminal, though pardoned; he may be deserving of punishment, though left unpunished; and the law may regard him as more dangerous to society than one never found guilty of crime, though it places no restraints upon him following his conviction." A pardon looks to the future. It is not retrospective. It makes no amends for the past. It affords no relief for what has been suffered by the offender. It does not impose upon the government any obligation to make reparation for what has been suffered. "Since the offense has been established by judicial proceedings, that which has been done or suffered while they were in force is presumed to have been rightfully done and justly suffered, and no satisfaction for it can be required." This would explain why petitioner, though pardoned, cannot be entitled to receive backpay for lost earnings and benefits.

In this ponencia, the Court wishes to stress one vital point: While we are prepared to concede that pardon may remit all the penal consequences of a criminal indictment if only to give meaning to the fiat that a pardon, being a presidential prerogative, should not be circumscribed by legislative action, we do not subscribe to the fictitious belief that pardon blots out the guilt of an individual and that once he is absolved, he should be treated as if he were innocent. For whatever may have been the judicial dicta in the past, we cannot perceive how pardon can produce such "moral changes" as to equate a pardoned convict in character and conduct with one who has constantly maintained the mark of a good, law-abiding citizen. Pardon cannot mask the acts constituting the crime. These are "historical" facts which, despite the public manifestation of mercy and forgiveness implicit in pardon, "ordinary, prudent men will take into account in their subsequent dealings with the actor." Pardon granted after conviction frees the individual from all the penalties and legal disabilities and restores him to all his civil rights. But unless expressly grounded on the person's innocence (which is rare), it cannot bring back lost reputation for honesty, integrity and fair dealing. This must be constantly kept in mind lest we lose track of the true character and purpose of the privilege.

Notwithstanding the expansive and effusive language of the Garland case, we are in full agreement with the commonly-held opinion that pardon does not ipso facto restore a convicted felon to public office necessarily relinquished or forfeited by reason of the conviction although such pardon undoubtedly restores his eligibility for appointment to that office. The rationale is plainly evident. Public offices are intended primarily for the collective protection, safety and benefit of the common good. They cannot be compromised to favor private interests. To insist on automatic reinstatement because of a mistaken notion that the pardon virtually acquitted one from the offense of estafa would be grossly untenable. A pardon, albeit full and plenary, cannot preclude the appointing power from refusing appointment to anyone deemed to be of bad character, a poor moral risk, or who is unsuitable by reason of the pardoned conviction.

What it the effect of accepted pardon to pending appeal?

Page 86: ARTICLE VII the executive department

It is our view that in the present case, it is not material when the pardon was bestowed, whether before or after conviction, for the result would still be the same. Having accepted the pardon, petitioner is deemed to have abandoned her appeal and her unreversed conviction by the Sandiganbayan assumed the character of finality.

e.5. Who may avail amnesty

3. MILITARY POWERS (Sec 18)Art II, Sec 13Art VIII, Sec 1, par 2

EN BANC[G.R. No. 159085. February 3, 2004.]SANLAKAS vs EXECUTIVE SECRETARY ANGELO REYES

FactsPetitioners questioned the constitutionality of Proclamation No. 427 and General Order No. 4

issued by the President on July 27, 2003 declaring "a state of rebellion" and calling upon the Armed Forces to suppress the rebellion, in the wake of the so-called "Oakwood Incident". The challenged issuances were lifted by the President five (5) days later, on August 1, 2003, through Proclamation No. 435. Party-list organizations, Sanlakas and Partido ng Manggagawa (PM), contend that Section 18, Article VII of the Constitution does not require the declaration of a state of rebellion to call out the armed forces. They further submit that, because of the cessation of the Oakwood occupation, there exists no sufficient factual basis for the proclamation by the President of a state of rebellion for an indefinite period.

Issue:Whether the proclamation of the state of rebellion circumvents the constitutional

requirements for the proclamation of a state of national emergency. Whether such proclamation is equivalent to martial law; thus would result to the suspension of the privilege of the writ of habeas corpus and warrantless arrests.

Held:No. It is true that for the purpose of exercising the calling out power the Constitution does

not require the President to make a declaration of a state of rebellion. Section 18, Article VII provides:

Sec. 18. The President shall be the Commander-in-Chief of all armed forces of the Philippines and whenever it becomes necessary, he may call out such armed forces to

Page 87: ARTICLE VII the executive department

prevent or suppress lawless violence, invasion or rebellion. In case of invasion or rebellion, when the public safety requires it, he may, for a period not exceeding sixty days, suspend the privilege of the writ of habeas corpus or place the Philippines or any part thereof under martial law. Within forty-eight hours from the proclamation of martial law or the suspension of the writ of habeas corpus, the President shall submit a report in person or in writing to the Congress. The Congress, voting jointly, by a vote of at least a majority of all its Members in regular or special session, may revoke such proclamation or suspension, which revocation shall not be set aside by the President. Upon the initiative of the President, the Congress may, in the same manner, extend such proclamation or suspension for a period to be determined by the Congress, if the invasion or rebellion shall persist and public safety requires it.

The Congress, if not in session, shall, within twenty-four hours following such proclamation or suspension, convene in accordance with its rules without need of a call.

The Supreme Court may review, in an appropriate proceeding filed by any citizen, the sufficiency of the factual basis for the proclamation of martial law or the suspension of the privilege of the writ of habeas corpus or the extension thereof, and must promulgate its decision thereon within thirty days from its filing.

A state of martial law does not suspend the operation of the Constitution, nor supplant the functioning of the civil courts or legislative assemblies, nor authorize the conferment of the jurisdiction on military courts and agencies over civilians where civil courts are able to function, nor automatically suspend the privilege of the writ.

The suspension of the privilege of the writ shall apply only to persons judicially charged for rebellion or offenses inherent in or directly connected with invasion.

During the suspension of the privilege of the writ, any person thus arrested or detained shall be judicially charged within three days, otherwise he shall be released.

The above provision grants the President, as Commander-in-Chief, a "sequence" of "graduated power[s]." From the most to the least benign, these are: the calling out power, the power to suspend the privilege of the writ of habeas corpus, and the power to declare martial law. In the exercise of the latter two powers, the Constitution requires the concurrence of two conditions, namely, an actual invasion or rebellion, and that public safety requires the exercise of such power. However, as we observed in Integrated Bar of the Philippines v. Zamora, "[t]hese conditions are not required in the exercise of the calling out power. The only criterion is that 'whenever it becomes necessary,' the President may call the armed forces 'to prevent or suppress lawless violence, invasion or rebellion.'"

Nevertheless, it is equally true that Section 18, Article VII does not expressly prohibit the President from declaring a state of rebellion. Note that the Constitution vests the President not only with Commander-in-Chief powers but, first and foremost, with Executive powers.

The President's authority to declare a state of rebellion springs in the main from her powers as chief executive and, at the same time, draws strength from her Commander-in-Chief powers.

Page 88: ARTICLE VII the executive department

Should there be any "confusion" generated by the issuance of Proclamation No. 427 and General Order No. 4, we clarify that, as the dissenters in Lacson correctly pointed out, the mere declaration of a state of rebellion cannot diminish or violate constitutionally protected rights. Indeed, if a state of martial law does not suspend the operation of the Constitution or automatically suspend the privilege of the writ of habeas corpus, then it is with more reason that a simple declaration of a state of rebellion could not bring about these conditions. At any rate, the presidential issuances themselves call for the suppression of the rebellion "with due regard to constitutional rights."

For the same reasons, apprehensions that the military and police authorities may resort to warrantless arrests are likewise unfounded. In Lacson vs. Perez, supra, majority of the Court held that "[i]n quelling or suppressing the rebellion, the authorities may only resort to warrantless arrests of persons suspected of rebellion, as provided under Section 5, Rule 113 of the Rules of Court, 63 if the circumstances so warrant. The warrantless arrest feared by petitioners is, thus, not based on the declaration of a 'state of rebellion.'" In other words, a person may be subjected to a warrantless arrest for the crime of rebellion whether or not the President has declared a state of rebellion, so long as the requisites for a valid warrantless arrest are present.It is not disputed that the President has full discretionary power to call out the armed forces and to determine the necessity for the exercise of such power. While the Court may examine whether the power was exercised within constitutional limits or in a manner constituting grave abuse of discretion, none of the petitioners here have, by way of proof, supported their assertion that the President acted without factual basis.

The argument that the declaration of a state of rebellion amounts to a declaration of martial law and, therefore, is a circumvention of the report requirement, is a leap of logic. There is no indication that military tribunals have replaced civil courts in the "theater of war" or that military authorities have taken over the functions of civil government. There is no allegation of curtailment of civil or political rights. There is no indication that the President has exercised judicial and legislative powers. In short, there is no illustration that the President has attempted to exercise or has exercised martial law powers.Nor by any stretch of the imagination can the declaration constitute an indirect exercise of emergency powers, which exercise depends upon a grant of Congress pursuant to Section 23 (2), Article VI of the Constitution:Sec. 23. (1) . . . .

(2) In times of war or other national emergency, the Congress may, by law, authorize the President, for a limited period and subject to such restrictions as it may prescribe, to exercise powers necessary and proper to carry out a declared national policy. Unless sooner withdrawn by resolution of the Congress, such powers shall cease upon the next adjournment thereof.

The petitions do not cite a specific instance where the President has attempted to or has exercised powers beyond her powers as Chief Executive or as Commander-in-Chief. The President, in declaring a state of rebellion and in calling out the armed forces, was merely exercising a wedding of her Chief Executive and Commander-in-Chief powers. These are purely executive

Page 89: ARTICLE VII the executive department

powers, vested on the President by Sections 1 and 18, Article VII, as opposed to the delegated legislative powers contemplated by Section 23 (2), Article VI.

EN BANC[G.R. No. 141284. August 15, 2000.]INTEGRATED BAR OF THE PHILIPPINES vs. HON. RONALDO B. ZAMORA

Facts:The President of the Philippines, Joseph Ejercito Estrada, in a verbal directive, ordered the

PNP and the Marines to conduct joint visibility patrols for the purpose of crime prevention and suppression. In compliance with the presidential mandate, the PNP Chief, through Police Chief Superintendent Edgar B. Aglipay, formulated Letter of Instruction 02/2000 (the "LOI") which detailed the manner by which the joint visibility patrols, called Task Force Tulungan, would be conducted. Task Force Tulungan was placed under the leadership of the Police Chief of Metro Manila. Invoking his powers as Commander-in-Chief under Section 18, Article VII of the Constitution, the President directed the AFP Chief of Staff and PNP Chief to coordinate with each other for the proper deployment and utilization of the Marines to assist the PNP in preventing or suppressing criminal or lawless violence. The President also declared that the services of the Marines in the anti-crime campaign are merely temporary in nature and for a reasonable period only, until such time when the situation shall have improved. The Integrated Bar of the Philippines (the "IBP") filed the instant petition to annul LOI 02/2000 and to declare the deployment of the Philippine Marines null and void and unconstitutional, arguing that the deployment of marines in Metro Manila is violative of the Constitution because no emergency situation obtains in Metro Manila as would justify, even only remotely, the deployment of soldiers for law enforcement work; hence, said deployment in derogation of Article II, Section 3 of the Constitution.

Issue:Whether the said deployment violates the constitution.

Held:No. When the President calls the armed forces to prevent or suppress lawless violence,

invasion or rebellion, he necessarily exercises a discretionary power solely vested in his wisdom. This is clear from the intent of the framers and from the text of the Constitution itself. The Court, thus, cannot be called upon to overrule the President's wisdom or substitute its own. However, this does not prevent an examination of whether such power was exercised within permissible constitutional limits or whether it was exercised in a manner constituting grave abuse of discretion. In view of the constitutional intent to give the President full discretionary power to determine the necessity of calling out the armed forces, it is incumbent upon the petitioner to show that the President's decision is totally bereft of factual basis. The present petition fails to discharge such heavy burden as there is no evidence to support the assertion that there exist no justification for calling out the armed forces. There is, likewise, no evidence to support the proposition that grave

Page 90: ARTICLE VII the executive department

abuse was committed because the power to call was exercised in such a manner as to violate the constitutional provision on civilian supremacy over the military. In the performance of this Court's duty of purposeful hesitation" before declaring an act of another branch as unconstitutional, only where such grave abuse of discretion is clearly shown shall the Court interfere with the President's judgment. To doubt is to sustain.

Under Section 18, Article VII of the Constitution, in the exercise of the power to suspend the privilege of the writ of habeas corpus or to impose martial law, two conditions must concur: (1) there must be an actual invasion or rebellion and, (2) public safety must require it. These conditions are not required in the case of the power to call out the Armed Forces. The only criterion is that "whenever it becomes necessary," the President may call the armed forces "to prevent or suppress lawless violence, invasion or rebellion." The implication is that the President is given full discretion and wide latitude in the exercise of the power to call as compared to the two other powers.

The President as Commander-in-Chief has a vast intelligence network to gather information, some of which may be classified as highly confidential or affecting the security of the state. In the exercise of the power to call, on-the-spot decisions may be imperatively necessary in emergency situations to avert great loss of human lives and mass destruction of property. Indeed, the decision to call out the military to prevent or suppress lawless violence must be done swiftly and decisively if it were to have any effect at all. Such a scenario is not farfetched when we consider the present situation in Mindanao, where the insurgency problem could spill over the other parts of the country. The determination of the necessity for the calling out power if subjected to unfettered judicial scrutiny could be a veritable prescription for disaster, as such power may be unduly straitjacketed by an injunction or a temporary restraining order every time it is exercised. Thus, it is the unclouded intent of the Constitution to vest upon the President, as Commander-in-Chief of the Armed Forces, full discretion to call forth the military when in his judgment it is necessary to do so in order to prevent or suppress lawless violence, invasion or rebellion. Unless the petitioner can show that the exercise of such discretion was gravely abused, the President's exercise of judgment deserves to be accorded respect from this Court.

EN BANC[G.R. No. 147780. May 10, 2001.]PANFILO LACSON vs. SECRETARY HERNANDO PEREZ

Facts:Faced by an "angry and violent mob armed with explosives, firearms, bladed weapons, clubs,

stones and other deadly weapons" assaulting and attempting to break into Malacañang. President Macapagal-Arroyo issued Proclamation No. 38 on May 1, 2001. In the said proclamation, the President declared that the National Capital Region was in a state of rebellion. She likewise issued General Order No. 1 directing the Armed Forces of the Philippines and the Philippine National Police to suppress the ongoing rebellion. Warrantless arrests of several alleged leaders and promoters of the "rebellion" were thereafter effected. Aggrieved by the warrantless arrests, and the declaration of a "state of rebellion", which allegedly gave a semblance of legality to the arrests,

Page 91: ARTICLE VII the executive department

four related petitions were filed before the Court. The petitions assailed the declaration of a state of rebellion by President Gloria Macapagal-Arroyo and the warrantless arrests allegedly effected by virtue thereof, as having no basis both in fact and in law. Significantly, on May 6, 2001, President Macapagal-Arroyo ordered the lifting of the declaration of a "state of rebellion" in Metro Manila.

Issue:Whether the declaration of a state of rebellion was without factual basis, hence issued with

grave abuse of discretion.

Held:No. Section 18, Article VII of the Constitution expressly provides that "[t]he President shall be

the Commander-in-Chief of all armed forces of the Philippines and whenever it becomes necessary, he may call out such armed forces to prevent or suppress lawless violence, invasion or rebellion . . ." thus, we held in Integrated Bar of the Philippines v. Hon. Zamora, (G.R. No. 141284, August 15, 2000): . . . The factual necessity of calling out the armed forces is not easily quantifiable and cannot be objectively established since matters considered for satisfying the same is a combination of several factors which are not always accessible to the courts. Besides the absence of textual standards that the court may use to judge necessity, information necessary to arrive at such judgment might also prove unmanageable for the courts. Certain pertinent information might be difficult to verify, or wholly unavailable to the courts. In many instances, the evidence upon which the President might decide that there is a need to call out the armed forces may be of a nature not constituting technical proof. On the other hand, the President as Commander-in-Chief has a vast intelligence network to gather information, some of which may be classified as highly confidential or affecting the security of the state. In the exercise of the power to call, on-the-spot decisions may be imperatively necessary in emergency situations to avert great loss of human lives and mass destruction of property. . . . The Court, in a proper case, may look into the sufficiency of the factual basis of the exercise of this power. However, this is no longer feasible at this time, Proclamation No. 38 having been lifted.

In quelling or suppressing the rebellion, the authorities may only resort to warrantless arrests of persons suspected of rebellion, as provided under Section 5, Rule 113 of the Rules of Court, if the circumstances so warrant. The warrantless arrest feared by petitioners is, thus, not based on the declaration of a "state of rebellion."

Moreover, petitioners' contention in G.R. No. 147780 (Lacson Petition), 147781 (Defensor-Santiago Petition), and 147799 (Lumbao Petition) that they are under imminent danger of being arrested without warrant do not justify their resort to the extraordinary remedies of mandamus and prohibition, since an individual subjected to warrantless arrest is not without adequate remedies in the ordinary course of law.

EN BANC[G.R. No. 171396. May 3, 2006.]PROF. RANDOLF S. DAVID vs. GLORIA MACAPAGAL-ARROYO

Page 92: ARTICLE VII the executive department

Facts:Petitioners allege that in issuing Presidential Proclamation No. 1017 (PP 1017) and General Order No. 5 (G.O. No. 5), President Gloria Macapagal-Arroyo committed grave abuse of discretion. On February 24, 2006, as the nation celebrated the 20th Anniversary of the Edsa People Power I, President Arroyo issued PP 1017 declaring a state of national emergency. Respondents stated that the proximate cause behind the executive issuances was the conspiracy among some military officers, leftist insurgents of the New People's Army (NPA), and some members of the political opposition in a plot to unseat or assassinate President Arroyo. They considered the aim to oust or assassinate the President and take-over the reigns of government as a clear and present danger. The mass demonstrations conducted during the commemoration of EDSA were dispersed by the anti-riot police. During the dispersal of the rallyists along EDSA, police arrested (without warrant) petitioner Randolf S. David, a professor at the University of the Philippines and newspaper columnist. Also arrested was his companion, Ronald Llamas, president of party-list Akbayan. petitioners Randolf S. David, et al. assailed PP 1017 on the grounds that (1) it encroaches on the emergency powers of Congress; (2) it is a subterfuge to avoid the constitutional requirements for the imposition of martial law; and (3) it violates the constitutional guarantees of freedom of the press, of speech and of assembly.

Issue:Whether President Arroyo abused her power in declaring a state of rebellion.

Held:No. In Sanlakas v. Executive Secretary, 111 this Court, through Mr. Justice Dante O. Tinga,

held that Section 18, Article VII of the Constitution grants the President, as Commander-in-Chief, a "sequence" of graduated powers. From the most to the least benign, these are: the calling-out power, the power to suspend the privilege of the writ of habeas corpus, and the power to declare Martial Law. Citing Integrated Bar of the Philippines v. Zamora, 112 the Court ruled that the only criterion for the exercise of the calling-out power is that "whenever it becomes necessary," the President may call the armed forces "to prevent or suppress lawless violence, invasion or rebellion." Are these conditions present in the instant cases? As stated earlier, considering the circumstances then prevailing, President Arroyo found it necessary to issue PP 1017. Owing to her Office's vast intelligence network, she is in the best position to determine the actual condition of the country.

Under the calling-out power, the President may summon the armed forces to aid him in suppressing lawless violence, invasion and rebellion. This involves ordinary police action. But every act that goes beyond the President's calling-out power is considered illegal or ultra vires. For this reason, a President must be careful in the exercise of his powers. He cannot invoke a greater power when he wishes to act under a lesser power. There lies the wisdom of our Constitution, the greater the power, the greater are the limitations.It is pertinent to state, however, that there is a distinction between the President's authority to declare a "state of rebellion" (in Sanlakas) and the authority to proclaim a state of national emergency. While President Arroyo's authority to declare a "state of rebellion" emanates from her powers as Chief Executive, the statutory authority cited in Sanlakas was Section 4, Chapter 2, Book II of the Revised Administrative Code of 1987, which provides:

Page 93: ARTICLE VII the executive department

SEC. 4. Proclamations. — Acts of the President fixing a date or declaring a status or condition of public moment or interest, upon the existence of which the operation of a specific law or regulation is made to depend, shall be promulgated in proclamations which shall have the force of an executive order.

4. EMERGENCYPOWERS (Art VI, Sec 23 [2])

EN BANC[G.R. No. 171396. May 3, 2006.]PROF. RANDOLF S. DAVID vs. GLORIA MACAPAGAL-ARROYO

Facts:The pertinent provision of PP 1017 states:

. . . and to enforce obedience to all the laws and to all decrees, orders, and regulations promulgated by me personally or upon my direction; and as provided in Section 17, Article XII of the Constitution do hereby declare a state of national emergency.

The import of this provision is that President Arroyo, during the state of national emergency under PP 1017, can call the military not only to enforce obedience "to all the laws and to all decrees . . ." but also to act pursuant to the provision of Section 17, Article XII which reads:Sec. 17. In times of national emergency, when the public interest so requires, the State may, during the emergency and under reasonable terms prescribed by it, temporarily take over or direct the operation of any privately-owned public utility or business affected with public interest.

Issue:Did the president properly invoked Sec 17 of Art XII of the Constitution incident to her

declaration of a state of national emergency. Does the executive power to declare state of national emergency carries with it the “exercise of emergency power” e.g. the take over of private establishment?

Held:No. During the existence of the state of national emergency, PP 1017 purports to grant the

President, without any authority or delegation from Congress, to take over or direct the operation of any privately-owned public utility or business affected with public interest.

A distinction must be drawn between the President's authority to declare "a state of national emergency" and to exercise emergency powers. To the first, as elucidated by the Court, Section 18, Article VII grants the President such power, hence, no legitimate constitutional objection can be raised. But to the second, manifold constitutional issues arise.

Section 23, Article VI of the Constitution reads:

Page 94: ARTICLE VII the executive department

SEC. 23. (1) The Congress, by a vote of two-thirds of both Houses in joint session assembled, voting separately, shall have the sole power to declare the existence of a state of war.

(2) In times of war or other national emergency, the Congress may, by law, authorize the President, for a limited period and subject to such restrictions as it may prescribe, to exercise powers necessary and proper to carry out a declared national policy. Unless sooner withdrawn by resolution of the Congress, such powers shall cease upon the next adjournment thereof.

It may be pointed out that the second paragraph of the above provision refers not only to war but also to "other national emergency." If the intention of the Framers of our Constitution was to withhold from the President the authority to declare a "state of national emergency" pursuant to Section 18, Article VII (calling-out power) and grant it to Congress (like the declaration of the existence of a state of war), then the Framers could have provided so. Clearly, they did not intend that Congress should first authorize the President before he can declare a "state of national emergency." The logical conclusion then is that President Arroyo could validly declare the existence of a state of national emergency even in the absence of a Congressional enactment.

But the exercise of emergency powers, such as the taking over of privately owned public utility or business affected with public interest, is a different matter. This requires a delegation from Congress.

Considering that Section 17 of Article XII and Section 23 of Article VI, previously quoted, relate to national emergencies, they must be read together to determine the limitation of the exercise of emergency powers.

Generally, Congress is the repository of emergency powers. This is evident in the tenor of Section 23 (2), Article VI authorizing it to delegate such powers to the President. Certainly, a body cannot delegate a power not reposed upon it. However, knowing that during grave emergencies, it may not be possible or practicable for Congress to meet and exercise its powers, the Framers of our Constitution deemed it wise to allow Congress to grant emergency powers to the President, subject to certain conditions, thus:

(1) There must be a war or other emergency.(2) The delegation must be for a limited period only.(3) The delegation must be subject to such restrictions as the Congress may prescribe.(4) The emergency powers must be exercised to carry out a national policy declared by

Congress.

Section 17, Article XII must be understood as an aspect of the emergency powers clause. The taking over of private business affected with public interest is just another facet of the emergency powers generally reposed upon Congress. Thus, when Section 17 states that the "the State may, during the emergency and under reasonable terms prescribed by it, temporarily take over or direct the operation of any privately owned public utility or business affected with public interest," it refers to Congress, not the President. Now, whether or not the President may exercise such power is dependent on whether Congress may delegate it to him pursuant to a law prescribing the reasonable terms thereof.

Page 95: ARTICLE VII the executive department

The order cannot properly be sustained as an exercise of the President's military power as Commander-in-Chief of the Armed Forces. The Government attempts to do so by citing a number of cases upholding broad powers in military commanders engaged in day-to-day fighting in a theater of war. Such cases need not concern us here. Even though "theater of war" be an expanding concept, we cannot with faithfulness to our constitutional system hold that the Commander-in-Chief of the Armed Forces has the ultimate power as such to take possession of private property in order to keep labor disputes from stopping production. This is a job for the nation's lawmakers, not for its military authorities.

Following our interpretation of Section 17, Article XII, invoked by President Arroyo in issuing PP 1017, this Court rules that such Proclamation does not authorize her during the emergency to temporarily take over or direct the operation of any privately owned public utility or business affected with public interest without authority from Congress.

Let it be emphasized that while the President alone can declare a state of national emergency, however, without legislation, he has no power to take over privately-owned public utility or business affected with public interest. The President cannot decide whether exceptional circumstances exist warranting the take over of privately-owned public utility or business affected with public interest. Nor can he determine when such exceptional circumstances have ceased. Likewise, without legislation, the President has no power to point out the types of businesses affected with public interest that should be taken over. In short, the President has no absolute authority to exercise all the powers of the State under Section 17, Article VII in the absence of an emergency powers act passed by Congress.

5. CONTRACTING AND GUARANTEEING FOREIGN LOANS (Sec 20, 21)6. POWER OVER FOREIGN AFFAIRS (Sec 21)

EN BANC[G.R. No. 138570. October 10, 2000.]BAYAN vs. EXECUTIVE SECRETARY RONALDO ZAMORA

Facts:The instant petitions for certiorari and prohibition assailed the agreement forged between

the RP and the USA — THE VISITING FORCES AGREEMENT, which formalized, among others, the use of installations in the Philippine territory by the US military personnel to strengthen their defense and security relationship. On October 5, 1998, President Joseph E. Estrada ratified the VFA, and then transmitted to the Senate his letter of ratification and the VFA for concurrence pursuant to Section 21, Art. VII of the 1987 Constitution. The Senate subsequently approved the VFA by a 2/3 vote of its members.From these consolidated petitions, petitioners — as legislators, non-governmental organizations, citizens and taxpayers — assailed the constitutionality of the VFA and imputed to respondents grave abuse of discretion in ratifying the agreement. Petitioners argue that Section 25, Article XVIII

Page 96: ARTICLE VII the executive department

is applicable considering that the VFA has for its subject the presence of foreign military troops in the Philippines. Respondents, on the contrary, maintain that Section 21, Article VII should apply inasmuch as the VFA is not a basing arrangement but an agreement which involves merely the temporary visits of United States personnel engaged in joint military exercises.

Issue: Is the VFA governed by the provisions of Section 21, Article VII or of Section 25, Article XVIII

of the Constitution?

Held:The 1987 Philippine Constitution contains two provisions requiring the concurrence of the

Senate on treaties or international agreements. Section 21, Article VII, which herein respondents invoke, reads: AcICTS

"No treaty or international agreement shall be valid and effective unless concurred in by at least two-thirds of all the Members of the Senate."

Section 25, Article XVIII, provides:

"After the expiration in 1991 of the Agreement between the Republic of the Philippines and the United States of America concerning Military Bases, foreign military bases, troops, or facilities shall not be allowed in the Philippines except under a treaty duly concurred in by the Senate and, when the Congress so requires, ratified by a majority of the votes cast by the people in a national referendum held for that purpose, and recognized as a treaty by the other contracting State."

Section 21, Article VII deals with treaties or international agreements in general, in which case, the concurrence of at least two-thirds (2/3) of all the Members of the Senate is required to make the subject treaty, or international agreement, valid and binding on the part of the Philippines. This provision lays down the general rule on treaties or international agreements and applies to any form of treaty with a wide variety of subject matter, such as, but not limited to, extradition or tax treaties or those economic in nature. All treaties or international agreements entered into by the Philippines, regardless of subject matter, coverage, or particular designation or appellation, requires the concurrence of the Senate to be valid and effective.

In contrast, Section 25, Article XVIII is a special provision that applies to treaties which involve the presence of foreign military bases, troops or facilities in the Philippines. Under this provision, the concurrence of the Senate is only one of the requisites to render compliance with the constitutional requirements and to consider the agreement binding on the Philippines. Section 25, Article XVIII further requires that "foreign military bases, troops, or facilities" may be allowed in the Philippines only by virtue of a treaty duly concurred in by the Senate, ratified by a majority of the votes cast in a national referendum held for that purpose if so required by Congress, and recognized as such by the other contracting state.

It is our considered view that both constitutional provisions, far from contradicting each other, actually share some common ground. These constitutional provisions both embody phrases

Page 97: ARTICLE VII the executive department

in the negative and thus, are deemed prohibitory in mandate and character. In particular, Section 21 opens with the clause "No treaty . . .," and Section 25 contains the phrase "shall not be allowed." Additionally, in both instances, the concurrence of the Senate is indispensable to render the treaty or international agreement valid and effective.

To our mind, the fact that the President referred the VFA to the Senate under Section 21, Article VII, and that the Senate extended its concurrence under the same provision, is immaterial. For in either case, whether under Section 21, Article VII or Section 25, Article XVIII, the fundamental law is crystalline that the concurrence of the Senate is mandatory to comply with the strict constitutional requirements.

On the whole, the VFA is an agreement which defines the treatment of United States troops and personnel visiting the Philippines. It provides for the guidelines to govern such visits of military personnel, and further defines the rights of the United States and the Philippine government in the matter of criminal jurisdiction, movement of vessel and aircraft, importation and exportation of equipment, materials and supplies.

Undoubtedly, Section 25, Article XVIII, which specifically deals with treaties involving foreign military bases, troops, or facilities, should apply in the instant case . To a certain extent and in a limited sense, however, the provisions of Section 21, Article VII will find applicability with regard to the issue and for the sole purpose of determining the number of votes required to obtain the valid concurrence of the Senate, as will be further discussed hereunder.

Moreover, it is specious to argue that Section 25, Article XVIII is inapplicable to mere transient agreements for the reason that there is no permanent placing of structure for the establishment of a military base. On this score, the Constitution makes no distinction between "transient" and "permanent." Certainly, we find nothing in Section 25, Article XVIII that requires foreign troops or facilities to be stationed or placed permanently in the Philippines.

In like manner, we do not subscribe to the argument that Section 25, Article XVIII is not controlling since no foreign military bases, but merely foreign troops and facilities, are involved in the VFA. Notably, a perusal of said constitutional provision reveals that the proscription covers "foreign military bases, troops, or facilities." Stated differently, this prohibition is not limited to the entry of troops and facilities without any foreign bases being established. The clause does not refer to "foreign military bases, troops, or facilities" collectively but treats them as separate and independent subjects. The use of comma and the disjunctive word "or" clearly signifies disassociation and independence of one thing from the others included in the enumeration, 28 such that, the provision contemplates three different situations — a military treaty the subject of which could be either (a) foreign bases, (b) foreign troops, or (c) foreign facilities — any of the three standing alone places it under the coverage of Section 25, Article XVIII.

At this juncture, we shall then resolve the issue of whether or not the requirements of Section 25 were complied with when the Senate gave its concurrence to the VFA.

Section 25, Article XVIII disallows foreign military bases, troops, or facilities in the country, unless the following conditions are sufficiently met, viz: (a) it must be under a treaty; (b) the treaty must be duly concurred in by the Senate and, when so required by Congress, ratified by a majority of the votes cast by the people in a national referendum; and (c) recognized as a treaty by the other contracting state.

Page 98: ARTICLE VII the executive department

There is no dispute as to the presence of the first two requisites in the case of the VFA.As to the matter of voting, Section 21, Article VII particularly requires that a treaty or

international agreement, to be valid and effective, must be concurred in by at least two-thirds of all the members of the Senate. On the other hand, Section 25, Article XVIII simply provides that the treaty be "duly concurred in by the Senate."

Applying the foregoing constitutional provisions, a two-thirds vote of all the members of the Senate is clearly required so that the concurrence contemplated by law may be validly obtained and deemed present. While it is true that Section 25, Article XVIII requires, among other things, that the treaty — the VFA, in the instant case — be a "duly concurred in by the Senate," it is very true however that said provision must be related and viewed in light of the clear mandate embodied in Section 21, Article VII, which in more specific terms, requires that the concurrence of a treaty, or international agreement, be made by a two-thirds vote of all the members of the Senate. Indeed, Section 25, Article XVIII must not be treated in isolation to Section 21, Article, VII.

This Court is of the firm view that the phrase "recognized as a treaty" means that the other contracting party accepts or acknowledges the agreement as a treaty. 32 To require the other contracting state, the United States of America in this case, to submit the VFA to the United States Senate for concurrence pursuant to its Constitution, 33 is to accord strict meaning to the phrase.

It is inconsequential whether the United States treats the VFA only as an executive agreement because, under international law, an executive agreement is as binding as a treaty. 35 To be sure, as long as the VFA possesses the elements of an agreement under international law, the said agreement is to be taken equally as a treaty.

In international law, there is no difference between treaties and executive agreements in their binding effect upon states concerned, as long as the negotiating functionaries have remained within their powers. 38 International law continues to make no distinction between treaties and executive agreements: they are equally binding obligations upon nations.

By constitutional fiat and by the intrinsic nature of his office, the President, as head of State, is the sole organ and authority in the external affairs of the country. In many ways, the President is the chief architect of the nation's foreign policy; his "dominance in the field of foreign relations is (then) conceded." 51 Wielding vast powers and influence, his conduct in the external affairs of the nation, as Jefferson describes, is "executive altogether."

As regards the power to enter into treaties or international agreements, the Constitution vests the same in the President, subject only to the concurrence of at least two-thirds vote of all the members of the Senate. In this light, the negotiation of the VFA and the subsequent ratification of the agreement are exclusive acts which pertain solely to the President, in the lawful exercise of his vast executive and diplomatic powers granted him no less than by the fundamental law itself. Into the field of negotiation the Senate cannot intrude, and Congress itself is powerless to invade it. Consequently, the acts or judgment calls of the President involving the VFA — specifically the acts of ratification and entering into a treaty and those necessary or incidental to the exercise of such principal acts — squarely fall within the sphere of his constitutional powers and thus, may not be validly struck down, much less calibrated by this Court, in the absence of clear showing of grave abuse of power or discretion.

It is the Court's considered view that the President, in ratifying the VFA and in submitting the same to the Senate for concurrence, acted within the confines and limits of the powers vested in

Page 99: ARTICLE VII the executive department

him by the Constitution. It is of no moment that the President, in the exercise of his wide latitude of discretion and in the honest belief that the VFA falls within the ambit of Section 21, Article VII of the Constitution, referred the VFA to the Senate for concurrence under the aforementioned provision. Certainly, no abuse of discretion, much less a grave, patent and whimsical abuse of judgment, may be imputed to the President in his act of ratifying the VFA and referring the same to the Senate for the purpose of complying with the concurrence requirement embodied in the fundamental law. In doing so, the President merely performed a constitutional task and exercised a prerogative that chiefly pertains to the functions of his office. Even if he erred in submitting the VFA to the Senate for concurrence under the provisions of Section 21 of Article VII, instead of Section 25 of Article XVIII of the Constitution, still, the President may not be faulted or scarred, much less be adjudged guilty of committing an abuse of discretion in some patent, gross, and capricious manner.

EN BANC[G.R. No. 158088. July 6, 2005.]SENATOR AQUILINO PIMENTEL vs. OFFICE OF THE EXECUTIVE SECRETARY

Facts:This is a petition for mandamus filed by petitioners to compel the Office of the Executive

Secretary and the Department of Foreign Affairs to transmit the signed copy of the Rome Statute of the International Criminal Court to the Senate of the Philippines for its concurrence in accordance with Section 21, Article VII of the 1987 Constitution. It is the theory of the petitioners that ratification of a treaty, under both domestic law and international law, is a function of the Senate. Hence, it is the duty of the executive department to transmit the signed copy of the Rome Statute to the Senate to allow it to exercise its discretion with respect to ratification of treaties. Moreover, petitioners submit that the Philippines has a ministerial duty to ratify the Rome Statute under treaty law and customary international law. Petitioners invoke the Vienna Convention on the Law of Treaties enjoining the states to refrain from acts which would defeat the object and purpose of a treaty when they have signed the treaty prior to ratification unless they have made their intention clear not to become parties to the treaty. On the other hand, respondents argue that the executive department has no duty to transmit the Rome Statute to the Senate for concurrence.

Issue:Whether the Executive Secretary and the Department of Foreign Affairs have a ministerial

duty to transmit to the Senate the copy of the Rome Statute signed by a member of the Philippine Mission to the United Nations even without the signature of the President. Does the power to ratify treaty belong to the Senate?

Held:No. In our system of government, the President, being the head of state, is regarded as the

sole organ and authority in external relations and is the country's sole representative with foreign nations. As the chief architect of foreign policy, the President acts as the country's mouthpiece with respect to international affairs. Hence, the President is vested with the authority to deal with

Page 100: ARTICLE VII the executive department

foreign states and governments, extend or withhold recognition, maintain diplomatic relations, enter into treaties, and otherwise transact the business of foreign relations. In the realm of treaty-making, the President has the sole authority to negotiate with other states.

Nonetheless, while the President has the sole authority to negotiate and enter into treaties , the Constitution provides a limitation to his power by requiring the concurrence of 2/3 of all the members of the Senate for the validity of the treaty entered into by him. Section 21, Article VII of the 1987 Constitution provides that "no treaty or international agreement shall be valid and effective unless concurred in by at least two-thirds of all the Members of the Senate."

The participation of the legislative branch in the treaty-making process was deemed essential to provide a check on the executive in the field of foreign relations. 14 By requiring the concurrence of the legislature in the treaties entered into by the President, the Constitution ensures a healthy system of checks and balance necessary in the nation's pursuit of political maturity and growth.

Justice Isagani Cruz, in his book on International Law, describes the treaty-making process in this wise:

The usual steps in the treaty-making process are: negotiation, signature, ratification, and exchange of the instruments of ratification. The treaty may then be submitted for registration and publication under the U.N. Charter, although this step is not essential to the validity of the agreement as between the parties.

Negotiation may be undertaken directly by the head of state but he now usually assigns this task to his authorized representatives. These representatives are provided with credentials known as full powers, which they exhibit to the other negotiators at the start of the formal discussions. It is standard practice for one of the parties to submit a draft of the proposed treaty which, together with the counter-proposals, becomes the basis of the subsequent negotiations. The negotiations may be brief or protracted, depending on the issues involved, and may even "collapse" in case the parties are unable to come to an agreement on the points under consideration.

If and when the negotiators finally decide on the terms of the treaty, the same is opened for signature. This step is primarily intended as a means of authenticating the instrument and for the purpose of symbolizing the good faith of the parties; but, significantly, it does not indicate the final consent of the state in cases where ratification of the treaty is required. The document is ordinarily signed in accordance with the alternat, that is, each of the several negotiators is allowed to sign first on the copy which he will bring home to his own state.

Ratification, which is the next step, is the formal act by which a state confirms and accepts the provisions of a treaty concluded by its representatives. The purpose of ratification is to enable the contracting states to examine the treaty more closely and to give them an opportunity to refuse to be bound by it should they find it inimical to their interests. It is for this reason that most treaties are made subject to the scrutiny and consent of a department of the government other than that which negotiated them.

The last step in the treaty-making process is the exchange of the instruments of ratification, which usually also signifies the effectivity of the treaty unless a different date has been agreed upon by the parties. Where ratification is dispensed with and no effectivity clause is embodied in the treaty, the instrument is deemed effective upon its signature.

Page 101: ARTICLE VII the executive department

Petitioners' arguments equate the signing of the treaty by the Philippine representative with ratification. It should be underscored that the signing of the treaty and the ratification are two separate and distinct steps in the treaty-making process. As earlier discussed, the signature is primarily intended as a means of authenticating the instrument and as a symbol of the good faith of the parties. It is usually performed by the state's authorized representative in the diplomatic mission. Ratification, on the other hand, is the formal act by which a state confirms and accepts the provisions of a treaty concluded by its representative. It is generally held to be an executive act, undertaken by the head of the state or of the government.

Petitioners' submission that the Philippines is bound under treaty law and international law to ratify the treaty which it has signed is without basis. The signature does not signify the final consent of the state to the treaty. It is the ratification that binds the state to the provisions thereof. In fact, the Rome Statute itself requires that the signature of the representatives of the states be subject to ratification, acceptance or approval of the signatory states. Ratification is the act by which the provisions of a treaty are formally confirmed and approved by a State. By ratifying a treaty signed in its behalf, a state expresses its willingness to be bound by the provisions of such treaty. After the treaty is signed by the state's representative, the President, being accountable to the people, is burdened with the responsibility and the duty to carefully study the contents of the treaty and ensure that they are not inimical to the interest of the state and its people. Thus, the President has the discretion even after the signing of the treaty by the Philippine representative whether or not to ratify the same. The Vienna Convention on the Law of Treaties does not contemplate to defeat or even restrain this power of the head of states. If that were so, the requirement of ratification of treaties would be pointless and futile. It has been held that a state has no legal or even moral duty to ratify a treaty which has been signed by its plenipotentiaries. There is no legal obligation to ratify a treaty, but it goes without saying that the refusal must be based on substantial grounds and not on superficial or whimsical reasons. Otherwise, the other state would be justified in taking offense.

It should be emphasized that under our Constitution, the power to ratify is vested in the President, subject to the concurrence of the Senate. The role of the Senate, however, is limited only to giving or withholding its consent, or concurrence, to the ratification. Hence, it is within the authority of the President to refuse to submit a treaty to the Senate or, having secured its consent for its ratification, refuse to ratify it. Although the refusal of a state to ratify a treaty which has been signed in its behalf is a serious step that should not be taken lightly, such decision is within the competence of the President alone, which cannot be encroached by this Court via a writ of mandamus. This Court has no jurisdiction over actions seeking to enjoin the President in the performance of his official duties. The Court, therefore, cannot issue the writ of mandamus prayed for by the petitioners as it is beyond its jurisdiction to compel the executive branch of the government to transmit the signed text of Rome Statute to the Senate.

i.1. Deportation of undesirable aliens

Page 102: ARTICLE VII the executive department

[G.R. No. L-23846. September 9, 1977.]GO TEK vs. DEPORTATION BOARD

Facts:On March 3, 1964 the chief prosecutor of the Deportation Board filed a complaint against Go

Tek, a Chinaman residing at Ilagan, Isabela and 1208-B, Misericordia Street, Sta. Cruz, Manila. It was alleged in the complaint that in December, 1963 certain agents of the National Bureau of Investigation (NBI) searched an office located at 1439 O'Donnel Street, Sta. Cruz, Manila, believed to be the headquarters of a guerilla unit of the "Emergency Intelligence Section, Army of the United States", and that among those arrested thereat was Go Tek, an alleged sector commander and intelligence and record officer of that guerilla unit. It was further alleged that fake dollar checks were found in Go Tek's possession and that, therefore, he had violated article 168 of the Revised Penal Code and rendered himself an undesirable alien.

The prosecutor prayed that after trial the Board should recommend to the President of the Philippines the immediate deportation of Go Tek as an undesirable alien. Go Tek filed a motion to dismiss on the ground that the complaint was premature because there was a pending case against him in the city fiscal's office of Manila for violation of article 168 (I.S. 64-7267). He contended that the Board had no jurisdiction to try the case in view of the obiter dictum in Qua Chee Gan vs. Deportation Board, 118 Phil. 868, 875, that the President may deport aliens only on the grounds specified in the law. On the other hand, The Board reasoned out that a criminal conviction is not a prerequisite before the State may exercise its right to deport an undesirable alien and that the Board is only a fact-finding body whose function is to make a report and recommendation to the President in whom is lodged the exclusive power to deport an alien or dismiss a deportation proceeding.

Issues:1. Whether the president can deport undesirable alien only on the ground specified by law.2. Is conviction of a crime necessary to warrant deportation?

Held:

First Issue:No. The President's power to deport aliens and the investigation of aliens subject to

deportation are provided for in the following provisions of the Revised Administrative Code:SEC. 69. Deportation of subject of foreign power. — A subject of a foreign power

residing in the Philippine Islands shall not be deported, expelled, or excluded from said Islands or repatriated to his own country by the Governor-General except upon prior investigation, conducted by said Executive or his authorized agent, of the ground upon which such action is contemplated. In such case the person concerned shall be informed of the charge or charges against him and he shall be allowed not less than three days for the preparation of his defense. He shall also have the right to be heard by himself or counsel, to produce witnesses in his own behalf, and to cross-examine the opposing witnesses."

Page 103: ARTICLE VII the executive department

On the other hand, section 37 of the Immigration Law provides that certain aliens may be arrested upon the warrant of the Commissioner of Immigration or of any other officer designated by him for the purpose and deported upon the Commissioner's warrant "after a determination by the Board of Commissioners of the existence of the ground for deportation as charged against the alien." Thirteen classes of aliens who may be deported by the Commissioner are specified in section 37 (See PO Siok Pin vs. Vivo, L-24792, February 14, 1975, 62 SCRA 363, 368).

So, under existing law, the deportation of an undesirable alien may be effected (1) by order of the President, after due investigation, pursuant to section 69 of the Revised Administrative Code and (2) by the Commissioner of Immigration, upon recommendation of the Board of Commissioners under section 37 of the Immigration Law (Qua Chee Gan vs. Deportation Board, supra).

The State has the inherent power to deport undesirable aliens (Chuoco Tiaco vs. Forbes, 228 U.S. 549, 57 L. Ed. 960, 40 Phil. 1122, 1125). That power may be exercised by the Chief Executive "when he deems such action necessary for the peace and domestic tranquility of the nation": Justice Johnson's opinion is that when the Chief Executive finds that there are aliens whose continued presence in the country is injurious to the public interest, "he may, even in the absence of express law, deport them". (Forbes vs. Chuoco Tiaco and Crossfield, 16 Phil. 534, 568, 569; In re McCulloch Dick, 38 Phil. 41).

"The right of a country to expel or deport aliens because their continued presence is detrimental to public welfare is absolute and unqualified" (Tiu Chun Hai and Go Tam vs. Commissioner of Immigration and the Director of NBI, 104 Phil. 949, 956).

Section 69 and Executive Order No. 398, reorganizing the Deportation Board, do not specify the grounds for deportation. Paragraph 1(a) of Executive Order No. 398 merely provides that "the Deportation Board, motu proprio or upon complaint of any person, is authorized to conduct investigations in the manner prescribed in section 69 of the Revised Administrative Code to determine whether a subject of a foreign power residing in the Philippines is an undesirable alien or not, and thereafter to recommend to the President of the Philippines the deportation of such alien."

As observed by Justice Labrador, there is no legal nor constitutional provision defining the power to deport aliens because the intention of the law is to grant the Chief Executive "full discretion to determine whether an alien's residence in the country is so undesirable as to affect or injure the security, welfare or interest of the state. The adjudication of facts upon which deportation is predicated also devolves on the Chief Executive whose decision is final and executory." (Tan Tong vs. Deportation Board, 96 Phil. 934, 936; Tan Sin vs. Deportation Board, 104 Phil. 868, 872).

It has been held that the Chief Executive is the sole and exclusive judge of the existence of facts which warrant the deportation of aliens, as disclosed in an investigation conducted in accordance with section 69. No other tribunal is at liberty to reexamine or to controvert the sufficiency of the evidence on which he acted. (Martin vs. Mott, 12 Wheat., 19, 31, cited in re McCulloch Dick, 38 Phil. 41, 62).

In the Dick case it was noted "that every alien forfeits his right of asylum in the country in which he resides, in the absence of treaty provisions to the contrary, when his conduct or his mode

Page 104: ARTICLE VII the executive department

of life renders his presence there inimical to the public interests". "The reasons may be summed up and condensed in a single word: the public interest of the State." (38 Phil. 41, 47, 100).

Second Issue:No. "It is fundamental that an executive order for deportation is not dependent on a prior

judicial conviction in a criminal case" (Ang Beng vs. Commissioner of Immigration, 100 Phil. 801, 803). Thus, it was held that the fact that an alien has been acquitted in a criminal proceeding of the particular charge does not prevent the deportation of such alien based on the same charge. Such acquittal does not constitute res judicata in the deportation proceedings. Conviction of a crime is not necessary to warrant deportation. (3 C.J.S. 743, note 40, citing Lewis vs. Frick, 233 U.S. 291, 58 L. Ed. 967 and U.S. ex. rel. Mastoras vs. McCandless, 61 F. 2nd 366; Tama Miyake vs. U.S. 257 F. 732).

And in the Tan Tong case, supra, it was ruled that the Deportation Board could take cognizance of the charge of illegal importation against an alien, as a ground for deportation, even if he has not been convicted of that offense.

7. POWER OVER LEGISLATIONj.1. to address congress (Sec 23)j.2. Preparation and submission of budget

EN BANC[G.R. No. 132988. July 19, 2000.]AQUILINO Q. PIMENTEL, JR. vs. Hon. ALEXANDER AGUIRRE

Facts:On December 27, 1997, the then President of the Philippines, Fidel V. Ramos, issued

Administrative Order (AO) 372. Subsequently, on December 10, 1998, President Joseph E. Estrada issued AO 43, amending Section 4 of AO 372, by reducing to five percent (5%) the amount of internal revenue allotment (IRA) to be withheld from local government units (LGUs.) In this original petition for certiorari and prohibition before the Supreme Court, petitioner seeks to annul Section 1 of AO 372, insofar as it requires LGUs to reduce their expenditures by 25% of their authorized regular appropriations for non-personal services; and to enjoin respondents from implementing Section 4 of the Order, which withholds a portion of their internal revenue allotments.

Issue: Does AO 43 violate the local fiscal autonomy?

Held:

Page 105: ARTICLE VII the executive department

Yes. Under existing law, local government units, in addition to having administrative autonomy in the exercise of their functions, enjoy fiscal autonomy as well. Fiscal autonomy means that local governments have the power to create their own sources of revenue in addition to their equitable share in the national taxes released by the national government, as well as the power to allocate their resources in accordance with their own priorities. It extends to the preparation of their budgets, and local officials in turn have to work within the constraints thereof. They are not formulated at the national level and imposed on local governments, whether they are relevant to local needs and resources or not. Hence, the necessity of a balancing of viewpoints and the harmonization of proposals from both local and national officials, who in any case are partners in the attainment of national goals.

Local fiscal autonomy does not however rule out any manner of national government intervention by way of supervision, in order to ensure that local programs, fiscal and otherwise, are consistent with national goals. Significantly, the President, by constitutional fiat, is the head of the economic and planning agency of the government, primarily responsible for formulating and implementing continuing, coordinated and integrated social and economic policies, plans and programs 26 for the entire country. However, under the Constitution, the formulation and the implementation of such policies and programs are subject to "consultations with the appropriate public agencies, various private sectors, and local government units." The President cannot do so unilaterally.

Consequently, the Local Government Code provides: 27

". . . [I]n the event the national government incurs an unmanaged public sector deficit, the President of the Philippines is hereby authorized, upon the recommendation of [the] Secretary of Finance, Secretary of the Interior and Local Government and Secretary of Budget and Management, and subject to consultation with the presiding officers of both Houses of Congress and the presidents of the liga, to make the necessary adjustments in the internal revenue allotment of local government units but in no case shall the allotment be less than thirty percent (30%) of the collection of national internal revenue taxes of the third fiscal year preceding the current fiscal year . . ."

There are therefore several requisites before the President may interfere in local fiscal matters: (1) an unmanaged public sector deficit of the national government; (2) consultations with the presiding officers of the Senate and the House of Representatives and the presidents of the various local leagues; and (3) the corresponding recommendation of the secretaries of the Department of Finance, Interior and Local Government, and Budget and Management. Furthermore, any adjustment in the allotment shall in no case be less than thirty percent (30%) of the collection of national internal revenue taxes of the third fiscal year preceding the current one.

Petitioner points out that respondents failed to comply with these requisites before the issuance and the implementation of AO 372. At the very least, they did not even try to show that the national government was suffering from an unmanageable public sector deficit. Neither did they claim having conducted consultations with the different leagues of local governments. Without these requisites, the President has no authority to adjust, much less to reduce, unilaterally the LGU's internal revenue allotment.

Page 106: ARTICLE VII the executive department

The solicitor general insists, however, that AO 372 is merely directory and has been issued by the President consistent with his power of supervision over local governments. It is intended only to advise all government agencies and instrumentalities to undertake cost-reduction measures that will help maintain economic stability in the country, which is facing economic difficulties. Besides, it does not contain any sanction in case of noncompliance. Being merely an advisory, therefore, Section 1 of AO 372 is well within the powers of the President. Since it is not a mandatory imposition, the directive cannot be characterized as an exercise of the power of control.

Section 4 of AO 372 cannot, however, be upheld. A basic feature of local fiscal autonomy is the automatic release of the shares of LGUs in the national internal revenue. This is mandated by no less than the Constitution. 28 The Local Government Code 29 specifies further that the release shall be made directly to the LGU concerned within five (5) days after every quarter of the year and "shall not be subject to any lien or holdback that may be imposed by the national government for whatever purpose." 30 As a rule, the term "shall" is a word of command that must be given a compulsory meaning. 31 The provision is, therefore, imperative.

Section 4 of AO 372, however, orders the withholding, effective January 1, 1998, of 10 percent of the LGUs' IRA "pending the assessment and evaluation by the Development Budget Coordinating Committee of the emerging fiscal situation" in the country. Such withholding clearly contravenes the Constitution and the law. Although temporary, it is equivalent to a holdback which means "something held back or withheld, often temporarily." Hence, the "temporary" nature of the retention by the national government does not matter. Any retention is prohibited.

In sum, while Section 1 of AO 372 may be upheld as an advisory effected in times of national crisis, Section 4 thereof has no color of validity at all. The latter provision effectively encroaches on the fiscal autonomy of local governments. Concededly, the President was well-intentioned in issuing his Order to withhold the LGUs' IRA, but the rule of law requires that even the best intentions must be carried out within the parameters of the Constitution and the law. Verily, laudable purposes must be carried out by legal methods.